Patent Bar good set 5

¡Supera tus tareas y exámenes ahora con Quizwiz!

The specification recites a nozzle for a highly caustic acid spray for decalcifying fresh water intake ducts at a water treatment plant. The nozzle is coated so that the nozzle itself does not dissolve in use. The coating is made from an epoxy latex combination, wherein the epoxy is mixed at the rate of 2-3 parts epoxy for every 1-2 parts latex. The original claim is as follows. 1. A coating for a high pressure nozzle assembly, comprising: a mixture of epoxy and latex, where latex is in the range of 1 - 2 parts for every 2 - 3 parts of epoxy by weight in the mixture. The examiner has found a prior art reference under 35 USC 102 (b) which discloses a coating mixture for a manure spreader nozzle, which is 2 parts epoxy and 2 parts latex by weight in the mixture. Your client has done further testing and determined that, in fact, an equal mixture of epoxy and latex doesn't work nearly as well, if at all, for the highly caustic decalcification spray. Testing does confirm, however, that a 3 part epoxy to 1 part latex mixture is very good and greatly extends the service life of the nozzle. You propose to amend the claim to overcome the reference, and propose the following. A Cancel Claim 1 and submit Claim 2: (New) A coating for a high pressure nozzle assembly, comprising: a mixture of epoxy and latex, where latex is 1 part for every 3 parts of epoxy by weight in the mixture. B In claim 1, line 2 delete, "in the range of 1-2 parts for every 2-", and insert "one part of said latex for every--." C In claim 1, line 2, delete "latex is in the range of 1-2 parts for every 2-", and insert "-said latex being at one part and--." D Claim 1. (Currently Amended) A coating for a high pressure nozzle assembly for dispensing an acid spray, comprising: a mixture of epoxy and latex, where said latex is in the range of 1 [-2] part[s] for every 3 parts of epoxy by weight in the mixture. E A and D

(A) is correct because it appropriately amends the application to take into account the prior art located by adding new claim 2. MPEP 714. (B) and (C) are incorrect because they are examples of interlineation, which is no longer an acceptable format for amending claims. (D) is inappropriate because it seems to delete by using single brackets. (E) is incorrect because (D) is incorrect.

A provisional application was filed on June 1, 2012 relating to an improved golf club. The drawings fully illustrate the exterior appearance of the club. The clubs were first sold in the United States on June 23, 2011. Today, August 27, 2012, your client contacts you and tells you she wants to obtain design patent protection on the club. Your advice would be: A No protection can be obtained because it is barred by 102(b). B A design patent can be filed and claim benefit of the provisional filing date. C Designs on golf clubs are not patentable subject matter. D File a utility application claiming benefit of the filing date of the provisional and file a design application as a divisional of the Utility application. E Do nothing.

(A) is correct.

John Logan filed a provisional application on March 22, 2012. On Saturday March 22, 2013, he sent by Express Mail a utility patent application to the USPTO containing the same disclosure as the provisional application. He complied fully with the requirements for filing an application using Express Mail. This application: A Has the benefit of the provisional application filing date of March 22, 2012 in determining whether something is prior art under 35 USC Section 102. B Takes no benefit because it was filed on a Saturday, Sunday or holiday when the PTO is closed. C Will be given a filing date of March 24, 2013. D Has the benefit of the provisional application filing date provided that a proper petition is filed with it. E None of the above.

(A) is correct.

Which of the following qualify as prior art under 102? A A sale by the inventor more than one year before the filing of the application. B A public use by the inventor within one year of filing the application. C A publication by the inventor within one year of filing the application. D A publication by an associate of the inventor of the inventor's work within one year of filing an application. E All the above qualify as prior art

(A) is correct. A sale, regardless of by whom, more than one year before the filing date, is unavoidable prior art. Uses and sales within one year of the filing date are not prior art if they are made by the inventor, either directly or indirectly. Thus, (B), (C) and (D)...and thus (E)...are incorrect.

Independently of each other, Oscar and Felix invented identical ergonomic golf clubs in the United States. Oscar filed his patent application in the U.S. on April 12, 2012, and a patent issued on March 12, 2013. On April 10, 2013, Felix filed his patent application in the USPTO claiming golf clubs identical to those claimed in Oscar's patent. There is no common assignee. Under which of the following provisions of 35 U.S.C. 102 is the U.S. patent to Oscar prior art with regard to the golf club claimed by Felix? A 35 U.S.C. 102(a) B 35 U.S.C. 102(b) C 35 U.S.C. 102(e) D A and C E A, B, and C

(A) is correct. AIA 102(a)(1). Based on the filing date of Felix's application, the law that will apply to Felix is AIA 102. AIA 102(a)(1) prevents a patent when the claimed invention was patented (in this case by Oscar) before the effective filing date of the claimed invention. Thus, Felix will have his claim rejected under 102(a)(1). (B) is incorrect because AIA 102(b) is not a basis for a rejection, but rather details exceptions. (C) is incorrect because there no longer is a section 102(e) in AIA 102. (D) and (E) are similarly incorrect.

Stanley Thaddeus Wojciehowicz is the inventor of an improved, combination coffeemaker/motion detector, which is useful for making better than mediocre coffee and doubles as a home security system. He first files a patent application in Poland on February 28, 2012. Subsequently, he files a patent application in the U.S. on February 27, 2013, which claims the benefit of his Polish filing date. The Polish application publishes on August 28, 2013, and the U.S. patent application publishes on August 29, 2013. The Polish patent is granted on March 14, 2014, and the U.S. patent is granted on October 22, 2014. On November 1, 2014, Carl Levitt files an application claiming a combination coffeemaker/motion detector with integrated radio. Which of the follow dates is the earliest date the Wojociehowicz disclosure will be prior art to Levitt? A February 28, 2012. B February 27, 2013. C August 28, 2013. D March 14, 2014. E October 22, 2014

(A) is correct. AIA 35 U.S.C. 102(d) provides that if the U.S. patent, U.S. patent application publication, or WIPO-published application claims priority to one or more prior-filed foreign or international applications under 35 U.S.C. 119 or 365, the patent or published application was effectively filed on the filing date of the earliest such application that describes the subject matter. Therefore, if the subject matter relied upon is described in the application to which there is a priority or benefit claim, a U.S. patent, a U.S. patent application publication, or WIPO-published application is effective as prior art as of the filing date of the earliest such application, regardless of where filed. Thus, the Polish filing date of February 28, 2012, is the earliest date the Wojociehowicz disclosure will be prior art to Levitt.

Joan Blondel is an employee of the Department of Agriculture in the Topeka office. Her specialty is corn and she has an idea for a new technique for plowing which she wishes to patent. Her underling Helmut Schmidt, at her direction, used the technique during the 2018 plowing season and carefully compared results with similar fields this year and in other years. The results were ready on June 23, 2018 and were a stunning and surprising proof that the invention works. The results are published one year later on June 23, 2019. When she mentioned the invention to her father one night at dinner in September 2019, he suggested that she might get even better results if she plowed against the grain on slopes. There was not sufficient interest to test whether her father's idea worked, but she decided to include it in the patent application which was filed in January 2020. Claim 14 specifically sets forth this concept. The examiner rejects all of the claims citing a patent to Jimbo Plowboy which issued on August 2, 2019, and which was filed on July 5, 2018. Plowboy discloses but does not claim either Joan's or her father's invention. Which of the following is true? A All of the claims are barred by the Plowboy patent under 102(a)(2). B All of the claims are barred by the Plowboy patent under 102(a)(1). C All of the claims can be patented because they are entitled to a date of invention based on a reduction to practice prior to the filing date of Plowboy. D Joan could remove the rejection on Plowboy by acquiring the patent so that her application and the patent would then be commonly owned. E None of the above.

(A) is correct. Joan cannot overcome the filing date of Plowboy under 102(a)(2). (B) is incorrect because Joan's publication can pre-date the issue date of Plowboy under 102(a)(1). (C) is incorrect because Dad's claims are not entitled to the benefit of any "date of invention" under AIA. (D) is incorrect. It has to be commonly owned at the time of filing, not afterwards. (E) is incorrect because (A) is correct.

Which of the following can you do as a matter of right in a non-provisional patent application? A File a substitute specification if accompanied by a statement of no new matter and a marked up copy. B Conduct an interview after a Final Rejection. C Appeal any Official Action which rejects the claims. D Submit an IDS after a First Official Action upon a showing that it could not have been submitted earlier. E Withdraw from representing a client at any time before the six-month deadline for responding to an Official Action.

(A) is correct. See 37 C.F.R. 1.125(b)

You are prosecuting an application in which the Patent Examiner rejected all of the claims as anticipated and set a three-month shortened period for response. The non-final action was dated June 3, 2017 and you received the action on June 12, 2017. On Monday September 5, 2017 you filed a proper response to the action using the Certificate of Mailing Procedure. On January 5, 2018 you receive a Notice of Abandonment from the Patent Office for failure to prosecute. Which of the following would be the most appropriate response to the notice: A Promptly advise the Patent Office in writing that a response was mailed, and include a copy of your response and establish that the response was properly and timely mailed. B Advise your client of the misfortune and explain that nothing can be done. C File a petition to revive the application, include a copy of the response, and a petition to revive the application as unavoidably abandoned. D Send the Patent Office another copy of the response. E File an original copy of your response together with a four-month extension of time.

(A) is correct. The Office Action was dated June 3, 2017, which means the shortened statutory period expired on September 3, 2017. However, we know from the question that Monday was September 5, 2017, which means that September 3 was a Saturday. When a filing is due on a Saturday, Sunday or Federal holiday in the District of Columbia, the applicant has until the next day that the USPTO is open within which to timely file. Therefore, filing on Monday, September 5, 2017, was timely and the application was not abandoned. The USPTO made an error issuing a Notice of Abandonment. Also notice that this particular response could have been timely filed even on Tuesday, September 6, 2017, because the first Monday in September is a Federal Holiday (i.e., Labor Day). (B) is incorrect because there is something that you can do, which is to advise the USPTO of the erroneous Notice of Abandonment. (C) is incorrect because the application never went abandoned. (D) is incorrect because, as it stands, the USPTO believes the application went abandoned. Furthermore, you timely filed, so filing another response isn't going to be helpful. (E) is incorrect because there is no such thing as a 4-month extension to respond to an Office Action. The shortened statutory period to respond to an Office Action is 3 months and the statutory period is 6 months; therefore, you can never obtain more than a 3-month extension to respond to an Office Action. Furthermore, you timely responded, so all you have to do is notify the USPTO of their error.

In an application filed with an ADS, an inventor's oath or declaration must: A Identify the inventor by his or her legal name. B Identify the PTO-provided serial number of the application to which the oath or declaration is directed. C Include the mailing address where the inventor customarily receives mail, and residential address if an inventor lives at a location different from where the inventor customarily receives mail. D Acknowledge the inventor's duty to disclose to the Patent Office information material to patentability. E All of the above.

(A) is correct. The balance of the necessary information that may otherwise be in the oath can be found in the ADS. Nationality is no longer required. In addition, the acknowledgement of the duty of disclosure is no longer required as included text, although the duty itself remains.

Patrick Wilson is the inventor of a collapsible motorcycle hammock, which incorporates a hammock that is releasably attached to a motorcycle. An International Patent Application is filed in the United States Patent Office on February 28, 2013. Wilson intends to file a US nonprovisional patent application claiming the benefit of his earlier filed International Application, but is tragically involved in horrible accident on the night of January 3, 2014. While he was sleeping, the hammock containing his motorcycle fell, which caused the motorcycle to fall on him. The resulting wounds caused a punctured lung and a fractured spine. He was not released from the hospital until March 15, 2014. Motivated by this neardeath accident, he incorporates certain safety features not previously disclosed in the International Application, as well as defining an embodiment of the invention that is a kit appropriate for the after-market. A nonprovisional patent application is filed April 3, 2014, which claims the benefit of the earlier filed International Application. The nonprovisional patent application contains 30 claims. Only claims 1 - 15 relate to the collapsible motorcycle hammock, Claims 16 - 24 incorporate the safety feature, and Claims 25 - 30 claim a kit. Realizing for the first time the expense associated with obtaining worldwide patent protection, Wilson decides to abandon the International Application, entering no countries in the national phase of the International process. In the First Office Action in the nonprovisional application, the patent examiner rejects claims 1 - 10 under 35 U.S.C. 103 based on a US patent to Roosevelt issued in 1921, in view of a published PCT, asserting priority to a French patent application filed by Dean on February 16, 2013, which published on August 15, 2014, and which formed the basis of a priority claim in the later-filed PCT application that designated the US and that published on September 28, 2014. Claims 11 - 15 are rejected under 35 U.S.C. 103 based on Roosevelt, in view of Dean and further in view of Gonzalez. Gonzalez is a Mexican patent issued on June 21, 2014, which was filed in Mexico on February 15, 2013, and which was used to support priority in a subsequent US filing that was published on August 28, 2014. Claims 16 - 24 are rejected under 35 U.S.C. 103 based on Roosevelt, in view of Dean and Gonzalez, and further in view of Costello, which is an Italian patent issued on November 22, 2015, on an application filed in Italy on November 1, 2013, which was used to support priority in a subsequent US filing that was published on April 15, 2015. Claims 25 - 30 are rejected under 35 U.S.C. 103 based on Roosevelt, in view of Cooke. Cooke is a UK published patent application, which published on May 1, 2014, which was filed on January 4, 2013, and which was used to support a subsequently published PCT application designating the US Which arguments can Wilson successfully make to overcome these obviousness rejections? A Wilson will not be able to remove any of these references. Therefore, he will need to either amend the claims or argue the obviousness rejections on their merits by pointing out that there is no teaching, suggestion or motivation to combine the references and the claims define an invention that provides unexpected results. B Wilson will be able to overcome the rejection of claims 1 - 24. Wilson unintentionally failed to file the nonprovisional patent application within 12 months of the filing of the International Application. Thus, he can restore his priority claim provided he filed within an additional 2-month grace period, which he did. Thus, he will be able to claim priority to the February 28, 2013 International filing, which will successfully remove Dean as a reference. C Wilson will be able to overcome the rejection of claims 11 - 15 because Gonzalez is a Mexican patent application filed on or after December 8, 1993. D Wilson will be able to overcome the rejection of claims 11 - 15 because Gonzalez was filed in Mexico prior to the effective date of first-to-file provisions in the United States. E Wilson will be able to overcome the rejection of claims 25 - 30 because the Cooke reference will only be prior art as of the date it is published on May 1, 2014.

(A) is correct. The first thing to notice is that Wilson will be allowed to claim priority to the International Application filed on February 28, 2013. He is well within the 30-month timeframe of the priority date associated with his PCT filing date. Unfortunately, not all of the invention is disclosed in the International application. Specifically, the nonprovisional patent application filed on April 3, 2014, contains claims directed to disclosure first appearing in that nonprovisional filing. Since this nonprovisional filing is made on or after March 16, 2013, the entire application will be treated under the AIA, which means that he will not be able to remove any of the obviousness references as prior art. That means he must amend the claims to overcome the rejections or argue the rejections substantively, which would require him to argue the lack of teaching, suggestion or motivation, as well as some kind of unexpected results, which will overcome the seven KSR rationales. (B) is incorrect because Dean will not be able to be removed as a reference. The AIA will apply due to the claiming of material first appearing in an application on or after March 16, 2013. Dean was filed in France on February 16, 2013, which is prior to Wilson's priority date of February 28, 2013. (C) and (D) are incorrect because Wilson will not be able to remove the Gonzalez reference. (C) is specifically wrong because it does not matter that Gonzalez was filed after the NAFTA date of December 8, 1993. (D) is specifically wrong because it does not matter when Gonzalez was filed; it only matters whether the application being examined is entitled to pre-AIA or AIA treatment. Because the AIA will apply due to Wilson claiming material first appearing in an application on or after March 16, 2013, the Gonzalez reference will be prior art as of its Mexican filing date of February 15, 2013, which pre-dates Wilson's priority date. (E) is similarly incorrect because the Cooke PCT will be prior art as of its UK filing date, which is January 4, 2013, which pre-dates Wilson's priority date.

Michael Arthur is the inventor of a cordless jump rope. Essentially, the invention is a jumprope simulator incorporating rotatable, counterbalancing weights within a handle and marketed to those individuals who really like to jump rope but who lack the physical prowess necessary to actually jump the rope. Arthur, who is very proud of his invention, writes up a description of the invention together with several drawings and shows them to various individuals at his health club, all of whom like the invention. The first such disclosure occurred on June 15, 2013. One fellow by the name of Bradley Brady likes it so much that he decides to file a patent application on the cordless jump rope, which occurs on November 15, 2013. Arthur subsequently files a patent application on December 15, 2013. The application filed by Brady fully discloses the Arthur invention. The application filed by Arthur includes 20 claims, 3 of which are in independent format. The claims filed by Arthur are quite different compared to the claims submitted by Brady. Will it be possible for Arthur to overcome a rejection by a patent examiner based on the Brady application? A Yes. Arthur can submit a Rule 1.130 affidavit. B Yes. Arthur can submit a Rule 1.131 affidavit. C Yes. Arthur can submit a Rule 1.132 affidavit. D Yes, but Arthur will be required to file a petition to institute a derivation proceeding under AIA 35 U.S. C. 135. E No, Arthur will not be able to remove the reference under the AIA first-to-file rules.

(A) is correct. Unless the other party (the suspected deriver) has submitted his or her own application, the issue for the applicant is disqualifying the prior art under AIA 35 U.S.C. 102(b) rather than showing derivation under AIA 35 U.S.C. 135. Here, Brady has filed a patent application based on Arthur's invention, but AIA 35 U.S.C. 135 applies to a claim to an invention that is the "same or substantially the same" as a claim of an earlier application. Thus, because Arthur's claims are quite different compared to the claims submitted by Brady, Arthur will not need to file a petition to institute a derivation proceeding and instead can utilize a 130 affidavit to avail himself of the exception pursuant to AIA 35 U.S.C. 102(b). Thus, (A) is correct and (D) is incorrect. (B) is incorrect because a 131 affidavit applies pre-AIA where the applicant is attempting to establish an earlier date of conception. (C) is incorrect because a 131 affidavit is one that you file to submit evidence to the Patent Office. It is not the correct procedural vehicle to remove a reference either under AIA or pre-AIA. (E) is incorrect because you can use a 130 affidavit in this scenario.

An acceptable Appeal Brief in the USPTO includes citations of case law authority to A Either the United States Reports or the West Reporter system. B the Federal Reporter only. C The United States Patent Quarterly only. D Commissioner's Decisions only. E Case copies provided with the Brief only

(A). The Board has a preference for United States Reports and West but, when not covered, any other reporter system is acceptable. (B)-(E) are all acceptable, but are wrong because of the use of "only." The Board rules no longer require citation to a particular reporter system and requires copies only when a case is not found in either US Reports or West.

Claim 1 in the application is directed to an apparatus. Claim 2 is directed to the same apparatus and further defines the apparatus. Claim 3 is directed to a method of using the apparatus set forth in claims 1 and 2. Claim 4 is an independent method claim directed to a method of using the apparatus. A rejection/objection of these claims would be: A Claim 3 is rejected under 35 USC 112(b), for being in improper multiple dependent form. B Claim 3 is objected to as being in improper multiple dependent form and is rejected under 35 USC 112(b), as being indefinite for mixing claim types. C Claim 4 is rejected under 35 USC 112(b), for mixing claims types in an application. D All of the claims are rejected under 112(a), as lacking antecedent in the specification. E B and C.

(B) is correct because claim 3 is an improper multiple dependent claim. Multiple dependent preambles must refer to the claims from which it depends in the alternative. Further, when writing a method claim, you do not mix claim types. Instead merely refer to "A method of using a meat slicer comprising..." (A) is incorrect because it is incomplete in comparison to answer (B). (C) is incorrect because claim 4 does not mix claim types. (D) is incorrect because there are no antecedent basis issues in these claims. Furthermore, a lack of antecedent basis would result in an objection and a rejection under 112(b), not under 112(a). (E) is incorrect because (C) is incorrect.

Bernard is an independent inventor who, like so many other independent inventors, does not have the funds necessary to seek advice from a patent attorney. He joined a local inventors group and has learned from some of the more experienced members that the new first-to-file law continues to include a grace period. In fact, he understands that the grace period allows an inventor such as himself to publish a disclosure of his invention and then subsequently file a patent patent application within 12 months. If you had the opportunity to speak with Bernard, which of the following would you NOT tell him? A The disclosure should be as complete as possible. B Any subsequent disclosure by another will be unable to be used as prior art as long a U.S. patent application is filed within 12 months. C An independently arrived at disclosure by another can be considered prior art and could prevent the issuance of a patent. D If the disclosure describes elements A, B and C, and a subsequent disclosure prior to filing discloses A, B, C and D, then at least element D would be considered prior art. E All of the above.

(B) is correct. Independent, subsequent disclosures may be prior art.

On September 23, 2016 a U.S. patent application was filed directed to a screwdriver. On January 8, 2017 a claim for priority was made and a certified copy filed of a Canadian application filed October 21, 2015. A continuation was filed in June of 2017 under 37 CFR 1.53 (b) and the parent abandoned shortly thereafter. No claim for priority was made in the continuation, which issued as a U.S. patent on January 13, 2018. Which of the following statements is true: A The failure to claim priority in the continuation can only be corrected by filing a reissue application. B The failure to claim priority in the continuation application can be corrected by obtaining a Certificate of Correction and also filing a Grantable Petition and fee for Late Assertion of Priority. C The failure to claim priority cannot be corrected and the priority is lost. D There is no need to claim priority in the Continuation since priority was claimed in the parent. E None of the above is true.

(B) is correct. MPEP 216.01.

Your new client Mickey Smith discloses to you a new invention which he has made relating to an automotive battery. In 2017 he had the idea that a better battery could be made using acid which was purified to an extraordinary degree, more than 99.99% pure. He built such a battery and it worked as he expected. At the time his battery was the subject of a newspaper article which did not describe what made it work so well. He was able to interest General Motors in buying the rights to his invention, but after much negotiation, no deal was struck. He continued to work over the years at improving his battery and this year made a remarkable improvement relating to an additive to the acid. He wants to obtain the best protection available under these facts. You should advise him that: A He should have filed a patent application many years ago and it is too late now. B He should file an application directed to both the original invention and the improvement. C A patent cannot be obtained on the original invention because it was made more than one year ago, but an application can be filed covering the improvement. D A patent cannot be obtained on the original invention because it was offered for sale more than a year ago. Patent protection may be available on the improvement if it is unobvious over the original invention. E He should file an application directed to the original invention. If it is found patentable, the application can then be amended to cover the improvement.

(B) is correct. On these facts, there is nothing to suggest that he is unable to file and obtain a patent. (A) is incorrect because Smith can file a patent application and obtain patent protection. (C) is incorrect because there is no per se prohibition against patenting an invention made more than one year ago. (D) is incorrect because the question does not mention any sale. The offer for the rights to an invention does not qualify as a sale event. (E) is incorrect because an application cannot be amended to add new matter not previously disclosed in the application.

Your client John Wells has at last received the Notice of Allowance covering his new Mexican jumping bean. Mr. Wells is a resident of Costa Rica, and tells you he is worried that someone will start some proceedings in the Patent Office or in some court affecting his patent and he will not be aware because postal delivery is so bad in his country. He wants his brother who lives in Los Angeles to take care of any and all business relating to the patent. You should: A Advise him not to worry because the examiner is a friend of yours and will call you if anything happens. B Prepare for Mr. Wells' signature a written designation of his brother as the one on whom service should be made concerning the patent. C Advise him that all papers from the Patent Office will be sent to you as attorney of record and you will forward them to him by a secure route. D Prepare and file a Notice of Foreign Patent Owner requesting that all papers be sent to Mr. Wells care of the American Embassy in Costa Rica. E Prepare and file an assignment of the patent to you together with an agreement that you will forward any and all monies received with respect to the patent to Mr. Wells.

(B) is correct. See 37 C.F.R. 1.33(a).

A Notice of Allowance was transmitted by the Patent Office to your office on November 24, 2017. Your staff docketed the issue fee for payment on or before May 24, 2018, and your client was advised that unless you received payment in full before that date you would not pay the fee. On May 23, 2018, you receive a call from your client saying that he has mailed a check, but the check does not arrive by May 24. The best course of action would be: A Pay the issue fee for your client. B Advise your client that, because of your error, the application is now abandoned, but can be revived because the abandonment was unintentional. C Pay the issue fee together with a three-month automatic extension of time. D Advise your client that you did not pay the issue fee because the funds were not received in time. E File a petition to withdraw from representation.

(B) is correct. The application has already gone abandoned because you have only three months to pay the issue fee. Thus, you had until February 24, 2018, within which to pay the issue fee. The application went abandoned as of February 25, 2018. (A) is incorrect because it is too late to pay the issue fee. (C) is incorrect because you cannot receive an extension of time to pay the issue fee. (D) is incorrect because it is not accurate. You gave the client until May 24, 2018. Therefore, your failure to pay the issue fee on or before February 24, 2018, had nothing to do with the client not providing payment by May 24, 2018. (E) is incorrect because the application has already gone abandoned. Thus, there is no need to withdraw.

Spencer Kelly is the inventor of an attachment to a shotgun barrel, which modifies the pattern of the shot released from the shotgun shell when it is fired. Kelly, who is very proud of his invention, writes up a description of the invention together with several drawings and shows them to various individuals at his gun club, all of whom like the invention. The first such disclosure occurred on August 22, 2013. One fellow by the name of Alex Lexington likes it so much that he decides to file a patent application on the shotgun barrel attachment, which occurs on October 22, 2013. Kelly subsequently files a patent application on January 1, 2014. The application filed by Lexington fully discloses the Kelly invention. The application filed by Kelly includes 25 claims, 5 of which are in independent format. Claims 1 - 5 filed by Kelly are identical to claims submitted by Lexington, Claims 6 - 10 filed by Kelly are substantially similar to the claims filed by Lexington, and Claims 11 - 25 are quite different when compared with any claims filed by Lexington. Will it be possible for Kelly to overcome a rejection by a patent examiner based on the Lexington application? A Yes. Kelly can submit a Rule 1.130 affidavit to overcome any rejection of claims 1 - 25. B Yes. Kelly can submit a Rule 1.130 affidavit to overcome any rejection of claims 11 - 25. C Yes. Kelly can submit a Rule 1.131 affidavit. D Yes. Kelly can submit a Rule 1.132 affidavit. E Yes, but Kelly will be required to file a petition to institute a derivation proceeding under AIA 35 U.S. C. 135.

(B) is the best answer. When a suspected deriver has submitted his or her own application, the critical question turns to whether the claims submitted are the "same or substantially the same" as a claim of an earlier application. If the claims submitted by Kelly are the same or substantially the same when compared to the claims filed by Lexington, Kelly will be required to pursue a derivation proceeding. If the claims are not the same or substantially similar to the claims filed by Lexington, then Kelly can take advantage of the AIA 35 U.S.C. 102(b) exception by utilizing a 130 affidavit. This question presents a mixture of the two. Claims 1 - 10 filed by Kelly are either identically or substantially similar to those filed by Lexington. Thus, Kelly cannot resort to a 130 affidavit with respect to those claims and must pursue a derivation proceeding. Claims 11 - 25 are, however, quite different than those claims submitted by Lexington; thus, a 130 affidavit can be used to remove the Lexington reference as prior art for those claims. Thus, (B) is correct and (A) is incorrect. (C) is incorrect because a 131 affidavit applies to pre-AIA where the applicant is attempting to establish an earlier date of conception. (D) is incorrect because a 132 affidavit is one that you file to submit evidence to the Patent Office. It is not the correct procedural vehicle to remove a reference either under AIA or pre-AIA. (E) is incorrect because it is overbroad. A derivation proceeding is not required if Kelly wishes to pursue claims 11 - 25 because those claims are not the same or substantially similar to claims previously filed by Lexington. So (E) isn't completely wrong, but (B) is a better answer.

Which of the following sections of 102 can be the basis for a provisional rejection? A 102(a)(1) B 102(b) C 102(a)(2) D 102(c) E (A) and (C) above

(C) is correct because a provisional rejection can only be made pursuant to 102(a)(2). It is provisional in nature because a 102(a)(2) date does not exist unless a patent application has published or a patent has issued. If there is a common owner or common inventor, then the patent examiner may issue a provisional 102(a)(2) rejection, which is a notice that once a 102(a)(2) date exists, it will be used to reject the claim(s) at issue. (A), (B), (D) and (E) are incorrect because, under these provisions, there is no ability or justification to issue a provisional or preliminary rejection.

1. Means for lifting a horse onto a trailer, comprising: a sling for attachment to a suspended winch, for placing beneath the horse and winching said horse upward, and bearing means adapted to enable the winch to swing while loaded with said horse onto said truck. A This claim will be rejected as an improper 35 USC 112(f) claim. B This claim will be rejected under 35 USC 112(a), for being inoperative. C This claim will be rejected under 35 USC 112(b), as being indefinite. D This claim will be rejected under 35 USC 102/103, as being an accumulation of parts. E This claim will not be rejected.

(C) is correct because the claim as provided does not explain how the winch is structurally in communication with the bearing means. Merely saying the winch is adapted to enable the winch to load the horse is not sufficient. General characterizations explaining intended function are insufficient. MPEP 2173 and MPEP 2173.02. (A) is incorrect because this claim is not a means plus function claim. Claims that use "means" language but then also recite structure (i.e., the winch) are not construed as implicating 112(f). (B) is incorrect because the claim would not be rejected as inoperative given that the invention would work if it were definite. (D) is incorrect because it is really nonsense. Every invention is in one way or another an accumulation of parts. Thankfully KSR has not been interpreted to render every collection of parts obvious. (E) is incorrect because the claim will be rejected as being indefinite for inferentially claiming the attachment of the winch to the bearing mechanism.

Which of the following is correct with respect to a derivation proceeding initiated at the USPTO? A In a derivation proceeding, the applicant must detail how the invention was arrived at (or derived) so that the examiner can determine sufficiency of disclosure for purposes of 35 USC 112. B No corroborating evidence is required where the affidavit is submitted by the inventor because the inventor is considered to be in the best place to offer accurate, reliable, firsthand information about what actually happened. C Parties to a derivation proceeding may resort to binding arbitration. However, the Board will not consider the arbitration award unless it is filed within 20 days of the date of the award. D A petition for derivation and supporting evidence must be served by a process server or other official designated by state law to serve court filings on the respondent at the correspondence address of record for the earlier application or subject patent. E The petition in a derivation proceeding must explain why the claimed invention is the same or substantially the same as the invention disclosed to the respondent, but need not offer a claim construction.

(C) is correct, at least insofar as it goes. Parties may resort to binding arbitration. Notwithstanding, the Board is not bound by, and may independently determine, any question of patentability. Furthermore, the Board will not consider the arbitration award unless it: (1) is binding on the parties; (2) is in writing; (3) states in a clear and definite manner each issue arbitrated and the disposition of each issue; and (4) is filed within 20 days of the date of the award. (A) is incorrect because this is not something that is required in the petition. The petitioner must show that at least one claim in the petitioner's patent application is: (1) the same or substantially the same as the respondent's claimed invention; and (2) the same or substantially the same as the invention disclosed to the respondent. Furthermore, the petition must: (1) provide sufficient information to identify the application or patent for which the petitioner seeks a derivation proceeding; (2) demonstrate that a claimed invention was derived from an inventor named in the petitioner's application, and that the inventor from whom the invention was derived did not authorize the filing of the earliest application claiming such invention; and (3) for each of the respondent's claims to the derived invention, (a) show why the claimed invention is the same or substantially the same as the invention disclosed to the respondent, and (b) identify how the claim is to be construed. (B) is incorrect because inventor testimony must be corroborated by independent evidence. (D) is incorrect because the parties may agree to service being made electronically. Otherwise, service may be by EXPRESS MAIL or by means at least as fast and reliable as EXPRESS MAIL. Personal service is not required. (E) is incorrect because the petitioner is specifically required to offer a claim construction in the petition.

Your client MegaGiant has discovered a patent to its competitor GigaCorp that it feels may be infringed by its new lawn mower. MegaGiant has learned as well that the lawn mower described in the patent was on sale in the United States by GigaCorp more than one year before the earliest effective filing date of the patent. MegaGiant is seeking your advice. Your advice is: A A Request for Reexamination can be filed based upon the prior sale. B All the claims of the patent are invalid under 35 USC 102(a)(2). C All the claims of the patent are invalid under 35 USC 102(a)(1). D If a Request for Reexamination is brought based upon a prior patent or publication, the PTO may also consider the prior sale. E MegaGiant may petition the PTO to reissue the patent.

(C) is correct.

Jocelyn Carter is the inventor of a unique and highly concealable spy camera, which incorporates facial recognition modules. Carter files a nonprovisional patent application on April 2, 2012. The nonprovisional patent application does not contain any additional disclosure, but is filed with 21 claims, 4 of which are independent claims. On March 14, 2013, a Continuation-in-Part (CIP) is filed, further disclosing a previously undisclosed computerimplemented filtering system for sorting the recognized faces. On March 17, 2013, a continuation of the April 2, 2012, nonprovisional patent application is filed. Which of the following accurately states which law these applications will be examined under? I. The April 2, 2012, nonprovisional patent application will be reviewed with the examiner applying pre-AIA. II. The March 14, 2013, CIP will be reviewed with the examiner applying pre-AIA. III. The March 17, 2013, Continuation will be reviewed with the examiner applying AIA. A I B II C I and II D I and III E I, II and III

(C) is correct. (I) is correct because the nonprovisional patent application filed on April 2, 2012, will be examined under pre-AIA because it was filed prior to March 16, 2013. (II) is correct because the CIP will be treated under pre-AIA. Although it contains a new embodiment not found in either the provisional or nonprovisional applications, it was filed right before the trigger date for AIA application. Applications filed on or after March 16, 2013, are the first that are potentially open for treatment under the AIA. (III) is incorrect because the continuation filed on March 17, 2013, will be treated under pre-AIA because as a continuation the disclosure is identical to the parent, which in this case was filed on April 13, 2012. Thus, even though it was filed on or after March 16, 2013, it is entitled to pre-AIA based on its priority date.

Sheldon Cooper is the inventor of an android capable of accepting the conscious being of a living human. He filed a non-provisional utility patent application on September 15, 2012, which was published on March 21, 2013. April 1, 2013, the patent examiner issued a First Office Action on the Merits, which rejected all claims. After a favorable interview, an Amendment and Response were filed and the examiner issued a Notice of Allowance on July 9, 2013. The issue fee is paid on August 31, 2013, and the patent issues November 5, 2013. When is the last day that a third-party preissuance submission could have been filed? A March 21, 2013 B April 1, 2013 C July 8, 2013 D August 31, 2013 E November 5, 2013

(C) is correct. A preissuance submission must be submitted BEFORE the critical date of July 9, 2013, NOT on that date. See 37 C.F.R. 1.290 (b), which says: "Any third-party submission under this section must be filed prior to the earlier of..." [emphasis supplied]

Which of the following can be extended by filing a request for an automatic extension of time and paying the required extension fee? A Issue fee. B Reply Brief. C Response to a Notice of Missing Parts. D Official Action in a Reexamination. E IDS.

(C) is correct. A response to a Notice of Missing Parts can be automatically extended. (A) is incorrect because no extension is available to pay the Issue Fee. You have only three months to pay the issue fee and failure to pay it timely would result in the application going abandoned. (B) and (D) are incorrect because extensions for a Reply Brief and Office Action response in Reexamination can be extended only for cause, which requires the applicant to request an extension in advance of being late and providing sufficient reason why an extension should be granted. (E) is incorrect because you cannot get an extension to file an Information Disclosure Statement (IDS). You can file an IDS for free without any statement up until the time the examiner beings work on the First Office Action. Between First Office Action and Final Rejection, you must either pay a fee or make a statement. After Final Rejection, you must both pay the fee and make a statement. The statement mentioned previously is one that explains that the prior art being submitted has been known for not more than 3 months by everyone substantively involved in the prosecution of the patent application (i.e., everyone who had a duty to disclose).

If the broad generic claim in an application includes an inoperative species of the invention___________________ A The claim is properly rejected under 35 USC 112(a). B The claim is properly rejected under 35 USC 112(b). C The claim cannot be rejected as long as other operable species of the invention are also within the scope of the claim. D The claim will not be rejected as long as the inoperable species does not correspond to the best mode or a preferred embodiment of the invention. E A and B.

(C) is correct. MPEP 2164.08(b). A claim will be rejected for lack of enablement only if too many embodiments captured by the claim are inoperative. How many is "too many"? Case law teaches that if 50% or more of the embodiments captured in a claim are inoperative then the claim can be rejected (or found invalid if already issued). Case law also teaches that if only 20% or fewer of the captured embodiments are inoperative, the claim is acceptable. Because there is no bright-line percentage, you are most likely to be tested on the concept, and not a fact-based question that asks if a certain percentage that does not work is unacceptable. Certainly, any number between 20% and 50% would present an essay question, not a multiple-choice question. (A) is incorrect because you do not know enough to reach this conclusion. If a small percentage of inoperative species are captured in the claim, the claim will not be rejected. (B) is incorrect because the claim would be rejected under 112(a) for lack of enablement, not under 112(b). (D) is incorrect because there is no special requirement that the inoperative species not correspond to the best mode. (E) is incorrect because both (A) and (B) are incorrect.

The invention is an assembled CD holder which includes a cover member hingedly connected to a backing member. Inside the cover and backing member, the CD holder includes a snap connected to the inside surface of the cover member which retains the outer rim edge of the CD which is to be placed inside. The prior art is a conventional CD holder which includes a cover member hingedly connected to a backing member. Inside the CD cover, in the space between the cover and the backing member, is a snap which retains the CD by gripping the inner rim of the through passage in the center of the CD. You are asked to write the claims for the invented CD cover for your friend the computer wiz, and you write the following claims. Which, if any, are not indefinite. A A CD holder comprising: a cover member, a backing member connected to said cover member, and a snap member inside said cover and backing member which retains an outer edge of a CD which is inserted into the CD cover. B A combination CD holder, comprising a cover member hingedly connected to a backing member, and a snap adapted to retain said CD by retaining an outer rim edge of said CD. C A CD holder comprising: snap means for holding an outer rim edge of a CD, said snap means attached inside a space, defined between a cover member which is hingedly connected to a backing member, to an inwardly directed surface of said cover member. D A CD adapted for placement into a holder that snaps to its outer rim. E A CD holder consisting of a cover member hingedly connected to a backing member and having therein a snap member attached thereto which retains an outer edge of a CD.

(C) is correct. MPEP 2173.02. (A) is incorrect because the location of the snap is not defined. Further, the snap is to be inside "said cover," but no cover has been introduced. There is a "cover member" member introduced, but the second time that you refer to the "cover member" it is referred to as "said cover member," which implies that "said cover" is something different. Further still, (A) is incorrect because the function of the snap does not assist in defining it structurally. (B) is incorrect because the snap is not connected to anything else. Further explaining what the snap is "adapted" to do does not define it structurally. (D) is incorrect because the invention is not a CD, but rather a CD holder. (E) is incorrect because the snap is not connected to anything else. Further explaining the function of the snap do does not define it structurally.

Your client Tami Jones has retained you to prosecute her application which was prepared and filed by another attorney on May 3, 2016. The other attorney responded to the first official action some months ago. She gives you copies of a rejection and references which were cited in her Australian application which was filed after the U.S. application. The Australian rejection bears a date stamp of March 4, 2018. From a brief review you conclude that these references are material to the patentability of the claims. The most appropriate action today, August 28, 2019, would be: A Call the other attorney and ask him why he did not cite these to the Patent Office. B File a petition asking that prosecution be suspended for an indefinite time until you can review these references and advise Ms. Jones. C Prepare and file an IDS accompanied by the required fee. D Prepare and file an IDS accompanied by a statement that the references were cited in a communication from a foreign patent office known to you less than three months. E Wait until you see if the examiner allows the application before doing anything.

(C) is correct. MPEP 609. (D) is wrong because it refers to communications known to you, not the applicant.

A Notice of Allowance is sent to you on March 14, 2013, which you send on to your client. Your client responds to you that he has reviewed the application carefully and has concluded that the broadest claim - claim 1 - contains an unnecessary limitation. He is afraid that his competitors will be able to avoid infringement unless the limitation is deleted. After studying the file you agree that the limitation can and should be deleted and that the claim is patentable even without the unnecessary limitation. The best course of action would be: A Telephone the examiner and ask her to delete the limitation and explain why that should be done. B File an amendment when you pay the issue fee amending the claim to delete the unnecessary limitation. C File a Request for Continued Examination under Rule 114 with an amendment submission deleting the limitation. D Pay the issue fee and seek a reissue after the patent issues. E Tell your client that it is now too late to correct the mistake.

(C) is correct. See 37 C.F.R. 114(a)(1) and 37 C.F.R. 1.313(a). Applications may be withdrawn from issue for further action either at the initiative of the Office or upon petition by the applicant. Generally speaking, to request that the Office withdraw an application from issue, the applicant must file a petition showing good and sufficient reasons why withdrawal of the application from issue is necessary, and pay the inevitable Patent Office fee. A petition is not required if a request for continued examination under § 1.114 is filed prior to payment of the issue fee. If the Office withdraws the application from issue, the Office will issue a new notice of allowance if the Office again allows the application. Thus, because the issue fee has not yet been paid, no petition is necessary and the claims can be fixed by filing an RCE.

Shawn Slippery was granted a design patent directed to a troll design on May 23, 2005. After many unsuccessful attempts, he managed to convince a young woman he met in a bar that his patent was valuable and she agreed to buy the patent for $20,000 in cash, which she had secreted on her person. She gave Shawn the money, and Shawn wrote the following on a napkin in the bar, which Mary stuffed in her purse. "I, Shawn Slippery hereby assign all rights in my troll design patent D 234,781 to Mary Wistful. Shawn Slippery June 3, 2005" Mary came to your office today and told you she was jogging yesterday with a friend who told her that Shawn had been trying to sell the same patent to others. While she has yet to use the patent in any way, she believes that someday it may be valuable. You should advise her that: A Every patent that successfully survives reexamination is strengthened by the experience and a reexamination should be filed. B The napkin cannot be recorded because it has to be on A4 paper and must be signed by Mary as well. C The napkin should be copied onto regular paper and that paper recorded. D She should keep her ears open and advise you immediately if Shawn sells the invention again. E She should immediately execute an assignment to you which you will prepare and which you will then record.

(C) is correct. See 37 C.F.R. 3.24(b).

A proper Final Rejection is issued on May 31, 2012 and a Notice of Appeal is mailed to the Patent Office with a proper Certificate of Mailing on August 3, 2012, and received on August 5, 2012. After further consideration you mail an amendment on October 14, 2012 including a proper Certificate of Mailing and a proper two month extension request and fee. The amendment is received on October 18, 2012. The examiner transmits an advisory action on December 1, 2012 indicating that the amendment has been entered, but it does not place the application in condition for allowance. What is the latest date that an appeal brief can be filed with automatic extensions of time to avoid abandonment of the application? A February 28, 2013. B February 3, 2013. C March 5, 2013. D October 3, 2012. E None of the above.

(C) is correct. See 37 C.F.R. 41.37(a). The Appeal Brief is due two months after the Office receives the Notice of Appeal, and the filing of the Appeal Brief is fully extendable. This means you have two months plus the 5 months of automatic extension available. The Notice of Appeal was received on August 5, 2012. Seven months from that date is March 5, 2013.

You represent a joint venture between GM and the University of Syracuse called JVGM. The examiner rejects your client's U.S. patent application, which was filed on March 15, 2018, based upon obviousness-type double patenting. The basis of the rejection is a recently issued U.S. patent, which was filed three years before your application, but opted out of publication, and is assigned to GM. The new joint venture application is jointly assigned to GM and the University of Syracuse under the terms of a joint development agreement. Which of the following is true? A Both rejections can be overcome by filing a CIP application, which combines the disclosures of both the patent and the application, and abandoning both. B The double patenting rejection can be overcome by the University of Syracuse assigning its rights to GM. C The double patenting rejection can be overcome by GM assigning its rights to JVGM and then filing a terminal disclaimer of the application. D The 103/102(a)(2) rejection can be overcome by establishing that the date of invention for the invention in the patent is prior to the date of invention for the claims in the application. E None of the above

(C) is correct. The double patenting rejection can only be overcome by filing a terminal disclaimer (or changing the claims). The 102(b)(2)(c) exception toes not apply inasmuch as this is a double patenting rejection, not a prior art rejection. This results because the law requires that the invention disclosed, along with all obvious variations of that invention, fall into the public domain as of the date of patent expiration. The terminal disclaimer will allow for JVGM to obtain a patent on an obvious variation of their own work (as GM), but also ensure that the rights to those obvious variations cease to exist as of the date the first patent term expires. Notice that (C) is the only choice that mentions filing a terminal disclaimer, but why does GM need to assign its rights to JVGM first? This is because JVGM will need to own 100% of the rights in order to make the necessary statements on the terminal disclaimer, which include a promise by JVGM that they will own both patents without separating the two. JVGM could license Syracuse and/or GM separately, but JVGM must own both patents after filing the terminal disclaimer. (A) is incorrect because this would not solve the fact that a patent has already issued. A terminal disclaimer is necessary to eliminate the offensive patent term that would otherwise extend the rights beyond the patent expiration date of the first patent. (B) is incorrect because it is not who owns the rights that is the problem. The problem here is that an obvious variation of the first patent would remain protected after the first patent has expired unless a terminal disclaimer is filed. (D) is incorrect because a date of invention is irrelevant under AIA 102. (E) is incorrect because (C) is correct.

Which of the following regarding reexamination is not true? A Every ex parte reexamination must have Reasons for Confirmation/Patentability. B An amended claim or a new claim may not enlarge the scope of the original claims. C Ex parte reexaminations of patents in litigation are automatically suspended until the litigation is concluded. D Multiple ex parte reexaminations of the same patent may be merged. E There is no issue fee for an ex parte reexamination.

(C) is correct. There is no provision to stop a reexamination. See MPEP 2210.

Which of the following statements is false? A A copy of an executed oath or declaration may be filed instead of the original. B Disclosure in a preliminary amendment filed with an application is part of the original disclosure, provided that it is referred to in the oath or declaration. C The requirement for an oath or declaration can be satisfied by a co-inventor acting on behalf of a missing or hostile inventor. D An assignment which is in a foreign language can be recorded if it is accompanied by a statement the assignment is correct. E No oath or declaration is required in a provisional application.

(D) is correct because it is false. Assignments cannot be recorded in a foreign language alone. In order to be recorded, an assignment must be at least accompanied by an English translation. (A) is incorrect because it is true. A copy of an oath or declaration may be filed, for example, when a continuation is filed. A continuation contains no new matter. (B) is incorrect because it is true. A preliminary amendment may be filed with a new application, which is frequently the case when a continuation or continuation-in-part is filed. If the preliminary amendment arrives at the time of filing and is mentioned in the oath or declaration, it is considered to be a part of the original filing. (C) is incorrect because it is true. A co-inventor can file a substitute statement in lieu of an oath or declaration in the case where the other co -inventor is missing or refuses to cooperate. (E) is incorrect because it is true. An oath or declaration is not required in a provisional patent application.

Which of the following statements regarding PCT is true? A Anyone can file a PCT application at the U.S. Patent Office. B A PCT application can be the basis of priority, but cannot claim priority from any other application. C The PCT application need not identify an "Applicant". D A foreign filing license is not required to file a PCT application in the U.S. Patent Office, but may be required before the applicant or the Patent Office can forward a copy to a foreign patent office. E The express mail procedure for obtaining a filing date cannot be used to file a PCT application.

(D) is correct. MPEP 1832.

Independently of each other, Bert and Ernie invented the same rubberized, floating toy in the United States. A U.S. Patent was granted to Bert on February 18, 2012 on an application filed on April 12, 2010, claiming the toy. On April 10, 2012, Ernie filed a patent application in the USPTO claiming the same toy. There is no common assignee. Under which of the following provisions of 35 U.S.C. 102 is the U.S. patent to Bert prior art with regard to the toy claimed by Ernie? A 35 U.S.C. 102(a) B 35 U.S.C. 102(b) C 35 U.S.C. 102(e) D A and C E A, B, and C

(D) is correct. Pre-AIA 102. Based on the dates, we are dealing with pre -AIA 102. The facts make out a prima facie case under 102(a) and 102 (e). The 102(e) rejection is based on Bert's filing date, which is two years prior to Ernie's filing date. The 102(a) rejection is supported because the same invention was already patented in the US before invention by Ernie. This is true because Ernie's date of invention is presumed to be his filing date and there are no facts in the question to support an earlier date of conception, let alone one that would overcome an (a) or an (e) rejection.

Following a final rejection, you filed a Notice of Appeal on June 3, 2016, and an Appeal Brief on Monday, August 4, 2016. On June 4, 2017, the examiner mailed you his Answer which did not raise any new ground of rejection nor any new argument. On August 4, you pay a Brief forwarding fee. Thereafter, on September 1, 2017, you filed a reply brief together with a Request for an Oral Hearing to take place on January 15, 2018, and a request for an automatic one-month extension and the associated fee. The status of the appeal is: A The reply brief will be considered, and the hearing will be on January 15, 2018. B The reply brief will be considered, but the date for the hearing will be set by the Board. C The reply brief will not be considered, and the date for the hearing will be set by the Board. D The reply brief will not be considered, and the oral hearing has been waived. E The appeal is dismissed because the reply brief was not timely filed.

(D) is correct. There are several keys to getting this question correct. First, you must know that you only have two months to file a Reply Brief, which can only be extended for cause. The Reply Brief was due on August 4, 2017, but it was filed on September 1, 2017, making it late. The request for automatic extension was ineffective since the Reply Brief can only be extended for cause. The second thing to notice is that you had not previously requested an Oral Hearing until you filed the Reply Brief. But since the Reply Brief filing was not timely, it will not be considered, and neither will the request for an Oral Hearing. That means that the appeal will continue and the Board will only consider the original Brief and the Examiner's Answer. No Oral Hearing will be held.

Able and Baker file a joint patent application claiming a new communications device. A U.S. patent application with claims to the device was granted on July 7, 2013. The application was never assigned. Able and Baker come to realize that Charlie, who was an active participant during the reduction to practice of the device, should have been included as a joint inventor because he contributed important matter as a completion of conception to several claims included in the application. Able, Baker and Charlie come to you and ask your advice concerning this matter. Which of the following, if any, would be your best advice? A Able should file a reissue application because he is the first named inventor. B Able, Baker and Charlie should file a request for supplemental examination. C Able and Baker should file a petition for correct inventorship pursuant to 37 C.F.R. 1.48. D Able, Baker and Charlie should file a request for a Certificate of Correction to correct inventorship with the appropriate statements and fee. E Inventorship cannot be changed after the patent has issued unless the error in identifying the inventors was made without any deceptive intent.

(D) is correct. Whenever through error a person is named in an issued patent as the inventor, or an inventor is not named in an issued patent, the Director, may, on application of all the parties and assignees, issue a certificate naming only the actual inventor or inventors. Thus, correction of inventorship should be accomplished by filing a request for Certificate of Correction if (1) the only change being made is to correct the inventorship; and (2) all parties are in agreement and inventorship is not contested. Such a request to correct inventorship of a patent must be accompanied by (1) the appropriate fee; (2) a statement from each person who is being added as an inventor and each person who is currently named as an inventor either agreeing to the change of inventorship or stating that he or she has no disagreement in regard to the requested change; and (3) a statement from all assignees agreeing to the change of inventorship in the patent. Here, there are no assignees, so all that is required is (1) and (2). (A) is incorrect because a reissue application is appropriate to correct an error in the patent that is deemed to make the patent wholly or partly inoperative or invalid. However, if the only change being made in the patent is correction of the inventorship, this can be accomplished by filing a request for a certificate of correction and does not require a reissue application. (B) is incorrect because supplemental examination is appropriate when the patent owner seeks to have the Office consider information believed to be relevant to the patent. Supplemental examination may be used, for example, to correct errors or omissions that may have occurred during the original examination of the patent application so as to inoculate the patent against charges of inequitable conduct. When filing a supplemental examination, items of information must be submitted for the Office to consider. Thus, it is not the appropriate procedural tool to use to correct inventorship. (C) is incorrect because Rule 48 pertains to correcting inventorship during the pendency of an application. Furthermore, a petition is no longer required under Rule 48 even to correct inventorship during a pending application. (E) is incorrect because the America Invents Act (AIA) removed "without deceptive intent" from U.S. patent laws. Therefore, it is now possible to fix many things regardless of whether there was deceptive intent.

Which of the following statements regarding continuations and CIPs are not true? A A CIP can be filed under 37 CFR 1.53(d) with a preliminary amendment adding the new matter to the original disclosure. B A freshly executed oath must be filed in every continuation within sixty days of filing. A new assignment is required only if a continuation is filed under Rule 53(b). D The examiner cannot make a first action in a continuation "Final" even if it is the same as the last action in the parent. E All of the above are untrue.

(E) is correct

Your client has invented a new animal shampoo in her basement lab. Her husband was looking for a gasoline additive for the chain saw and, in the dimly lit basement, mistakenly picked up some of her shampoo and used it in the chain saw fuel mix. Surprisingly, the chain saw had more power and, even more remarkably, produced no visible smoke at all. Upon returning to the lab and conducting tests, your client confirms that the shampoo acts as a catalyst and makes the fuel in the chain saw burn at a 100% conversion rate into pure steam and carbon dioxide. Your client and co-inventor husband hasten to your office, chain saw in hand, to seek immediate help in writing the claims. You are away on extended leave in the Caymans on a patent attorney diving retreat and are unable to help. A senior partner, who has not written claims in years, but who has been left behind to mind the firm, is available to help. He crafts the following claims. Which claims will the Patent Office not reject, if any. A An animal shampoo for use in a chain saw fuel whereby 100 % burning takes place and steam and carbon dioxide are the result. B A chain saw fuel comprising shampoo and gasoline mixed so that no pollution comes out of the saw upon activation of the motor. C A mixture of animal shampoo and gasoline for use as a fuel, wherein complete combustion of the fuel occurs in a chainsaw. D A catalyst for use as a fuel additive which results in 100% conversion of the fuel into steam and carbon dioxide. E The Patent Office will reject all of the above.

(E) is correct because all of the claims will be rejected by the Patent Office. In fact, each of the claims suffers from the same infirmity. MPEP 2111.02. Notice how in (A) though (D) the claims all incorporate the hoped-for result rather than providing a definite formula for the compound.

Your client comes into your office with an ashen face following receipt of the patent you obtained for her. The claims barely leave out an important disclosed but sadly unclaimed invention because of inartful narrowness in the claims. That very invention has been copied by her ex-husband's company and is being sold widely. You quickly explain the basis for filing a correction to the patent. You make five statements. Which, if any, are incorrect? A She can file either an ex parte reexamination or reissue at this early date to fix the claims. A reexam is better since the claims are already being infringed. B She can file a reissue to broaden the claims to include subject matter given up to avoid prior art, but she must act quickly to file within two years of the issue date. C She can file a reissue, but not an ex parte reexamination, and she must act within two years to file the reissue application. D A certificate of correction is not possible to correct the defect in this patent. E A and B.

(E) is correct because both (A) and (B) are incorrect statements. MPEP 1412.02(I)(C), MPEP 1412.03 and MPEP 2258(III). (A) is an incorrect statement because you cannot use reexamination to broaden claims. (B) is an incorrect statement because you cannot capture through reissue that which you specifically gave up through argumentation in the underlying application. (C) is a correct statement because you cannot use reexamination to broaden and any broadening would have to take place through a reissue application that is filed within two years after the underlying patent was granted. (D) is a correct statement because a Certificate of Correction is only available for mistakes of a clerical or administrative nature.

Mary Rose can prove she conceived and reduced her invention to practice respectively on February 27, 2017 and on August 20, 2017 in the United States. On March 14, 2018 she filed a provisional application completely describing her invention and on March 13, 2019 she filed a utility application in the United States claiming the benefit of her provisional filing. For each of the following facts, determine which paragraph, if any, of 35 USC applies. Unbeknownst to Mary, General Mega Corporation filed a patent application in Belgium on September 30, 2017, which issued on March 15, 2018. A 102(a)(1) B 102(a)(2) C 102(c) D 102(e) E none of the above

(E) is correct because, on these facts, there is no valid rejection that can be issued. The Belgian patent application would not be prior art until it is published or issued. Mary can get behind the March 15, 2018 date of the Belgian patent with the earlier dated provisional patent application filed March 14, 2018.

Hubert Davis is the inventor of a set of ear plugs that actually, and truly, blocks out even the most annoying noises. Davis filed a non-provisional patent application on October 1, 2011, and was ultimately awarded his first patent on April 10, 2013. Prior to the issuance of the patent, Davis filed a continuation pursuant to Rule 1.53, which was accomplished on March 17, 2013. A First Action in the continuation was mailed on July 27, 2013. Davis quickly responded and achieved a Notice of Allowance on September 15, 2013, with this second patent issuing on November 28, 2013. Mike Johnson would like to challenge these patents owned by Davis, but would prefer to avoid all out litigation in the district courts. Johnson walks into your office on December 19, 2013. What recommendation do you make? A Post-grant review can be requested against both the first and the second patent because both were issued after March 16, 2013. B Post-grant review can be requested against both the first patent and the second patent because both were issued after September 16, 2012. C Inter partes review cannot be requested against the first patent, because inter partes review is only applicable to patents issued on applications filed on or after March 16, 2013. D Inter partes reexamination can be requested. E None of the above.

(E) is correct. (A) and (B) are incorrect because post-grant review is not available for either patent. A petition for a post-grant review of a patent must be filed no later than the date that is nine months after the date of the grant of a patent or of the issuance of a reissue patent. Among other things, the petitioner must certify that the patent for which review is sought is available for post-grant review. The post-grant review provisions, however, only apply to patents issued from applications that have an effective filing date on or after March 16, 2013. Thus, in this situation, post-grant review is not an option. Davis's original filing date is October 1, 2011, which applies to both the first patent issued and to the patent issued as the result of the filing of a continuation. Since the application filed on March 17, 2013 is a continuation, the effective filing date will be October 1, 2011. (C) is incorrect because inter partes review is available to challenge any patent regardless of whether granted under first-to-file or first-to-invent rules. (D) is incorrect because inter partes reexamination is no longer available.

Oscar files a provisional patent application on April 1, 2012, in which he fully describes a single embodiment of his invention. Oscar then subsequently files a non-provisional patent application on March 16, 2013, which claims benefit to the earlier-filed provisional patent application. His non-provisional patent application contains more structural detail than does the provisional filing, but no additional embodiments. This new structural detail is deemed necessary to define over newly discovered prior art, and is contained in the claims. When is the latest that Oscar must notify the Patent Office that his non-provisional patent application contains matter not contained within his provisional patent application? A Never. There is no obligation to notify the Patent Office that a non-provisional patent application contains claims with more structural detail than disclosed in a provisional patent application. B March 16, 2013. C April 1, 2013. D July 16, 2013. E August 1, 2013

(E) is correct. MPEP 2159.04, 37 CFR 1.78(a)(6) and MPEP 210(III).

Wilbur's brother Frank also has a patent covering an improved screwdriver. The patentable feature is a variable power motor. The patent was issued on July 31, 2008, and filed on October 12, 2005. Frank asks your advice regarding the status of the patent. He tells you that he has not received any communication from the Patent Office since his patent issued and wonders if the patent is still in force. Since he was in prison during those years, and lived mostly under assumed names, likely any communication from the Patent Office would not have reached him anyway. Your best advice, today August 28, 2013 would be: A The patent is still in force. B The patent lapsed in 2012 because the maintenance fee was not paid timely and cannot be revived. C The patent lapsed in 2012 because the maintenance fee was not paid timely, but because Frank did not receive proper notice from the Patent Office, it can be revived. D The patent lapsed in 2012 because the maintenance fee was not paid timely, but can be revived by filing a petition stating that the failure to pay was unavoidable, and paying the required fees. E The patent lapsed in 2012 because the maintenance fee was not paid timely, but can be revived by filing a petition stating that the failure to pay was unintentional, and paying the required fees.

(E) is correct. See 37 C.F.R. 1.378 and 37 C.F.R. 1.137.

With respect to AIA 102, which of the following is true? A The availability of a disclosure as prior art may be measured from the effective filing date of the claimed invention, no matter where that filing occurred. B The AIA adopts a global view of prior art disclosures and thus does not require that a public use or sale activity be in this country to be a prior art activity. C The catch-all "otherwise available to the public" means, among other things, that secret prior sales are not considered prior art. D The patent or application shall be considered to have been effectively filed, with respect to any subject matter described in the patent or application, as of the earlier of the actual filing date of the patent or application for the patent containing the claim invention: or (2) the filing date of the earliest application for which the patent or application is is entitled, as to such invention, to a right of priority or the benefit of an earlier filing date. E All of the above.

(E) is correct. See response to Comment 7 in Examination Guideline, 78 FR 11059 at 11062 (February 14, 2013). Also see at 11073.

Mary Rose can prove she conceived and reduced her invention to practice respectively on February 27, 2017 and on August 20, 2017 in the United States. On March 14, 2018 she filed a provisional application completely describing her invention and on March 13, 2019 she filed a utility application in the United States claiming the benefit of her provisional filing. For each of the following facts, determine which paragraph, if any, of 35 USC 101 or 102 applies as prior art. Mary reduced the invention to practice in Canada. The invention was disclosed but not claimed in an application to Smith filed in Canada on February 26, 2017, and which issued as a patent on May 28, 2018. A 102(a)(1) B 102(b) C 102(a)(2) D 102(c) E none of the above

(E) is correct. The Canadian patent only becomes prior art after Mary has already filed her provisional patent application in the US.

Wilbur is now obsessed with his patent problems and spends a week searching at the Patent Office for other prior art. Sadly, he finds another patent to Smith which issued on August 4, 2018, and which discloses, but does not claim, everything in Wilbur's issued patent. Wilbur's patent was filed on June 24, 2015 and issued on May 3, 2017. He conceived the idea in 2014 in France where he also reduced it to practice. Smith filed a PCT application in France on January 5, 2013, in which he designated the U.S. The Smith IA published in French in July 2014. He filed the papers to enter the national stage in the U.S. on June 3, 2015. The inventor Smith at that time was travelling in China and the declaration was filed on July 14, 2015 in response to the Notice of Missing Parts. Wilbur is ready to submit a 1.131 affidavit as to his own inventive activity in France. Which of the sections of 102 invalidate Wilbur's patent? A 102(a)(2) B 102(a)(1) C 102(c) D 102(b) E (A) and (B) above

(E) is correct. The Smith PCT obtains a 102(a)(2) date as of the PCT filing date of January 5, 2013. The PCT also has a 102(a)(1) date of July 2014. Wilbur's filing date is after both of these dates, so he loses out!

In addition to the above facts of Question 9, John filed a patent application in Canada in April of 2018, which issued as a Canadian patent in December 2019. John now believes his previously filed nonprovisional patent application is defective beyond repair and wants to file a new nonprovisional patent application in the US that does not claim priority to the previously filed US nonprovisional application. Which of the following sections of 102 bar patentability as of today, March 1, 2020, based only on the prior Canadian filing? A 102(a)(1) B 102(a)(2) C 102(c) D 102(b) E none of the above

(E) is correct. The prior Canadian patent is his own disclosure within one year of his US filing date, and can be disqualified as prior art under 102 (b)(1).

Which of the following statements is not true? A The one-month limit to complete a response found incomplete can be extended. B An application can be accepted for filing and given a filing date even though the inventors are not identified. C Claims, which are cancelled by a Preliminary Amendment filed with the application, are not counted in determining the additional claim fee(s). D The substance of telephone discussions with an examiner must be made of record in the application file. E A double patenting rejection can be made only if the two applications have at least one inventor in common.

(E) is correct. You have no way of knowing the answer to this question from the material presented on Day 1 and 2. The purpose is to remind you not to be flustered when presented with a question out of left field. Rather you should circle the question to return to at the end, or invest a modest time in trying to look up the answers in the MPEP.

You are a registered patent practitioner who prosecuted an original U.S. patent application for a foreign client. A U.S. patent has now been granted. The U.S. application included a proper claim for priority of the first-filed foreign patent application. When you received the patent, you noticed that the front page of the patent did not reflect a claim for priority. Upon review of the patent file, you discovered that even though a priority claim had been made, a certified copy of the foreign application had not been filed in the application. Which of the following would be the most appropriate advice to give to your client? A A request for ex parte reexamination should be filed to correct this error. B A certified copy of the foreign patent application should be sent to the USPTO with a request to place the certified copy into the patent file. C A reissue application should be filed to correct this error. D A petition should be filed with the Office of Petitions requesting that the certified copy of the foreign patent application be placed in the USPTO file. E A certificate of correction should be sought to correct this error.

A reissue application can be filed to correct an error in a patent. There must be at least one error in the patent to provide grounds for reissue of the patent. If there is no error in the patent, the patent will not be reissued. The most common bases for filing a reissue application are: (1) the claims are too narrow or too broad; (2) the disclosure contains inaccuracies; (3) applicant failed to or incorrectly claimed foreign priority; and (4) applicant failed to make reference to or incorrectly made reference to prior copending applications. In this situation, a claim for foreign priority was made, but it was not made correctly. In order to properly claim foreign priority, a certified copy of the foreign application must be provided to the Office, unless it has been previously submitted. In the question, you are told that this application is an "original U.S. patent application," which means that it could not have previously been submitted to the Office. Therefore, it must be submitted in this application to make the proper claim for foreign priority. (A) is incorrect because ex parte reexamination is for a substantial new question of patentability relating to one or more claims; it is not used to fix an incorrectly made priority claim. (B) is incorrect because merely sending in a certified copy at this point will not act as a proper perfection of foreign priority, not to mention the Office will not simply add something like this to a patent file without following the proper procedures. (D) is incorrect because a petition to add the certified copy to the file will not result in foreign priority being properly claimed, and again the Office will not simply add something like this after the close of prosecution. (E) is incorrect because a certificate of correction, by itself, is made to fix clerical or administrative errors only. Those that picked (E) may have done so because a certificate of correction can be obtained in some scenarios to have a late, but properly made, claim of foreign priority noted in the patent grant prior to issuance. Also, in some instances, see MPEP 216.01, under "PERFECTING PRIORITY CLAIM VIA CERTIFICATE OF CORRECTION: 37 CFR 1.55(g) eliminates the need to file a reissue application in order to perfect a claim for foreign priority. Where the priority claim required under 37 CFR 1.55 was timely filed in the application but was not included on the patent because the requirement under 37 CFR 1.55 for a certified copy was not satisfied, the patent may be corrected to include the priority claim via a certificate of correction under 35 U.S.C. 255 and 37 CFR 1.323, accompanied by a grantable petition under 37 CFR 1.55(f) or, in the case of a design application, a grantable petition under 37 CFR 1.55(g).

With respect to obviousness, which of the following are true? 1.In cases involving new chemical compounds, it is necessary to identify some reason that would have led a chemist to modify a known compound in a particular manner to establish prima facie obviousness of a new claimed compound. 2.If a patent examiner makes a prima facie showing of obviousness, the applicant may rebut based on unexpected results by demonstrating that the claimed invention exhibits some superior property or advantage that a person of ordinary skill in the relevant art would have found surprising or unexpected. 3.In order to demonstrate that a claimed invention is obvious, a patent examiner can demonstrate how one of ordinary skill in the art would be able to retrace the steps of the applicant in order to achieve the claimed invention. A Statement 1 B Statement 2 C Statement 3 D Statements 1 and 2 E Statements 1, 2 and 3.

ANSWER E. Statement 1, see Ortho-McNeil Pharmaceutical, Inc. v. Mylan Labs, Inc. Statement 2, see Procter & Gamble Co. v. Teva Pharmaceuticals USA, Inc. Statement 3 states that an examiner can use the knowledge of one of ordinary skill in the art at the time the invention was made to re-visit and re-assess (i.e., retrace) each step taken by the inventor to determine whether it was obvious. Cf. Ortho-McNeil Pharmaceutical, Inc. v. Mylan Labs, Inc.

A claim in a pending patent application for an electric toothbrush is rejected under 35 USC 102 as being anticipated by a U.S. Patent, which was issued to Lancer, the sole name inventor, for a similar electric toothbrush. The Lancer patent was issued one year and one day before the filing date of the application in question. The claim in the pending application contains a limitation specifying the location of an on/off switch. In accordance with USPTO rules and procedures set forth in the MPEP, which of the following arguments, if true, would overcome the 102 rejection? A The Lancer patent discloses and claims an electric toothbrush, but does not mention whether its toothbrush includes a power supply. B Evidence is submitted to show the electric toothbrush claimed in the application is commercially successful. C The Lancer patent teaches away from the bristles of the claimed toothbrush. D Lancer is one of the three named inventors of the claimed toothbrush in the pending application. E The on/off switch in the Lancer patent is on a different side of the body than that recited in the claim for the electric toothbrush in the patent application.

ANSWER: (A) and (E) are accepted as correct answers. Regarding (E), see MPEP § 2131. To anticipate a claim, the elements of a reference "must be arranged as required by the claim...." See MPEP § 2131, citing In re Bond, 910 F.2d 831, 15 USPQ2d 1566 (Fed. Cir. 1990). In (E), the on/off switch of Lancer's toothbrush is arranged differently than that of the claimed toothbrush. (A) is accepted as correct because the given facts do not specify the location of the power supply as being included within the toothbrush. Though the description of the toothbrush as being electric can imply an inherent source of power, it may also may imply an external power source for the electric toothbrush. Accordingly, (A) is also accepted as a correct answer in the circumstances. (B) is incorrect because evidence of secondary considerations, such as commercial success, is irrelevant to a 35 U.S.C. § 102 rejection. See MPEP § 2131.04. (C) is incorrect. "'Arguments that the alleged anticipatory prior art...'teaches away from the invention'...[are] not 'germane' to a rejection under section 102.'" MPEP § 2131.05 (quoting Twin Disc, Inc. v. United States, 231 USPQ 417, 424 (Cl. Ct. 1986)). (D) is incorrect. "The term 'others' in 35 U.S.C. 102(a) refers to any entity which is different from the inventive entity. The entity need only differ by one person to be 'by others.' This holds true for all types of references eligible as prior art under 35 U.S.C. 102(a) including publications...." MPEP § 2132. Here, because Lancer is only one of three inventors of the claim, the patent is by others.

Independent claim 1, fully supported by the specification in a patent application states: Claim 1. An apparatus comprising: a plastic valve; a copper pipe connected to the plastic valve; and an aluminum pipe connected to the plastic valve. Which of the following claims, presented in the application, provide the basis for a proper rejection under 35 USC 112, second paragraph? Claim 2. The apparatus of claim 1, wherein said pipe is statically charged. Claim 3. The apparatus of claim 1, wherein the outer circumference of said copper pipe is statically charged. Claim 4. The apparatus of claim 1, further comprising a thermostat connected to said plastic valve. A Claim 2. B Claim 3. C Claim 4. D Claims 2 and 3. E Claims 3 and 4.

ANSWER: (A) is accepted as the most correct answer. MPEP § 2173.05 (e). Claim 2 is indefinite because it is not clear which "said pipe" the claim is referring to since claim 1 recites a copper pipe and an aluminum pipe. Accordingly, claim 2 provides the basis for a proper rejection under 35 U.S.C. § 112, second paragraph. In (B), claim 3 would be construed as definite, inasmuch as "the outer circumference" is an inherent part of the pipe and would not require antecedent recitation. Thus claim 3 does not provide a proper basis for the rejection. Therefore, (B) and (D) would be incorrect. Claim 4 is definite inasmuch as there is antecedent basis for "said plastic valve." Therefore, (C) and (E) are incorrect.

Roberto files a U.S. patent application fourteen months after he perfects an invention in Europe in 2009. Which of the following would establish a statutory bar against the granting of a U.S. patent to Roberto? A A foreign patent issued to Roberto 11 months prior to the filing date of Roberto's U.S. patent application. The foreign patent was granted on an application that was filed 23 months prior to the effective filing date of Roberto's U.S. patent application. The foreign patent application and the U.S. patent application claim the same invention. B The invention was described in a printed publication in the United States, 11 months prior to the filing date of the U.S. patent application. C The invention was in public use in the United States, less than one year prior to the filing date of the U.S. patent application. D The invention was on sale in a foreign (NAFTA member) country, more than one year prior to the filing date of the U.S. patent application. E None of the above.

ANSWER: (A) is the correct answer. 35 U.S.C. § 102 (d), and MPEP § 706.02(c). (A) is correct because the foreign patent establishes a bar under 35 U.S.C. §102 (d). MPEP § 706.02 (e). (B) is incorrect because the invention is not described in a printed publication more than one year prior to the date of the U.S. application. 35 U.S.C. § 102(b). (C) is incorrect because the invention is not in public use more than one year prior to the date of the U.S. application. MPEP § 2133. (D) is incorrect because the sale is not in the United States. 35 U.S.C. § 102(b); MPEP §§ 706.02(c) and 2133.03(d). (E) is incorrect because (A) is correct.

The sole claim in an application filed by A and having an effective filing date of June 5, 2016, recites an electrical signal amplifier comprising a plurality of germanium transistors connected together in a particular configuration. The claim is rejected under 35 USC 103 as being obviousness over a primary nonpatent reference publication (Reference P) in view of a secondary nonpatent reference publication (Reference S). Reference P has an effective date of April 3, 2016, and names A and B as the authors. Reference S has an effective date of December 10, 2015, and names C as the sole author. Reference P discloses an electrical signal amplifier including a plurality of silicon transistors connected together in the same configuration as that set forth in the claim. Reference S discloses a signal amplifier employing germanium transistors connected in a configuration different from the claimed configuration. The applicant does not deny that the references render the claimed subject matter prima facie obvious. Which, if any, of the declarations under 37 CFR 1.130 set forth below should be sufficient under the patent laws, rules and procedures as related in the MPEP to overcome the rejection? A An uncontradicted declaration by A asserting that the subject matter relied on by the examiner in reference P constitutes A's sole invention, with the result that Reference P is not available as prior art against the claim. B A declaration by A asserting that "the claimed amplifier has satisfied a long-felt need in the art." C A declaration by A and accompanying copies of competitors' advertisements which conclusively show that those competitors have exactly copied appellant's commercial embodiment of the claimed amplifier. D A declaration by A and supporting documentation establishing that ever since the filing date of A's application, sales of the commercial embodiment of A's claimed amplifier have consistently constituted ninety percent or more of the relevant signal amplifier market in the United States. E None of the above.

ANSWER: (A) is the correct answer. See MPEP § 717.01 (III), which states "Under certain circumstances an affidavit or declaration may be submitted which attempts to attribute an activity, a reference or part of a reference to the applicant. If successful, the activity or the reference is no longer applicable. . . . An uncontradicted 'unequivocal statement' from the applicant regarding the subject matter disclosed in an article, patent, or published application will be accepted as establishing inventorship. In re DeBaun, 687 F.2d 459, 463, 214 USPQ 933, 936 (CCPA 1982)." (B) is insufficient in the absence of "objective evidence that an art recognized problem existed in the art for a long period of time without solution." MPEP § 716.04 under heading "THE CLAIMED INVENTION MUST SATISFY A LONG-FELT NEED WHICH WAS RECOGNIZED, PERSISTENT, AND NOT SOLVED BY OTHERS." Regarding (C), see MPEP § 716.06: "[M]ore than the mere fact of copying is necessary to make that action significant because copying may be attributable to other factors such as a lack of concern for patent property or contempt for the patentees ability to enforce the patent. Cable Electric Products, Inc. v. Genmark, Inc., 770 F.2d 1015, 226 USPQ 881 (Fed. Cir. 1985)." (D) is insufficient in the absence of evidence demonstrating that the sales are attributable to the technical merits of the invention rather than to other factors, such as "heavy promotion or advertising, shift in advertising, consumption by purchasers normally tied to applicant or assignee, or other business events extraneous to the merits of the claimed invention, etc. In re Mageli, 470 F.2d 1380, 176 USPQ 305 (CCPA 1973) (conclusory statements or opinions that increased sales were due to the merits of the invention are entitled to little weight); In re Noznick, 478 F.2d 1260, 178 USPQ 43 (CCPA 1973)." MPEP § 716.03 (b) under heading "COMMERCIAL SUCCESS MUST BE DERIVED FROM THE CLAIMED INVENTION." (E) is incorrect because (A) is correct.

Evidence that a claim may not comply with 35 USC 112(b) occurs in accordance with the USPTO rules and the procedure set forth in the MPEP where: A Remarks filed by applicant in a reply or brief regarding the scope of the invention differ and do not correspond in scope with the claim. B There is a lack of agreement between the language in the claims and the language set forth in the specification. C The scope of the claimed subject matter is narrowed during pendency of the application by deleting the originally much broader claims, and presenting claims to only the preferred embodiment within the originally much broader claims. D Claims in a continuation application are directed to originally disclosed subject matter (in the parent and continuation applications) which applicants did not regard as part of their invention when the parent application was filed. E All of the above.

ANSWER: (A) is the most correct answer. In accordance with MPEP § 2172, under the heading "II. Evidence To The Contrary," states that evidence that shows a claim does not correspond in scope with that which applicant regards as applicant's invention may be found, for example, in contentions or admissions contained in briefs or remarks filed by applicant. In re Prater, 415 F.2d 1393, 162 USPQ 541 (CCPA 1969). (B) is incorrect. MPEP § 2172, under the heading "II. Evidence To The Contrary," states, "As noted in In re Ehrreich, 590 F.2d 902, 200 USPQ 504 (CCPA 1979) agreement, or lack thereof, between the claims and the specification is properly considered only with respect to 35 U.S.C. 112, first paragraph; it is irrelevant to compliance with the second paragraph of that section." (C) is incorrect. MPEP § 2172, under the heading "III. Shift In Claims Permitted," indicates that the second paragraph of 35 U.S.C. § 112 does not prohibit applicants from changing what they regard as their invention during the pendency of the application. In re Saunders, 444 F.2d 599, 170 USPQ 213 (CCPA 1971) (Applicant was permitted to claim and submit comparative evidence with respect to claimed subject matter which originally was only the preferred embodiment within much broader claims (directed to a method). (D) is incorrect. MPEP § 2172, under the heading "III. Shift In Claims Permitted," indicates that the fact that claims in a continuation application were directed to originally disclosed subject matter which applicants had not regarded as part of their invention when the parent application was filed was held not to prevent the continuation application from receiving benefits of the filing date of the parent application under 35 U.S.C. § 120. In re Brower, 433 F.2d 813, 167 USPQ 684 (CCPA 1970). (E) is incorrect because (B), (C), and (D) are incorrect.

Inventor files an application containing the following original Claim 1: 1. A widget comprising element A, and element B. In a first Office action on the merits, a primary examiner rejects claim 1 under 35 USC 103 as being obvious over reference X. Reference X explicitly discloses a widget having element A, but it does not disclose element B. The examiner, however, takes official notice of the fact that element B is commonly associated with element A in the art and on that basis concludes that it would have been obvious to provide element B in the reference X widget. In reply to the Office action, the registered practitioner representing the applicant makes no amendments, but instead requests reconsideration of the rejection by demanding that examiner show proof that element B is commonly associated with element A in the art. Which of the following actions, if taken by the examiner in the next Office action would be in accord with the USPTO rules and the procedures set forth in the MPEP? I. Vacate the rejection and allow the claim. II. Cite a reference that teaches element B commonly associated with element A in the art and make the rejection final. III. Deny entry of applicant's request for reconsideration on the ground that it is not responsive to the rejection and allow applicant time to submit a responsive amendment. A I and II only. B II only. C II and III only. D I, II, and III. E I and III only

ANSWER: (A) is the most correct answer. MPEP § 2144.03 provides that when an applicant seasonably traverses an officially noticed fact, the examiner may cite a reference teaching the noticed fact and make the next action final. Here, applicant did seasonably traverse the noticed fact by demanding proof in response to the rejection. II is therefore an appropriate action by the examiner. I is also an appropriate action because the examiner should vacate a rejection based on official notice if no support for the noticed fact can be found in response to a challenge by the applicant. See In re Ahlert, 424 F.2d 1088, 1091 (C.C.P.A. 1970) ("[a]ssertions of technical facts in areas of esoteric technology must always be supported by citation to some reference work" and "[a] llegations concerning specific "knowledge" of the prior art, which might be peculiar to a particular art should also be supported"). (B) is incorrect because (A) is correct. (C), (D), and (E) are incorrect because action III is improper. An applicant is entitled to respond to a rejection by requesting reconsideration, with or without amending the application. 37 CFR § 1.111(a)(1). Applicant is also required to timely challenge a noticed fact in order to preserve the issue for appeal. MPEP § 2144.03

A registered practitioner filed in the USPTO a client's utility patent application on December 30, 2016. The application was filed with a request for nonpublication, certifying that the invention disclosed in the U.S. application has not and will not be the subject of an application in another country, or under a multilateral international agreement, that requires eighteen month publication. Subsequently, the client files an application in Japan on the invention and some recent improvements to the invention. The improvements are not disclosed or supported in the utility application. Japan is a country that requires eighteen month publication. Two months after filing the application in Japan, and before filing any other papers in the USPTO, the client remembers that a nonpublication request was filed and informs the practitioner about the application that was filed in Japan. Which of the following courses of action is in accordance with the patent laws, rules and procedures as related in the MPEP? A The application is abandoned because the practitioner did not rescind the nonpublication request and provide notice of foreign filing within 45 days of having filed the application in Japan. The applicant must now file a petition and fee to revive under 37 CFR 1.137(b). B The application is abandoned because the applicant did not rescind the nonpublication request before filing the application in Japan. The applicant must now file a petition and fee to revive under 37 CFR 1.137(b). C The applicant should file an amendment to the specification of the U.S. application, adding the recent improvements to the disclosure in the specification. D The application is abandoned because the applicant did not rescind the nonpublication request by notifying the Office under 37 CFR 1.213(c) within the appropriate time. The applicant must now file a petition and fee to revive under 37 CFR 1.137(b). E The applicant could today notify the USPTO of the foreign filing. It is not necessary to file a petition and fee to revive for the application to continue to be examined in the USPTO.

ANSWER: (A) is the most correct answer. See 35 U.S.C. 122(b)(2)(B) (iii); 37 CFR § 1.213; MPEP § 901.03 for information on nonpublication requests. See 37 CFR § 1.137(f); MPEP § 711.03(c), under the heading "3. Abandonment for Failure to Notify the Office of a Foreign Filing After Submission of a Non-Publication Request." (B) is incorrect. The notice of foreign filing can be filed as late as 45 days after the foreign filing before the U.S. application becomes abandoned. (C) is incorrect. See MPEP § 608.04(a). The improvements would constitute new matter and new matter cannot be added to the disclosure of an application after the filing date of the application. (D) is not correct. The applicant is required to provide notice of foreign filing, not merely rescind the nonpublication request within the appropriate time. (E) is not correct. The applicant was required to provide notice of foreign filing within 45 days of filing in Japan, and two months have passed. As a result, a petition to revive under 37 CFR § 1.137(b) is required for examination to continue. Also see 37 CFR § 1.137(f).

With the exception that under 37 CFR 1.53 an application for patent may be assigned a filing date without payment of the basic filing fee, USPTO fees and charges payable to the USPTO requesting any action by the Office for which a fee or charge is payable, are required to be paid, in accordance with the MPEP and USPTO rules and procedure: A in advance, that is, at the time of requesting any action. B upon written notice from the USPTO. C within 20 days of requesting any action. D by the end of the fiscal year. E there are no fees.

ANSWER: (A) is the most correct answer. See 37 CFR § 1.22(a); MPEP § 509. Answers (B) through (E) have no factual basis or foundation in the MPEP.

Regarding a power of attorney or authorization of agent in a patent application, which of the following is in accordance with the USPTO rules and the procedure set forth in the MPEP? A All notices and official letters for the patent owner or owners in a reexamination proceeding will be directed to the attorney or agent of record in the patent file at the address listed on the register of patent attorneys and agents. B Powers of attorney to firms submitted in applications filed in the year 2016 are recognized by the USPTO. C The associate attorney may appoint another attorney. D The filing and recording of an assignment will operate as a revocation of a power or authorization previously given. E Revocation of the power of the principal attorney or agent does not revoke powers granted by him or her to other attorneys or agents.

ANSWER: (A) is the most correct answer. See 37 CFR § 1.33(c). (B) is incorrect. See MPEP § 403. Powers of attorney to firms filed in executed applications filed after July 2, 1971, are not recognized by the Patent and Trademark Office. However, the firm's address will be considered to be the correspondence address. (C) is incorrect. See MPEP §§ 402.02 and 406. The associate attorney may not appoint another attorney. (D) is incorrect. 37 CFR § 1.36. An assignment will not itself operate as a revocation of a power or authorization previously given. (E) is incorrect. See MPEP § 402.05. Revocation of the power of the principal attorney or agent revokes powers granted by him or her to other attorneys or agents.

In accordance with the USPTO rules and the procedures set forth in the MPEP, which of the following papers is precluded from receiving the benefit of a certificate of mailing or transmission under 37 CFR 1.8? A An amendment, replying to an Office action setting a period for reply, transmitted by mail with a certificate of mailing to the USPTO from a foreign country. B An amendment, replying to an Office action setting a period for reply, transmitted by facsimile with a certificate of transmission to the USPTO from a foreign country. C An information disclosure statement (IDS) under 37 CFR 1.97 and 1.98 transmitted after the first Office action. D A request for continued examination (RCE) under 37 CFR 1.114. E An appeal brief

ANSWER: (A) is the most correct answer. See MPEP § 512, which states "The Certificate of Mailing procedure does not apply to papers mailed in a foreign country." (B) is not correct. See MPEP § 512. Certificate of transmission procedure applies to correspondence transmitted to the Office from a foreign country and an amendment is not prohibited from being transmitted by facsimile and is not precluded from receiving the benefits under 37 CFR § 1.8. (C) is not correct. See MPEP § 609, under the heading "Time for Filing." An IDS will be considered to have been filed on the date of mailing if accompanied by a properly executed certificate of mailing or facsimile transmission under 37 CFR § 1.8. (D) is not correct. See MPEP § 706.07(h) Comparison Chart. An RCE is entitled to the benefit of a certificate of mailing or transmission under 37 CFR § 1.8. (E) is not correct. See MPEP § 1206. An appeal brief is entitled to the benefit of a certificate of mailing or transmission under 37 CFR § 1.8 because it is required to be filed in the Office within a set time period which is 2 months from the date of appeal.

Which of the following practices or procedures may be properly employed in accordance with the USPTO rules and the procedures set forth in the MPEP to overcome a rejection properly based on 35 USC 102(a)? A Perfecting a claim to priority under 35 USC 119(a)-(d) based on a foreign application having a foreign priority filing date that antedates the reference. B Filing a declaration under 37 CFR 1.131 showing that the cited prior art antedates the invention. C Filing a declaration under 37 CFR 1.132 showing that the reference invention is by "others." D Perfecting priority under 35 USC 119(e) or 120 by, in part, amending the declaration of the application to contain a specific reference to a prior application having a filing date prior to the reference. E (A), (B) (C), and (D).

ANSWER: (A) is the most correct answer. See MPEP § 706.02(b), under the heading "Overcoming a 35 U.S.C. § 102 Rejection Based on a Printed Publication or Patent." (B), and (C) are incorrect because they present showings that support the rejection. See MPEP § 706.02(b), supra. (D) are not correct because to perfect priority under 35 U.S.C. §§ 119(e) or 120 it is, inter alia, necessary to amend the specification of the application to contain a specific reference to a prior application having a filing date prior to the reference. See MPEP § 706.02(b), supra. Furthermore, the declaration is not to be amended. (E) is incorrect because (B), (C) and (D) are incorrect.

A utility application filed in May 2016 has been prosecuted through a second action final rejection. In the final rejection some claims were allowed and other claims were finally rejected. Which of the following accords with the patent laws, rules and the procedures as related in the MPEP for a proper reply to a second action final rejection in the utility application? A An amendment canceling all rejected claims and complying with 37 CFR 1.116. B Only a Notice of Appeal. C The appropriate fee for a request for continued examination (RCE). D A continued prosecution application (CPA) under 37 CFR 1.53(d). E All of the above

ANSWER: (A) is the most correct answer. The filing of an amendment complying with 37 CFR § 1.116 is a proper reply under 37 CFR § 1.113 to a final rejection. See MPEP § 714.13, under the heading "Entry Not A Matter of Right," which states, in pertinent part , "A reply under 37 CFR 1.113 is limited to: (A) an amendment complying with 37 CFR 1.116." (B) is not the most correct answer because the Notice of Appeal must be accompanied by the appeal fee required by 37 CFR § 1.17(b). (C) is not the most correct answer because the RCE must be accompanied by a submission (i.e., an amendment that meets the reply requirement of 37 CFR § 1.111). (D) is not the correct answer because CPA practice does not apply to utility or plant applications if the prior application has a filing date on or after May 29, 2000. See MPEP § 706.07(h), paragraphs I and IV. (E) is not the correct answer since (A) is a proper reply.

Under which circumstances must a patent practitioner decline representation or terminate representation if it has already commenced? A The representation will result in violation of USPTO Rules of Professional conduct. B The client has used the practitioner's services to perpetrate a crime or fraud. C A client insists upon taking action that the practitioner considers repugnant or with which the practitioner has a fundamental disagreement. D The representation will result in an unreasonable financial burden on the practitioner or has been rendered unreasonably difficult by the client. E All of the above.

ANSWER: (A). 37 C.F.R. 11.116 relates to declining or terminating representation. Subsection (a) deals with declining representation and Subsection (b) deals with situations where a practitioner may withdraw from representation. Practitioners shall not represent a client and must decline representation, or withdrawal if representation has commenced: (1) if representation would result in a violation of the USPTO Rules of Professional Conduct or other law; (2) the practitioner has become physically or mentally impaired to the point the practitioner is unable to represent the client's interests; or (3) the practitioner has been discharged by the client. Under Subsection (b), which deals with permissive rather than mandatory withdrawals, a practitioner may withdraw from representing a client for a larger variety of reasons. (B), (C) and (D) are all permissive reasons why a practitioner may withdraw, but the questions asks "must" withdraw. (E) is incorrect because (A) is correct. As an aside, it is worth noting that under Subsection (c), a practitioner must comply with applicable laws and tribunal rulings and continue representation if ordered to do so even if there is good cause for terminating the representation.

Lisa Allen is the inventor of a container for dispensing a liquid beverage concentrate and associated methods. The liquid beverage concentrate is held in a compartment, and a liquid beverage is held in a second beverage compartment. The first and second beverage compartments can be opened to link and thereby combine the concentrate with the liquid beverage, which is accomplished by moving the cartridge to unblock a flow path. Desiring patent protection for her invention, on June 3, 2019, Allen contacted the Law Offices of Andersen, Anderson & Armstrong. An appointment was set, and she met with patent attorney Scott Berry on Friday, June 14, 2019. Allen and Berry discussed the merits of doing a patent search and filing a provisional patent application. Allen, confident that her invention was unique and patentable, and desirous of a patent as quickly as possible, elected to immediately pursue a nonprovisional patent application. Allen further instructed Berry to file the application with papers and the proper fee to accelerate the application under the so-called Track One acceleration program. After she had indicated a desire to move forward with representation, and given Allen's desire to expedite matters, Berry had Allen sign a representation agreement while she was present in his Office. Berry also had Allen sign a §1.63 Declaration. The representation agreement quoted Allen $10,000 for the preparation and filing of the application, not including USPTO filing fees, which the agreement explained would be Allen's responsibility upon filing, which could be paid by credit card directly to the USPTO at the time of filing. The representation agreement further provided a breakdown in when fees would be considered earned, with 75% of the fee becoming earned upon delivery to the client of a complete draft application and 25% due upon filing of the application. Allen provided Berry with a check for $10,000, which Berry instructed his legal secretary to deposit into the firm's trust account, which occurred on Wednesday, June 19, 2019. On July 28, 2019, Berry delivered to Allen a complete draft of the nonprovisional application. In his communication to her, Berry explained that this was the first draft of the application, invited Allen to review and comment on the draft, and reminded her that at the time of filing she would be responsible for the filing fees. Allen became outraged because she had thought the filing fees were included in the $10,000 fee, despite the agreement clearly stating otherwise. Allen demanded a complete refund. Berry refused. Allen fired Berry. Berry withdrew $7,500 from client trust and informed Allen that amount had been considered earned and he returned the $2,500 unearned portion to her. Allen filed a complaint with the Office of Enrollment and Discipline at the USPTO. Based on these facts, what actions, if any, violate the Professional Rules of Conduct? A The §1.63 Declaration B Refusing the client a full refund C Providing a quote in a representation agreement that did not include all costs D Allowing the client to believe the fee included USPTO filing fees E None of the above

ANSWER: (A). Among many other things, patent prosecution rules mandate that the required §1.63 oath or declaration state that the person making the oath or declaration has reviewed and understands the contents of the application, including the claims. The practice of having a client sign a §1.63 oath or declaration prior to the preparation or completion of the application is plainly contrary to patent prosecution rules and to the Rules of Professional Conduct applicable to registered patent practitioners. See In the Matter of Raymond Y. Chan, Proceeding No. D2011-21. The outcome for Mr. Chan, who routinely engaged in this practice, was a public reprimand and a thirty-six (36) month probationary period. (B) is incorrect because the client is not entitled to a full refund. The representation agreement clearly explained that the cost of a first draft would be 75% of the fee, which is what Berry charged. (C) is incorrect. Although it is probably wise to quote or at least mention the likely costs that the client will incur through filing, there is no rule that requires this be done. (D) is incorrect. It is usually true that, when confusion arises, the presumption will normally be that it is the fault of the practitioner for allowing such confusion, but here the question affirmatively states the representation agreement is clear and not ambiguous. If the client is confused even when the practitioner has done everything possible to alleviate such confusion, the mistake or confusion by the client will not work to re-write a clear contractual arrangement. (E) is incorrect because (A) is correct.

Which of the following is in accordance with the practice and procedures of Chapter 600 of the MPEP? A Minor handwritten alterations to the claims in a patent application, added by the inventor post oath execution, may be accepted if the signed oath still applies in accord with CFR 1.63. B The Office will consider evidence of whether noninitialed and/or nondated alterations were made before or after the signing of the oath or declaration rather than require a new oath or declaration. C Any alteration to a patent application made by the applicant may be made after the application was signed and sworn to. D Non-initialed or non-dated handwritten alterations to the claims on an application filed in the USPTO are considered to be a minor informality. Thus, the Office personnel should not object to the same. E It is proper for an applicant to sign an oath or declaration even when the oath or declaration (i) does not identify a patent application or (ii) is not attached to or physically located together with the patent application.

ANSWER: (A). MPEP 608.01 provides that alterations are acceptable as long as the oath still applies. (B) is incorrect. The Office will not consider whether noninitialed and/or nondated alterations were made before or after the signing of the oath or declaration; it does not matter as long as the oath still applies. (C) is incorrect because not all alterations are allowed. (D) is incorrect because the Office may seek to determine whether the oath applies. (E) is incorrect because it is improper for an applicant to sign an oath or declaration which is not attached to or does not identify the patent application.

Identity Secure Patent Holding, LLC is the owner of a portfolio of U.S. patents relating to various computer-implemented methods of protecting and securing the identities of consumer data while in transit through the Internet. Many large e-commerce websites have a license to the patent portfolio and specifically and intentionally incorporate the Identity Secure technology into their security protocols, touting to shoppers that they are using state-of-the-art, industry-leading technology to protect information from hackers. As you might expect, there are a number of large e-Commerce websites that use the Identity Secure technology, but do not have a license and have indicated in no uncertain terms that they will not enter into license discussions. They have also threatened to file inter partes review challenges against several of the core patents in the portfolio. Given these threats, and the need to develop a plan to enforce the portfolio of patents, Identity Secure approached attorneys at B&H Plates. Over the next two months, B&H Plates attorneys and Identity Secure personnel engaged in face-to-face meetings and other privileged communications. Identity Secure provided confidential information, including information relating to validity and possible invalidity of claims, and work product analysis of the patents in the portfolio to attorneys for B&H Plates, and B&H Plates provided the advice sought. B&H Plates billed Identity Secure for the time its attorneys spent on this matter, and Identity Secure paid those bills. Subsequently, Identity Secure retained Stark, Logan & Strange to represent them to enforce the patent portfolio. One of the targets of enforcement was Abundant Life Technologies, who hired B&H Plates to prepare and file an inter partes review challenging the validity of all claims to U.S. Patent No. 11,999,992. Under what circumstances can B&H Plates ethically represent Abundant Life Technologies in an IPR challenging the claims of Identity Secure's '992 patent? A There are no circumstances under which B&H Plates can represent Abundant Life. B B&H Plates cannot represent Abundant Life unless Identity Secure agrees to the representation and waives any actual or potential conflict. C B&H Plates cannot represent Abundant Life unless Identity Secure agrees to the representation in writing. D B&H Plates cannot represent Abundant Life unless Identity Secure gives informed consent, confirmed in writing, and the disqualified practitioner is timely screened from participation in the matter. E The disqualified practitioners who originally provided the advice to Identity Secure are prohibited from ethically representing Abundant Life in the IPR relating to the '992 patent.

ANSWER: (A). This fact pattern creates a real mess. It is based on a real-life situation involving a large law firm, where ultimately, despite creating a screen they believed was ethical under the rules, the law firm still, nevertheless, decided to withdraw. Certainly, the attorneys involved in giving Identity Secure advice relating to enforcement strategy and reviewing confidential information relating to the patents and portfolio cannot represent Abundant Life in the IPR relating to the '992 patent, which is answer (E). However, there is much more to this question, which thus makes answer (E) incorrect because it is incomplete. To arrive at the correct answer, one must very carefully parse 37 C.F.R. 11.109 and 37 C.F.R. 11.110. Rule 11.110 relates to imputed conflicts and allows a firm to continue representing a client even if one or more practitioners would themselves be disqualified for ethical reasons, provided an ethical screen is put in place. If you read Rule 11.110 closely, you will notice that the aspect relevant here pertaining to former clients is (a)(2), which allows an ethical screen to be put in place where the disqualification of the practitioner is required by Rule 11.109(a) or (b). If you then turn to Rule 11.109, you see there are subparts (a), (b) and (c). Subparts (a) and (b) relate to disqualification where representation may be materially adverse to the former client, Rule 11.109(c) requires disqualification where the practitioner possesses some information about the matter. In this situation, the attorneys who provided the enforcement strategy to Identity Secure were given confidential information relating to the validity of the patents in the portfolio. Therefore, the conflict arises under 11.109(c) and cannot be solved with an ethical screen or client waiver. Therefore, (B), (C) and (D) are all incorrect.

Daniel Dryden and James Hardy are inventors of novel cell compositions and methods for their preparation and use. Dryden and Hardy discovered that some fraction of hepatocytes (i.e., liver cells) are capable of surviving multiple freeze-thaw cycles. Armed with this discovery, the inventors developed an improved process of preserving hepatocytes. Dryden and Hardy filed a patent application on the invention, which included claims covering a new and useful method of preserving hepatocytes. The claims recite pooling hepatocytes from multiple donors, producing a preparation of hepatocytes capable of being frozen and thawed at least two times, performing density gradient fractionation to separate viable and non-viable hepatocytes, recovering the viable hepatocytes, and cryopreserving the recovered viablehepatocytes. The examiner rejected the claims as being patent invalid under 35 U.S.C. 101. Under step one of the Alice/Mayo framework, the patent examiner determined that the claims are directed to an ineligible law of nature: the discovery that hepatocytes are capable of surviving multiple freeze-thaw cycles. Under step two of the Alice/Mayo framework, the patent examiner explained that the claims lack the requisite inventive concept, observing that, upon discovering the cells' capability of surviving multiple freeze-thaw cycles, the inventors simply reapplied a well-understood freezing process. Thus, there was not "significantly more" present to transform the claims directed to a judicial exception into claims that were patent eligible. If properly argued, can this rejection be successfully overcome? A No. The claims would not qualify as being patent eligible because they do not fall within one of the four statutory categories of invention. B Yes. The claims are not directed to a law of nature and, therefore, would be patent eligible. C No. The claims cover a law of nature that has been found by the courts to be patent ineligible. D Yes. The claims are directed to one of the four statutory categories of invention. Although the claims would properly be considered to be directed to a law of nature, the claims as a whole amount to significantly more than the law of nature. E No. The claims are directed to one of the four statutory categories of invention. Although the claims would properly be considered to be directed to a law of nature, the claims as a whole do not amount to significantly more than the abstract idea.

ANSWER: (B) This question is modeled directly from Rapid Litigation Management v. CellzDirect, decided by the Federal Circuit on July 5, 2016. The question is adapted to be a rejection from the examiner. In the actual case, the district court held the claims that had already been issued to be invalid for the same reasons mentioned in the question. The Federal Circuit explained that the proper analysis requires the first inquiry to be whether the claims are directed to a patent-ineligible concept. If the answer is no, the inquiry is over and the claim is patent eligible. In this case, the Federal Circuit found that the claims were not directed to a patent-ineligible concept and, therefore, the 101 inquiry ended with a finding that the claims were patent eligible. The Federal Circuit explained: "The '929 patent does not simply claim hepatocytes' ability to survive multiple freeze-thaw cycles. The '929 patent instead claims a method of producing a desired preparation of multicryopreserved hepatocytes. This new and improved technique, for producing a tangible and useful result, falls squarely outside those categories of inventions that are 'directed to' patent-ineligible concepts." Notwithstanding, the Federal Circuit went on to say that even if they were to agree that the claims were directed to a patent-ineligible concept, the claims would still be patent eligible under the second step, which asks whether, considered both individually and as an ordered combination, the additional elements transform the nature of the claim into a patent-eligible application. In order to be patent eligible under step two, more than well-understood, routine, conventional activity already engaged in by the scientific community is required in order to transform the claim into something significantly more than a patent upon the concept itself. The Federal Circuit explained that the claims covered an invention that provided a significant improvement for a variety of reasons, and also because the discovery was used to achieve a new and useful preservation process. It is worth noting, perhaps, that the limitation mentioned relating to the hepatocytes coming from multiple donors was found in claim 5. The Federal Circuit found that claims that did not contain this limitation were still patent eligible

Paul, a registered patent practitioner and counsel for Superior Aircraft, Inc. ("Superior"), filed a patent application naming chief engineer Davis as sole inventor, and claiming a titanium and aluminum alloy designed for use in advanced gas turbine engines in aircraft. The application described the alloy as having unexpectedly excellent and improved room temperature ductility. The application was filed with an assignment document transferring all right, title and interest in the application to Superior. During prosecution of the application, the examiner had an interview with Paul and Davis of Superior. The examiner noted the existence of a prior art publication that disclosed test data demonstrating that the claimed alloys exhibited poor room temperature ductility, and stated that he had personal knowledge that the alloy was old and well known. Davis agreed with the examiner, and stated that such information was "old hat," but that they overcame the ductility problem by simply resorting to a 3-step process of microstructure refinement. Paul concurred and pointed to the fact that not only had they disclosed the process in the application, but that microstructure refinement of alloys to improve ductility was so well-known that the technique was even taught in metallurgy courses in college. Which of the following statements is false? A The examiner may reject the alloy claims on the basis of the prior art publication. B The examiner may not reject the alloy claims on the basis of the prior art publication, because the alloys of the application are characterized by unexpected, improved ductility properties. C The examiner may rely upon the chief engineer's statement as an admission against patentability. D The examiner may rely upon the patent counsel's statement as an admission against patentability. E The examiner, having facts within his or her personal knowledge, may rely on the facts in rejecting the alloy claims.

ANSWER: (B) is best choice because it is a false statement. MPEP § 2112.01 cites Titanium Metals Corp. v. Banner, 778 F.2d 660. 227 USPQ 773 (Fed. Cir. 1985), as stating, "it was immaterial what properties the alloys had...because the composition is the same and thus must necessarily exhibit the properties." (A) is not correct because it is a true statement. (C), (D) and (E) are incorrect because the stated reliance is permitted. 37 C.F.R. § 1.104(c)(3); MPEP § 706.

Which of the following statement(s) is in accordance with patent laws, rules and procedures as related in the MPEP regarding filing of an Application Data Sheet (ADS) in the USPTO? A All non-provisional applications must include an ADS when the application is originally filed. B If an ADS is filed at the same time as an oath or declaration under 37 CFR 1.63 or 1.67 and the information supplied in the two documents is inconsistent, the information provided in the ADS will always govern. C If an ADS is filed at the same time as an oath or declaration under 37 CFR 1.63 or 1.67 and the information supplied in the two documents is inconsistent, the oath or declaration will govern any inconsistency related the claiming of benefit under 35 USC 119(e), 120, 121 or 365(c). D If an ADS is filed after an oath or declaration under 37 CFR 1.63 or 1.67 is filed, and the information supplied in the two documents is inconsistent, the information provided in the ADS will always govern. E The oath or declaration under 37 CFR 1.63 or 1.67 governs inconsistencies with the ADS when the inconsistency concerns setting forth the name(s) of the inventor(s) under 35 USC 115.

ANSWER: (B) is correct. See 37 CFR § 1.76(d), post-AIA

A patent application includes the following Claim 1: Claim 1. A method of making an electrical device comprising the steps of: (i) heating a base made of carbon to a first temperature in the range of 1875 degrees C to 1925 degrees C; (ii) passing a first gas over said heated base, said first gas comprising a mixture of hydrogen, SiCl4, phosphorus, and methane, whereby said first gas decomposes over said heated base and thereby forms a first deposited layer of silicon, phosphorus and carbon on said heated base; (iii) heating said base having said deposited layer to a second temperature of approximately 1620 degrees C; and (iv) passing a second gas over said base heated to said second temperature, said second gas consisting of a mixture of hydrogen, SiCl4, AlCl3, and methane, whereby said second gas decomposes over said heated base to form a second deposit layer adjacent said first layer, said second layer comprising silicon, aluminum and carbon. Assuming proper support in the specification, which of the following claims, if presented in the same application, is a proper claim in accordance with the USPTO rules and the procedures set forth in the MPEP? A Claim 2. The method of claim 1, wherein said first temperature is in the range of 1800 degrees C to 2000 degrees C. B Claim 3. The method of claim 1, wherein said first gas further comprises an inert gas. C Claim 4. The method of claim 1, wherein said second gas further comprises Argon. D Claim 5. The method of claim 1, wherein said first gas is an inert gas such as Argon. E Claim 6. The method of claim 1, wherein said second gas consists of a mixture of hydrogen, SiCl4 and AlCl3 only.

ANSWER: (B) is the most correct answer. 37 CFR § 1.75(c). Answers (A) and (E) are incorrect because they improperly seek to broaden the parent claim. 37 CFR § 1.75(c). Answer (A) broadens the range by going below the stated limit. Answer (E) broadens by trying to remove a recited component of the second gas, and covering subject matter that is not covered by the parent claim. Answer (C) is incorrect because claim 1 uses the close ended claim term "consists" in connection with the second gas, which precludes the addition of further components to the second gas in claim 4. Answer (D) is incorrect because the use of the exemplary language "such as" is improper is improper under 35 U.S.C. § 112, second paragraph, and because it is inconsistent with claim 1. See MPEP § 2173.05(d)

Which of the following is not prohibited conduct for a practitioner under the USPTO Code of Professional Responsibility? A Entering into an agreement with the client to limit the amount of any damages which the client may collect for any mistakes the practitioner may make during prosecution of the client's patent application in exchange for prosecuting the application at a reduced fee. B Encouraging the client to meet with an opposing party for settlement discussions. C Failing to disclose controlling legal authority which is adverse to the practitioner's client's interest when arguing the patentability of claims in a patent application. D In reply to an Office action, stating honestly and truthfully in the remarks accompanying an amendment that the practitioner has personally used the device and found it to be very efficient and better than the prior art. E Investing the funds the client advanced for the practitioner legal fees (not costs and expenses) in long term United States Treasury Bills in order to obtain guaranteed protection of the principal.

ANSWER: (B) is the most correct answer. CFR Part 11 generally. As to (A), practitioner may not limit damages. As to (C), see Rule 56. As to (D), personal use is entitled to no weight. As to (E), client funds advanced for legal services are required to be deposited in a bank account.

Which of the following best describes a situation for which a reply to the examiner's Office action in a pre-AIA filed application including both an affidavit filed under 37 CFR 1.131 and an affidavit filed under 37 CFR 1.132 may be in accord with the patent laws, rules and procedures as related in the MPEP? A In a timely reply to a non-final Office action, where the examiner's sole rejection of appellant's claims is based on an alleged violation of the enablement requirement of 35 USC 112. B In a timely reply to non-final Office action, where the examiner's sole rejection of appellant's claims is a rejection under 35 USC 103(a) employing a non-patent document that was published less than one year prior to the filing date of appellant's patent application. C In a timely reply to a non-final Office action, where the examiner's sole rejection of appellant's claims is a rejection under 35 USC 103(a) employing a non-commonly owned U.S. patent as prior art under 35 USC 102 (e) that claims the same invention as applicant. D In a timely reply to an examiner's answer presenting the affidavits for the first time, where in the examiner's first Office action and final rejection, the examiner maintains the same rejection under 35 USC 103(a) of all of appellant's claims based in part on a nonpatent document that was published less than one year prior to the filing date of appellant's patent application. E In a timely reply to a final Office action presenting the affidavits for the first time, where in the examiner's first Office action, the examiner's sole rejection of appellant's claims is a rejection under 35 USC 103(a) employing a non-patent document that was published less than one year prior to the filing date of appellant's patent application.

ANSWER: (B) is the most correct answer. MPEP § 715, under the heading "Situations Where 37 CFR 1.131 Affidavits or Declarations Can Be Used," provides that an affidavit under 37 CFR § 1.131 may be used to antedate a reference that qualifies as prior art under 35 U.S.C. § 102 (a) but not 35 U.S.C. § 102(b). Also, MPEP § 716 provides that objective evidence traversing a rejection may be presented in a timely submitted affidavit under 37 CFR § 1.132. As for (A), an affidavit under 37 CFR § 1.131 would not serve any useful purpose in those situations. See MPEP § 715. Regarding (C), an affidavit under 37 CFR § 1.131 would be inappropriate. See MPEP § 715, under the heading "Situations Where 37 CXFR 1.131 Affidavits or Declarations Are Inappropriate." As for (D) and (E), a reply including affidavits under 37 CFR § 1.131 and 37 CFR § 1.132 normally would be considered untimely under the circumstances set forth in those answers. See 37 CFR §§ 1.116, 1.192 (a) and 1.195. Also, see MPEP §§ 715.09 and 716.01. Consequently, (B) is the most correct answer.

The Potter patent application was filed on June 6, 2015, claiming subject matter invented by Potter. The Potter application properly claims priority to a German application filed on June 6, 2014. A first Office action contains a rejection of all the claims of the application under 35 USC 103 based on a U.S. patent application publication to Smith in view of a U.S. patent to Jones. A registered practitioner prosecuting the Potter application ascertains that the relevant subject matter in Smith's published application and Potter's claimed invention were, at the time Potter's invention was made, owned by ABC Company or subject to an obligation of assignment to ABC Company. The practitioner also observes that the Smith patent application was filed on April 10, 2014 and that the patent application was published on December 5, 2015. Smith and Potter do not claim the same patentable invention. To overcome the rejection without amending the claims, which of the following timely replies would comply with the USPTO rules and the procedures set forth in the MPEP to be an effective reply for overcoming the rejection? A A reply that only contains arguments that Smith fails to teach all the elements in the only independent claim, and which specifically points out the claimed element that Smith lacks. B A reply that properly states that the invention of the Potter application and the Smith application were commonly owned by ABC Company at the time of the invention and later filing of the Potter application. C A reply that consists of an affidavit or declaration under 37 CFR 1.132 stating that the affiant has never seen the invention in the Potter application before. D A reply that consists of an affidavit or declaration under 37 CFR 1.131 properly proving invention of the claimed subject matter of Potter application only prior to June 6, 2014. E A reply that consists of a proper terminal disclaimer and affidavit or declaration under 37 CFR 1.130.

ANSWER: (B) is the most correct answer. See 35 U.S.C. § 102(b)(2)(c). The prior art exception in 35 U.S.C. § 102 is applicable because the Smith reference is only prior art under 35 U.S.C. § 102(a)(2), was applied in a rejection under 35 U.S.C. § 103, and was commonly owned at the time Potter made the invention claimed by Potter. (A) is not a correct answer in that one cannot show nonobviousness by attacking the references individually where the rejections are based on a combination of references. See MPEP § 2145. (C) is not a correct answer. An affirmation that the affiant has never seen the invention before is not relevant to the issue of nonobviousness of the claimed subject matter. See MPEP 716. (D) is not a correct answer. Invention must be proved prior to the effective filing date of Smith, which is April 10, 2014. See MPEP § 715. (E) is not a correct answer. A terminal disclaimer and affidavit or declaration under 37 CFR § 1.130 are not proper because the Potter application and the Smith reference are not claiming the same patentable invention.

In accordance with the USPTO rules and the procedures set forth in the MPEP, which of the following is true? A When the subject matter of an appeal is particularly difficult to understand, a patentability report is prepared by an examiner in order to present the technical background of the case to the Board of Patent Appeals and Interferences. B In those appeals in which an oral hearing has been confirmed and either the Board of Patent Appeals and Interferences or the primary examiner has indicated a desire for the examiner to participate in the oral argument, oral argument may be presented by the examiner whether or not the appellant appears. C If a patent applicant files a notice of appeal which is unsigned, it will be returned for signature, but the applicant will still receive the filing date of the unsigned notice of appeal. D Statements made in information disclosure statements are not binding on an applicant once the patent has issued since the sole purpose of the statement is to satisfy the duty of disclosure before the Office. E None of the above.

ANSWER: (B) is the most correct answer. See MPEP § 1209, under the heading "Participation by Examiner." "oral argument will generally only be held if the appellant actually appears for oral argument. If the appellant fails to make an appearance for oral argument, the hearing typically will be waived, and the primary examiner will not be given an opportunity to participate unless the panel hearing the case deems that a presentation by the primary examiner would be useful." [emphasis added] As to (A), see MPEP § 705. As to (C), signature requirement does not apply. MPEP § 1205. The notice will not be returned. As to (D), see Gentry Gallery v. Berkline Corp., 134 F.3d 1473, 45 U.S.P.Q.2d 1498 (Fed. Cir. 1998).

In accordance with the USPTO rules and the procedures set forth in the MPEP, which of the following presents a Markush group in accordance with proper PTO practice and procedure? A R is selected from the group consisting of A, B, C, or D. B R is selected from the group consisting of A, B, C, and D. C R is selected from the group comprising A, B, C, and D. D R is selected from the group comprising A, B, C or D. E R is A, B, C, and D.

ANSWER: (B) is the most correct answer. See MPEP § 2173.05(h). A Markush group is an acceptable form of alternative expression provided the introductory phrase "consisting of," and the conjunctive "and" are used. (A) and (D) are incorrect because the conjunctive "or" is used. (C) and (D) are incorrect because the introductory phrase "comprising" is used. (E) is incorrect because R must simultaneous be A, B, C, and D, as opposed to being a single member of the group, i.e., no language provides for the selection of one of the members of the group of A, B, C, and D. MPEP § 2173.05(h).

Which, if any, of the following would be considered patent ineligible? I. The claims cover a well-known concept of categorical data storage, i.e., the idea of collecting information in classified form, then separating and transmitting that information according to its classification. II. The claims cover digital image halftoning. Halftoning techniques allow computers to present many shades and color tones with a limited number of pixels, which allows computer displays and printers to render an approximation of an image by using fewer colors or shades of gray than the original image. One method of generating a digital halftoned image is called ''thresholding'' that uses a two-dimensional array called a ''mask.'' The claimed method incorporates algorithms and formulas that control the masks and halftoning, but apply them in a technique that improves the generated digital halftoned image. III. The claims cover a GPS satellite navigation system comprising satellites orbiting the Earth that permits a GPS-enabled receiver to detect signals from at least four satellites and use that information to calculate its distance from each satellite and thus its precise position on Earth through trilateration. The claim sets forth the steps of calculating the absolute position, which is a mathematical concept. IV. The claims relate to managing a bingo game while allowing a player to repeatedly play the same sets of numbers in multiple sessions. V. The claim addresses the problem of retaining Web site visitors from being diverted from a host's Web site to an advertiser's Web site, for which ''the claimed solution is necessarily rooted in computer technology in order to overcome a problem specifically arising in the realm of computer networks.'' The claim includes additional elements including: (1) sorting visually perceptible elements; (2) on link activation, automatically identifying the host; and (3) instructing the web server to construct and serve to the visitor a new, hybrid Web page that merges content associated with the products of the third-party merchant with the stored visually perceptible elements from the identified host Web site. A I. B I and III C I and IV D I, III and IV E All of the above

ANSWER: (C) Only I and IV are properly considered patent ineligible. I relates to Cyberfone Systems v. CNN Interactive Group (U.S. Patent No. 8,019,060), which was found to an abstract idea that was patent ineligible. II relates to Research Corp. Tech. v. Microsoft Corp. (U.S. Patent No. 5,111,310), where the invention presented functional and palpable applications in the field of computer technology with specific applications or improvements to technologies in the marketplace. So, although the claimed method uses algorithms and formulas, the claim was found eligible. III relates to SiRF Technology v. ITC (U.S. Patent No. 6,417,801). The court interpreted the claim such that the method could not be performed without a GPS receiver. With this interpretation, the presence of the GPS receiver in the claim places a meaningful limit on the scope of the claim. It is essential to the operation of the claimed method and plays a significant part in permitting the claimed method to be performed. As such, although performance of the claim requires calculations, the claim was found eligible. IV relates to Planet Bingo, LLC v. VKGS LLC (U.S. Patent No. 6,398,646). Managing the game of bingo consists solely of mental steps which can be carried out by a human using pen and paper. The claims do not impose any requirements that would make the invention impossible to carry out manually. V relates to DDR Holdings, LLC v. Hotels.com, L.P. (U.S. Patent No. 7,818,399). In DDR, the court held that, unlike in Ultramercial, the claim does not generically recite ''use the Internet'' to perform a business practice, but instead recites a specific way to automate the creation of a composite Web page by an outsource provider that incorporates elements from multiple sources in order to solve a problem faced by Web sites on the Internet. Therefore, the court held that the claim is patent eligible.

The USPTO notifies John, a registered patent agent who is representing applicant A, that after a reasonable search, the USPTO has been unable to locate applicant A's patent application. By which of the following procedures may John avoid abandonment of applicant A's application within the time period set by the USPTO? A Provide the USPTO with a copy of his record of all the correspondence between his office and the USPTO, assuming the existence of such record. B Provide the USPTO with a list of all the correspondence between his office and the USPTO, assuming the existence of such list, and a statement that the list is complete and accurate. C Provide the USPTO with a statement that he does not possess any record of the correspondence between his office and the USPTO because his files were destroyed. D Provide the USPTO with a record of all the correspondence between his office and the USPTO, and a statement that the papers produced are his complete record of all the correspondence between his office and the USPTO, assuming the existence of such record. E Provide the USPTO with a copy of his record of all the correspondence between his office and the USPTO, assuming the existence of such record, a list of all such correspondence, and a statement that he is not aware of any correspondence between his office and the USPTO that is not among his records.

ANSWER: (C) is correct. 37 C.F.R. § 1.251(a)(3). (C) is correct because there is compliance with 37 C.F.R. § 1.251(a)(3). (A) is wrong because along with a copy of the record, he is required to provide a list of all correspondence, and a statement that the copy is complete and accurate and that he is not aware of any correspondence between his office and the USPTO that is not among his records. 37 C.F.R. § 1.251 (a)(1)(ii). (B) is wrong because along with a list of all correspondence and a statement that the copy of his record of all the correspondence is complete and accurate, he is required to provide a copy of his record of all the correspondence, and the statement must recite that he is not aware of any correspondence between his office and the USPTO that is not among his records. 37 C.F.R. § 1.251(a)(1)(i). (D) is wrong because the statement omits the recitation that he is not aware of any correspondence between his office and the USPTO that is not among his records. 37 C.F.R. § 1.251(a)(2)(ii). (E) is wrong because the statement omits the recitation that the copy of his record of all the correspondence is complete and accurate. 37 C.F.R. § 1.251(a)(1)(iii).

A registered practitioner timely files a petition under 37 CFR 1.181 while the application is pending before the primary examiner to challenge the prematureness of the final rejection that set a shortened statutory period for reply. Assume the petition is filed within two months of the date on the final rejection. What is the next response that should be docketed by the practitioner in accordance with the patent laws, rules and the procedures as related in the MPEP to avoid a penalty or payment of fees? A A reply to the final rejection within 6 months. B A status inquiry 6 months after filing the petition. C A reply to the final rejection within the shortened statutory time period set for reply in the final rejection. D No reply is necessary until a decision is received on the petition. E All of the above.

ANSWER: (C) is correct. 37 CFR § 1.181(f); MPEP §§ 714.13 and 1002: 37 CFR § 1.181(f) provides "The mere filing of a petition will not stay the period for reply to an Examiner's action which may be running against an application, nor act as a stay of other proceedings." MPEP §§ 714.13 states "[i]t should be noted that under 37 CFR 1.181(f), the filing of a 37 CFR 1.181 petition will not stay the period for reply to an examiner's action which may be running against an application." See also MPEP § 1002. Thus, if a petition to vacate a final rejection as premature is filed within 2 months from the date of the final rejection, the period for reply to the final rejection is not extended even if the petition is not reached for decision within that period. However, if the petition is granted and the applicant has filed an otherwise full reply to the rejection within the period for reply, the case is not abandoned. (C) is correct because the petition does not stay the time for responding to the final and a reply should be filed within the shortened statutory period to avoid fees. (A) is incorrect because any reply after the shorted statutory period set for reply in the final rejection will require at least payment of fees for an extension of time. MPEP § 1002 states "The mere filing of a petition will not stay the period for replying to an examiner's action which may be running against an application, nor act as a stay of other proceedings (37 CFR 1.181(f)). For example, if a petition to vacate a final rejection as premature is filed within 2 months from the date of the final rejection, the period for reply to the final rejection is not extended even if the petition is not reached for decision within that period." (B) is incorrect because the application will be abandoned 6 months after the date of the final rejection for lack of reply to the final rejection, 35 U.S.C. 133, and a status inquiry filed 6 months after filing the petition would be in an abandoned application. Moreover, a petition status inquiry filed at any time is not a proper reply to the final rejection. See 37 CFR §§ 1.113(c) and 1.116. (D) is incorrect because the filing of a petition does not stay the period for reply. 37 CFR § 1.181(f); MPEP §§ 714.13 and 1002. (E) is incorrect because (C) is correct, and (A), (B) and (D) are incorrect.

Which of the following statements is true in a pre-AIA filing? A In the context of 35 USC 102(b), a magazine need only be placed in the mail to be effective as a printed publication. B The earliest date declassified printed material may be taken as prima facie evidence of prior knowledge under 35 USC 102(a) is as of the date the material is cataloged and placed on the shelf of a public library. C Declassified printed material is effective as a printed publication under 35 USC 102(b) as of the date of its release following declassification. D The American Inventors Protection Act (AIPA) amended 35 USC 102(e) to provide that U.S. patents, U.S. application publications, and certain international application publications can be used as prior art under 35 USC 102(e) based on their earliest effective filing date only against applications filed on or after November 29, 2000. E The American Inventors Protection Act (AIPA) amended 35 USC 102(e) to provide that U.S. patents, U.S. application publications, and certain international application publications can be used as prior art under 35 USC 102(e) based on their earliest effective filing date only against applications filed prior to November 29, 2000 which have been voluntarily published.

ANSWER: (C) is correct. MPEP § 707.05(f) states, "In the use of [declassified material] ... as an anticipatory publication, the date of release following declassification is the effective date of publication within the meaning of the statute." (A) is wrong. MPEP § 706.02(a) states, "A magazine is effective as a printed publication under 35 U.S.C. § 102(b) as of the date it reached the addressee and not the date it was placed in the mail." (B) is wrong. MPEP § 707.05(f) states, "For the purpose of anticipation predicated upon prior knowledge under 35 U.S.C. §102(a) the above noted declassified material may be taken as prima facie evidence of such prior knowledge as of its printing date even though such material was classified at that time." (D) and (E) are wrong. The AIPA amended 35 U.S.C. § 102(e) to provide that U.S. patents, U.S. application publications, and certain international application publications can be used as prior art under 35 U.S.C. § 102(e) based on their earliest effective filing date against applications filed on or after November 29, 2000, and applications filed prior to November 29, 2000 which have been voluntarily published. MPEP § 706.02(a).

On January 2, 2016, a registered practitioner filed a patent application with the USPTO for inventor Bloc. The application includes a specification and a single claim to the invention which reads as follows: 1. Compound Y. In the specification, Bloc explains that compound Y is an intermediate in the chemical manufacture of synthetic Z. With respect to synthetic Z, the specification discloses its structural formula and further states that synthetic Z is modeled on the natural form of Z to give it the same therapeutic ability to alleviate pain. The specification goes on to state that synthetic Z is also a cure for cancer. On June 2, 2016, the practitioner received an Office action from the primary examiner rejecting the claim. The claim is rejected under 35 USC 101 as being inoperative; that is, the synthetic Z does not operate to produce a cure for cancer (i.e., incredible utility). Bloc believes he is entitled to a patent to his compound Y. In accordance with USPTO rules and procedures set forth in the MPEP, how best should the practitioner reply to the rejection of the claim? A Advise Bloc that he should give up because a cure for cancer is indeed incredible and is unproven. B File a reply arguing that a cure for cancer is not incredible and he can prove it if given the chance. C File a reply arguing that whether or not a cure for cancer is incredible is superfluous since Bloc has disclosed another utility - alleviating pain, which is not incredible. D File a reply arguing that the claim is directed to compound Y, not synthetic Z. E File a reply arguing that synthetic Z is modeled on the natural form of Z.

ANSWER: (C) is the best answer. MPEP §§ 2107.01 and 2107.02. MPEP § 2107.01, under the heading "Therapeutic or Pharmacological Utility," cites In re Chilowsky, 229 F.2d 457, 461-2, 108 USPQ 321, 325 (CCPA 1956); In re Gazave, 379 F.2d 973, 978, 154 USPQ 92, 96 (CCPA 1967); and Nelson v. Bowler, 626 F.2d 853, 856, 206 USPQ 881, 883 (CCPA 1980) as taking the position that "[i]nventions asserted to have utility in the treatment of human or animal disorders are subject to the same legal requirements for utility as inventions in any other field of technology." MPEP § 2107.02, under the heading "The Claimed Invention Is The Focus Of The Utility Requirement," states "...regardless Answer Reasoning: of the category of invention that is claimed (e.g., product or process), an applicant need only make one credible assertion of specific utility for the claimed invention to satisfy 35 U.S.C. 101 and 35 U.S.C. 112; additional statements of utility, even if not "credible," do not render the claimed invention lacking in utility. See, e.g.,...In re Gottlieb, 328 F.2d 1016, 1019, 140 USPQ 665, 668 (CCPA 1964) ('Having found that the antibiotic is useful for some purpose, it becomes unnecessary to decide whether it is in fact useful for the other purposes 'indicated' in the specification as possibly useful.')." The issue is whether Mr. Bloc has disclosed a specific utility for the claimed compound Y sufficient to satisfy the practical utility requirement of 35 U.S.C § 101. According to the set of facts, we know that compound Y is an intermediate in the chemical manufacture of synthetic Z. We are given two utilities for synthetic Z: 1) alleviating pain, a utility it shares with the natural form of Z; and, 2) curing cancer. The examiner focuses on the disclosure that synthetic Z is a cure for cancer. Even if one were to agree that synthetic Z's ability to cure cancer amounts to an incredible utility, a claim to the intermediate compound Y would not run afoul of the utility requirement of 35 U.S.C. § 101 where another substantial, credible and specific utility is alternatively demonstrated. Here, the specification discloses that synthetic Z, like the natural form of Z, alleviates pain. The alleviation of pain is another substantial, credible and specific utility and serves to give compound Y an alternative utility to that of being used to make a cancer-curing substance. An applicant need not show that all disclosed utilities are credible. An applicant need only show that one of the disclosed utilities is in fact credible. In re Gottlieb, supra. The establishment of a credible, substantial and specific utility renders the disclosure of an additional incredible utility superfluous, and therefore ultimately irrelevant. Accordingly, Mr. Bloc's best course of action is to make the argument that he has disclosed another substantial, credible, and specific utility, notwithstanding the disclosure of curing cancer. (A) is not the most correct answer. The advice could prevent him from getting a patent to which he may be entitled. (B) is not the most correct answer. A cure for cancer is ostensibly incredible. It is hardly a response to the examiner's rejection to ask for the chance to prove one can cure cancer. (D) is not the most correct answer. While it is true that the utility requirement is addressed to the claimed invention, which here is compound Y not synthetic Z, it is not enough to respond by repeating what the invention is but, rather, to show that the invention has indeed a substantial, credible, and specific utility. Whatever is claimed as the invention, it must comply with the utility requirement of 35 U.S.C. § 101. Here the examiner states that the claim does not comply, as evidenced by the incredible utility of the final product. It is Mr. Bloc's responsibility to then show that compound Y does comply with 35 U.S.C. § 101. (E) is not the most correct answer. Noting that synthetic Z is modeled on natural Z does not go far enough in establishing a substantial, credible and specific utility for compound Y. It is synthetic Z's therapeutic ability to alleviate pain which establishes the necessary alternative utility.

A registered practitioner files an international application submission that includes a description, claims and drawings in the United States Receiving Office (RO/US) on Wednesday, January 8, 2003. The submission did not include the required request, international and search fees, or the designation of a PCT contracting State. The RO/US mails an "Invitation to Correct the Purported International Application," dated January 10, 2003, to the practitioner indicating that the designation of at least one Contracting State, as required by PCT Article 11(1)(iii)(b), was not included. A one-month period for response is set in the Invitation. On Monday, February 10, 2003, the practitioner submits by facsimile a designation sheet of the Request Form designating every available Contracting State, and authorization to charge all required fees. In accordance with the patent laws, rules and procedures as related in the MPEP, will the application be accorded an international filing date? A Yes. The application will be accorded a filing date of January 8, 2003. B Yes. The application will be accorded an international filing date of February 10, 2003. C No. The application will not be accorded an international filing date because the failure to designate at least one contracting State cannot be cured by a facsimile transmission. D No. The application was given a one-month period for response. The practitioner would have had to have filed the response on Friday, February 7, 2003 in order to have been timely. E None of the above

ANSWER: (C) is the correct answer. 37 CFR §§ 1.6(d)(3) and 1.8(a)(2) (i)(d); MPEP § 502 (reproducing Rule 1.6(d)(3)); MPEP § 512 (reproducing Rule 1.8(a)(2)(i)(d)); and MPEP §1817.01. As stated in MPEP § 1817.01, "[a]ll designations must be made in the international application on filing; none may be added later." The application will not be accorded an international filing date since the practitioner has tried to cure the failure to designate at least one contracting State by filing a paper using facsimile which is not permitted according to 37 CFR §§ 1.6 (d)(3) and 1.8(a)(2)(i)(d). (But, bear in mind since 1/1/04 - a designation is automatic as a part of a PCT filing.) (A) is wrong because applicant has failed to comply with Article 11(1)(iii)(b) on such date. See MPEP § 1810 (reproducing PCT Article 11(1)(iii)(b). (B) is wrong because according to 37 CFR 1.6(d)(3) and 37 CFR 1.8 (a)(2)(i)(d), applicant cannot file an international application by facsimile. See MPEP § 502 (reproducing 37 CFR § 1.6(d)(3)); MPEP § 512 (reproducing 37 CFR § 1.8(a)(2)(i)(d)). Since no designations were included on filing, the application papers cannot be accorded an international filing date. See PCT Article 11(1)(iii)(b). Applicant cannot correct this by filing the designation sheet by facsimile. See MPEP § 502 (reproducing Rule 1.6 (d)(3)); MPEP § 512 (reproducing Rule 1.8(a)(2)(i)(d)). (D) is wrong because according to PCT Rule 80.5, when a response is due on a day where the receiving Office is not open for business, applicant has until the next business day. See Appendix T of the MPEP. (E) is incorrect because (C) is correct.

A claim in an application recites "[a] composition containing: (a) 35-55% polypropylene; and (b) 45-65% polyethylene." The sole prior art reference describes, as the only relevant disclosure, a composition containing 34.9% polypropylene and 65.1% polyethylene. In accordance with USPTO rules and procedures set forth in the MPEP, the primary examiner should properly: A Indicate the claim allowable over the prior art because there is no teaching, motivation or suggestion to increase the amount of polypropylene from 34.9% to 35% and decrease the amount of polyethylene from 65.1% to 65%. B Reject the claim under 35 USC 102 as anticipated by the prior art reference. C Reject the claim under 35 USC 103 as obvious over the prior art reference. D Reject the claim alternatively under 35 USC 102 as anticipated by or under 35 USC 103 as obvious over the prior art reference. E None of the above.

ANSWER: (C) is the most correct answer. A prima facie case of obviousness exists where the claimed ranges and the prior art are close enough that one of ordinary skill in the art would have expected them to have the same properties. See MPEP § 2144.05. In Titanium Metals Corp. v. Banner, 778 F.2d 775, 783, 227 USPQ 773, 779 (Fed. Cir. 1985), a claim recited a titanium base alloy consisting essentially of 0.8% nickel, 0.3% molybdenum, up to 0.1% maximum iron, and the balance titanium. A prior art reference described two similar alloys: (i) one with 0.25% molybdenum, 0.75% nickel, and balance titanium; and (ii) another with 0.31% molybdenum, 0.94% nickel, and balance titanium. The court held: As admitted by appellee's affidavit evidence from James A. Hall, the Russian article discloses two alloys having compositions very close to that of claim 3, which is 0.3% Mo and 0.8% Ni, balance titanium. The two alloys in the prior art have 0.25% Mo0.75% Ni and 0.31% Mo-0.94% Ni, respectively. The proportions are so close that prima facie one skilled in the art would have expected them to have the same properties. Appellee produced no evidence to rebut that prima facie case. The specific alloy of claim 3 must therefore be considered to have been obvious from known alloys. Id. Thus, (A) is incorrect. (B) and (D) are incorrect because a claim is anticipated by a prior art reference only when the prior art discloses, either expressly or inherently, every limitation of the claimed invention. (E) is incorrect because (C) is correct.

Mark Twine obtains a patent directed to a machine for manufacturing string. The patent contains a single claim (Claim 1) which recites six claim elements. The entire interest in Twine's patent is assigned to the S. Clemens String Co., and Twine is available and willing to cooperate with S. Clemens String Co. to file a reissue application. A subsequent reissue application includes Claim 2, which is similar to original Claim 1. However, one of the elements recited in Claim 2 is broader than its counterpart element in the original claim. The remaining five elements are narrower than their respective counterpart elements in the original patent claim. Which of the following scenarios accords with the USPTO rules and the procedures set forth in the MPEP? A The S. Clemens String Co. files the reissue application more than 2 years after the issue date of the original patent application. B The S. Clemens String Co. files the reissue application less than 2 years after the issue date of the original patent but more than 2 years after original application filing date. C Mark Twine files the reissue application less than 2 years after the issue date of the original patent but more than 2 years after original application filing date. D Mark Twine files the reissue application more than 2 years after the issue date of the original patent. E Mark Twine and the S. Clemens String Co. jointly file the reissue application more than 2 years after the issue date of the original patent.

ANSWER: (C) is the most correct answer. Answers (A), (D) and (E) are incorrect because a broadening reissue application must be filed within two years of issuance of the original patent. 35 U.S.C. § 251; MPEP § 1412.03. Answer (B) is incorrect because the assignee may not file a broadening reissue application. MPEP § 706.03(x).

The primary examiner has rejected claims 1-10 under 35 USC 103 as being unpatentable over the Smith patent in view of the Jones reference. Appellant properly argues that there is no motivation or KSR rationale to combine the teachings of Smith and Jones. The examiner repeats the rejection of claims 1-10 as being "unpatentable over Smith in view of Jones." The examiner additionally cites a patent to Brown that was necessary to provide motivation for combining the teachings of Smith and Jones. The examiner does not list Brown in the statement of the rejection. Appellant timely appeals to the PTAB, and files a proper appeal brief. The examiner files an examiner's answer addressing the rejection of claims 1-10 under 35 USC 103 as being unpatentable over Smith in view of Jones, and cites Brown in the argument as providing motivation to combine Smith and Jones. In accordance with the patent laws, rules and procedures as related in the MPEP, what will be the most proper decision of the Board? A The Board will affirm the rejection based on Smith and Jones only. B The Board will affirm the rejection based on Smith, Jones and Brown. C The Board will reverse the rejection based on Smith and Jones only. D The Board will reverse the rejection based on Smith, Jones and Brown. E None of the above.

ANSWER: (C) is the most correct answer. MPEP § 1207.03. If the claimed invention is rendered obvious by Smith in view of Jones and Brown, the statement of rejection must include all three references. Reliance on Brown to support the rejection is a different rejection from a rejection relying only on Smith in view of Jones. In accordance with MPEP § 1208.01, the Board will not consider the teachings of Brown because Brown was used to support the rejection, but was not listed in the statement of the rejection. As stated in MPEP § 1208.01, "Even if the prior art reference is cited to support the rejection in a minor capacity, it should be positively included in the statement of rejection. In re Hoch, 428 F.2d 1341, 1342 n.3, 166 USPQ 406, 407 n.3 (CCPA 1970)." Therefore, (B) and (D) are clearly wrong. (A) is incorrect. The decision cannot affirm the rejection since there is no motivation for combining the teachings of Smith and Jones absent the teachings of Brown. Therefore, the rejection must be reversed, not affirmed.

Assuming that a rejection has been properly made final, which of the following statements is not in accordance with the patent laws, rules and procedures as related in the MPEP? A An objection and requirement to delete new matter from the specification is subject to supervisory review by petition under 37 CFR 1.181. B A rejection of claims for lack of support by the specification (new matter) is reviewable by appeal to the Board of Patent Appeals and Interferences. C If both the claims and the specification contain the same new matter, and there has been both a rejection and objection by the primary examiner, the new matter issue should be decided by petition, and is not appealable. D If both the claims and the specification contain the same new matter, and there has been both a rejection and objection by the examiner, the new matter issue is appealable, and should not be decided by petition. E None of the above.

ANSWER: (C) is the most correct answer. MPEP § 2163.06, under the heading "Review Of New Matter Objections And Rejections," states "A rejection of claims is reviewable by the Board of Patent Appeals and Interferences, whereas an objection and requirement to delete new matter is subject to supervisory review by petition under 37 CFR 1.181. If both the claims and specification contain new matter either directly or indirectly, and there has been both a rejection and objection by the examiner, the issue becomes appealable and should not be decided by petition." Answer (C) is not accord with the USPTO rules and the procedures set forth in the MPEP. (A), (B) and (D) are incorrect. They are in accord with proper USPTO procedure. See MPEP § 2163.06, under the heading "Review Of New Matter Objections And Rejections." (E) is not correct because (C) is correct. MPEP § 2163.06.

Assuming that any rejection has been properly made final, which of the following statements is not in accordance with the USPTO rules and the procedures set forth in the MPEP? A An objection and requirement to delete new matter from the specification is subject to supervisory review by petition under 37 CFR 1.181. B A rejection of claims for lack of support by the specification (new matter) is reviewable by appeal to the Board of Patent Appeals and Interferences. C If both the claims and the specification contain the same new matter, and there has been both a rejection and objection by the primary examiner, the new matter issue should be decided by petition, and is not appealable. D If both the claims and the specification contain the same new matter, and there has been both a rejection and objection by the examiner, the new matter issue is appealable, and should not be decided by petition. E None of the above.

ANSWER: (C) is the most correct answer. MPEP § 2163.06, under the heading "Review Of New Matter Objections And Rejections," states "[a] rejection of claims is reviewable by the Board of Patent Appeals and Interferences, whereas an objection and requirement to delete new matter is subject to supervisory review by petition under 37 CFR 1.181 . If both the claims and specification contain new matter either directly or indirectly, and there has been both a rejection and objection by the examiner, the issue becomes appealable and should not be decided by petition." Answer (C) is not in accordance with the USPTO rules and the procedures set forth in the MPEP. (A), (B) and (D) are incorrect. They are in accord with proper USPTO procedure. See MPEP § 2163.06, under the heading "Review Of New Matter Objections And Rejections." (E) is not correct because (C) is correct. MPEP § 2163.06.

Sam is a sole proprietor of Sam's Labs, which has no other employees. Sam invented a new drug while doing research under a Government contract. Sam desires to file a patent application for his invention and assign it to Sam's Labs. Sam has licensed Rick, also a sole proprietor with no employees, to make and use his invention. Sam wants to claim small entity status when filing a patent application for his invention. Sam also wants to grant the Government a license, but will not do so if he will be denied small entity status. Sam has limited resources and wants to know whether, how, and to what extent he may claim small entity status. Which of the following is not in accord with the USPTO rules and the procedures set forth in the MPEP? A Sam's Labs is a small business concern for the purposes of claiming small entity status for fee reduction purposes. B If Sam grants a license to the Government resulting from a rights determination under Executive Order 10096, it will not constitute a license so as to prohibit claiming small entity status. C The establishment of small entity status permits the recipient to pay reduced fees for all patent application processing fees charged by the USPTO. D Sam may establish small entity status by a written assertion of entitlement to small entity status. A written assertion must: (i) be clearly identifiable; (ii) be signed; and (iii) convey the concept of entitlement to small entity status, such as by stating that applicant is a small entity, or that small entity status is entitled to be asserted for the application or patent. E While no specific words or wording are required to assert small entity status, the intent to assert small entity status must be clearly indicated in order to comply with the assertion requirement.

ANSWER: (C) is the most correct answer. Not all fees are subject to the small entity reduction. See, for example, 37 CFR § 1.17(q). As to (A), a small business concern for the purposes of claiming small entity status for fee reduction purposes is any business concern that: (i) has not assigned, granted, conveyed, or licensed, and is under no obligation under contract or law to assign, grant, convey, or license, any rights in the invention to any person, concern, or organization which would not qualify for small entity status as a person, small business concern, or nonprofit organization. and (ii) meets the standards set forth in the appropriate section of the code of federal regulations to be eligible for reduced patent fees. Sam's Labs meets all of the elements required by 37 CFR § 1.27 (a)(2). Statement (B) contains all of the elements required by 37 CFR § 1.27 (a)(4). Statement (D) contains all of the elements required by 37 CFR § 1.27 (c)(1). Statement (E) contains all of the elements required by 37 CFR § 1.27 (c)(1)(iii).

The claims in a patent application having been twice or finally rejected, the applicant files a timely Notice of Appeal on January 2, 2016. In accordance with USPTO rules and procedures set forth in the MPEP, which of the following situations should the USPTO not notify the applicant that the Appeal Brief is defective and allow him an opportunity to correct the deficiency? A The Appeal Brief is filed on July 10, 2016, without a request for extension of time under 37 CFR 1.136. B The Appeal Brief is submitted unsigned. C The Appeal Brief states that the claims do not stand or fall together, and presents argument as to why the claims are separately patentable, but the primary examiner does not agree with the applicant's argument. D The Appeal Brief does not state whether the claims stand or fall together, but presents arguments why the claims subject to the same rejection are separately patentable. E The Appeal Brief does not address any of the grounds of rejection stated by the primary examiner, has no argument section, and is non-compliant as to 37 CFR 41.37.

ANSWER: (C) is the most correct answer. See MPEP § 1207. If the examiner disagrees with the reasons given, the reason for disagreement should be addressed in the Examiner's Answer. If the brief includes a statement that a grouping of claims does not stand or fall together but does not provide reasons, [the examiner is to] notify appellant of the non -compliance. If the examiner disagrees with appellant's statement in the brief that certain claims do not stand or fall together, the examiner explains in the examiner's answer why the claim grouping listed in the brief is not agreed with and why, if appropriate, e.g., the claims as listed by the appellant are not separately patentable. Answer (A) is incorrect. The Appeal Brief was filed less than seven months after the Notice of Appeal was filed. The applicant should be notified of the deficiency and provided an opportunity to request a fivemonth extension of time. Answer (B) is incorrect. See MPEP § 1205.03.. Answer (D) is incorrect. Where the applicant omits the statement required by 37 CFR § Part 41, yet presents arguments in the argument section of the brief, the applicant should be notified of the noncompliance and given time to correct the deficiency. See 37 CFR § Part 41; and MPEP § 1205.02. Answer (E) is incorrect. MPEP § 1205.03. Where the brief is noncompliant in some material way under 41.37, the applicant will be notified

To satisfy the written description requirement of the first paragraph of 35 USC 112, an applicant must show possession of the invention. An applicant's lack of possession of the invention may be evidenced by: A Describing an actual reduction to practice of the claimed invention. B Describing the claimed invention with all of its limitations using such descriptive means as words, structures, figures, diagrams, and formulas that fully set forth the claimed invention. C Requiring an essential feature in the original claims, where the feature is not described in the specification or the claims, and is not conventional in the art or known to one of ordinary skill in the art. D Amending a claim to add a limitation that is supported in the specification through implicit or inherent disclosure. E Amending a claim to correct an obvious error by the appropriate correction.

ANSWER: (C) is the most correct answer. See, "Guidelines for Examination of Patent Applications under 35 U.S.C. § 112, ¶ 1, 'Written Description' Requirement," MPEP 2163. "The claimed invention as a whole may not be adequately described if the claims require an essential or critical feature that is not described in the specification and is not conventional in the art or known to one of ordinary skill in the art." (A) is not the most correct answer. See, "Guidelines for Examination of Patent Applications under 35 U.S.C. § 112, ¶ 1, 'Written Description' Requirement," MPEP 2163. Describing an actual reduction to practice of the claimed invention is a means of showing possession of the invention. (B) is not the most correct answer. See, "Guidelines for Examination of Patent Applications under 35 U.S.C. § 112, ¶ 1, 'Written Description' Requirement," MPEP 2163. (D) is not the most correct answer. See, "Guidelines for Examination of Patent Applications under 35 U.S.C. § 112, ¶ 1, 'Written Description' Requirement," MPEP 2163, which states, "While there is no in haec verba requirement, newly added claim limitations must be supported by in the specification through express, implicit, or inherent disclosure." (E) is not the most correct answer. See, "Guidelines for Examination of Patent Applications under 35 U.S.C. § 112, ¶ 1, 'Written Description' Requirement," MPEP 2163, left column, second paragraph, which states, "An amendment to correct an obvious error does not constitute new matter where one skilled in the art would not only recognize the existence of the error in the specification, but also recognize the appropriate correction."

Which of the following statements is or are in accord with the patent laws, rules and procedures as related in the MPEP? (1) In a 35 USC 103 obviousness analysis, the proper question is whether the differences between the prior art and the claims would have been obvious to one of ordinary skill in the art. (2) In a 35 USC 103 obviousness analysis, an inventor's assertion the he has discovered the source or cause of an identified problem should never be considered. (3) A 35 USC 103 obviousness analysis requires consideration not just of what is literally recited in the claims, but also of any properties inherent in the claimed subject matter that are disclosed in the specification. A Statement 1 B Statement 2 C Statement 3 D Statements 1 and 2 E Statements 1 and 3

ANSWER: (C) is the most correct answer. The principle in Statement 3, that consideration of inherent properties is part of proper consideration of the invention as a whole, is recited in MPEP § 2141.02, under the heading "Disclosed Inherent Properties Are Part Of 'As A Whole' Inquiry," and in In re Antonie, 559 F.2d 618, 620, 195 USPQ 6, 8 (CCPA 1977). (A) is incorrect, because the proper question is whether the invention as a whole, not just the differences, would have been obvious. See MPEP § 2141.02, under the heading "The Claimed Invention As A Whole Must Be Considered," (citing Stratoflex, Inc. v. Aeroquip Corp., 713 F.2d 1530, 218 USPQ 871 (Fed. Cir. 1983). (B) is incorrect because an examiner should consider such assertions by an inventor as part of the "subject matter as a whole." See MPEP § 2141.02 (citing In re Sponnoble, 405 F.2d 578, 585, 160 USPQ 237, 243 (CCPA 1969)). (D) and (E) are incorrect because they include incorrect Statements 1 and/or 2.

In accordance with the patent laws, rules and procedures as related in the MPEP, which of the following facts are required for pre-AIA 35 USC 102(g) to form the basis for an ex parte rejection in a "transitional case": (1) The subject matter at issue has been actually reduced to practice by another before the applicant's invention. (2) There has been no abandonment, suppression or concealment. (3) A U.S. patent application for the subject matter at issue has been filed by another prior to the filing of the applicant's application. (4) A U.S. patent has been granted for the subject matter at issue prior to the filing of the applicant's application. A Fact (1) only B Fact (2) only C Facts (1) and (2) D Facts (1), (2) and (3) E Facts (1), (2), (3) and (4)

ANSWER: (C) is the most correct, as a 35 U.S.C. § 102(g) rejection requires actual reduction to practice by another, and lack of abandonment, suppression, or concealment. MPEP § 2138 states "35 U.S.C. 102(g) may form the basis for an ex parte rejection if: (1) the subject matter at issue has been actually reduced to practice by another before the applicant's invention; and (2) there has been no abandonment, suppression or concealment. See, e.g., Amgen, Inc. v. Chugai Pharmaceutical Co., 927 F.2d 1200, 1205, 18 USPQ2d 1016, 1020 (Fed. Cir. 1991); New Idea Farm Equipment Corp. v. Sperry Corp., 916 F.2d 1561, 1566, 16 USPQ2d 1424, 1428 (Fed. Cir. 1990); E.I. DuPont de Nemours & Co. v. Phillips Petroleum Co., 849 F.2d 1430, 1434, 7 USPQ2d 1129, 1132 (Fed. Cir. 1988); Kimberly-Clark v. Johnson & Johnson, 745 F.2d 1437, 1444-46, 223 USPQ 603, 606-08 (Fed. Cir. 1984)." (A) is incorrect, as actual reduction to practice is not sufficient to establish a 35 U.S.C. § 102(g) rejection where the subject matter has been abandoned, suppressed, or concealed. MPEP § 2138. (B) is incorrect, as abandonment, suppression, or concealment is not sufficient to establish a 35 U.S.C. § 102(g) rejection where the subject matter has been reduced to practice in that conception alone is not sufficient. See Kimberly-Clark v. Johnson & Johnson, 745 F.2d 1437, 1445, 223 USPQ 603, 607 (Fed. Cir. 1984). MPEP § 2138. (D) is incorrect because no prior patent application is required for a § 102(g) rejection. MPEP § 2138. Similarly, (E) is incorrect, because no prior patent application nor issued patent is required for a rejection under 35 U.S.C. § 102(g).

Where subject matter for which there is an enabling disclosure, but is not shown in the drawing or described in the detailed description preceding the claim(s), which of the following is not in accordance with the provisions of the MPEP? A In establishing a disclosure, applicant may rely not only on the description and drawing as filed but also on the original claims to show compliance with 35 USC 112(a). B Where subject matter not shown in the drawing or described in the description is claimed in the application as filed, and such original claim itself constitutes a clear disclosure of this subject matter, then the claim should be treated on its merits, and the applicant should be required to amend the drawing and description to show this subject matter. C If subject matter appearing in the original claim is not found in the drawing or detailed description, the claim should be rejected for noncompliance with 35 USC 112(a). D If the subject matter found in the claim is lacking in the drawing or detailed description, it is the drawing and description that are defective, not the claim. E The subject matter found in the original claims, but lacking in the drawing or detailed description, must be sufficiently specific and detailed to support an amendment of the drawing and detailed description.

ANSWER: (C). According to MPEP § 608.01(l), "[t]he claim should not be attacked either by objection or rejection because this subject matter is lacking in the drawing and description." As to (A), (B), (D), and (E), see MPEP § 608.01(l).

PhotoDetection Solutions, LLC is the owner of a portfolio of U.S. patents relating to various high-powered telephoto photography-related innovations for use by municipalities to better target traffic-related violations and increase enforcement revenues. The technologies have also shown promise for consumer use. In June 2019, BigFruit Co. approached PhotoDetection to discuss a joint research project to attempt to miniaturize the technology for deployment in smartphones, which would greatly increase photo zoom without the typically associated degradation in photo quality. PhotoDetection retained Arnold Anderson, of Flair, Blanchard & Anderson (the patent attorney who had been representing them for several years with respect to filing and prosecuting patent applications at the United States Patent and Trademark Office) to represent them in the negotiations. After PhotoDetection provided voluminous technical detail under a confidentiality agreement, no deal was struck with BigFruit Co. Subsequently, after having a falling out with is partners Ric and Tully, Arnold left the firm and became a partner in the much larger firm of McMahon & Hogan. Arnold retained PhotoDetection as a client, who followed him to the firm of McMahon & Hogan, where he continued to represent PhotoDectection as the company worked toward miniaturizing the technology for use in smaller and smaller applications, ultimately working toward realizing an embodiment suitable for use with smartphones. In August 2019, BigFruit Co. approached Robert Backlund, a senior partner with McMahon & Hogan. BigFruit wished to retain Backlund to prepare and file patent applications on miniaturized high-powered telephoto photography related innovations for use in smartphones. Can Backlund represent BigFruit? A Yes, provided PhotoDetection provides informed consent. B Yes, provided PhotoDetection provides informed consent, confirmed in writing. C Yes, provided both PhotoDetection and BigFruits provide informed consent, confirmed in writing. D No, Backlund may not represent BigFruit because Anderson has a conflict. E No, Backlund may not represent BigFruit unless Anderson ceases representing PhotoDetection.

ANSWER: (C). Pursuant to § 11.110(a), while practitioners are associated in a firm, none of them shall knowingly represent a client when any one of them practicing alone would be prohibited from doing so by either § 11.107 or § 11.109. The pertinent rule here is § 11.107, which relates to conflicts with current clients. A current conflict exists in this situation because both PhotoDetection and BigFruit are seeking patent claims on the same technology. However, pursuant to § 11.107 (b), a practitioner may still engage in representation if: (1) The practitioner reasonably believes that the practitioner will be able to provide competent and diligent representation to each affected client; (2) The representation is not prohibited by law; (3) The representation does not involve the assertion of a claim by one client against another client represented by the practitioner in the same litigation or other proceeding before a tribunal; and (4) Each affected client gives informed consent, confirmed in writing. In this situation, given that two different patent attorneys are involved, it should be reasonable to believe that competent and diligent representation can be provided to both. The representation is not prohibited by law, and it does not involve the assertion of a claim by one against the other. Therefore, if both give informed consent in writing, this will not be a problem. (E) is incorrect because ceasing to represent PhotoDetection as a client is unnecessary. Furthermore, (E) merely makes PhotoDetection a former client, which creates obligations under § 11.109 to a former client and does not in and of itself solve any ethical issue.

Gonzo Gargoyle is the inventor of a process for generating customized stock quotes and alerting a remote subscriber that the quotes can be accessed from their computer or mobile device. The stock quote alerts are generated by filtering received stock quotes, building stock quote alerts and formatting the alerts into data blocks based upon subscriber preference information. If the subscriber's computer is offline, the data blocks are transmitted over a wireless communication channel to the subscriber's wireless device which, when connected to the computer, causes the computer to auto-launch a stock viewer application to display the alert and provide access to more detailed information about the stock quote. Gargoyle files a provisional patent application adequately describing the invention on March 15, 2016. Immediately thereafter, Gargoyle hires several computer programmers, all of who sign confidentiality agreements and agree to assign any and all of their intellectual property creations over to the newly formed Gecko Wall Street Traders, LLC, which is wholly owned by Gargoyle. In addition to creating a working system, Gargoyle's team integrates a biometric security feature, which was not disclosed in the provisional patent application. On March 14, 2017, a nonprovisional patent application is filed that discloses both the original embodiment from the provisional patent application and newly created biometric feature set, but which claims only the embodiment originally disclosed in the provisional application. Gargoyle files a petition for prioritized examination and pays the fee. In the First Office Action, dated July 5, 2017, the Examiner rejects all of the claims in the application as being patent ineligible pursuant to 35 U.S.C. 101. If properly argued by Gargoyle or his representatives, can this rejection be successfully overcome? A No. Claims to a process for generating customized stock quotes as described would not qualify as being patent eligible because they do not fall within one of the four statutory categories of invention. B Yes. Claims to a process for generating customized stock quotes as described are not directed to an abstract idea and, therefore, would be patent eligible as an improvement. C No. Claims to a process for generating customized stock quotes as described cover a mathematical relationship and algorithm that have been found by the courts to be patent -ineligible abstract ideas. D Yes. Claims to a process for generating customized stock quotes as described are directed to one of the four statutory categories of invention. Although the claims would properly be considered to be directed to an abstract idea, the claims as a whole amount to significantly more than the abstract idea. E No. Claims to a process for generating customized stock quotes as described are directed to one of the four statutory categories of invention. Although the claims would properly be considered to be directed to an abstract idea, the claims as a whole do not amount to significantly more than the abstract idea.

ANSWER: (D) This is a difficult question, made more difficult by the fact that a clearly patent-eligible biometric feature set is added to the nonprovisional patent application, but not claimed. Some who answered this question no doubt thought that if that biometric feature set were claimed, it would be patent eligible. That is no doubt correct. But it is also correct to say that this would be patent eligible even without the addition of that biometric feature set in the claims. The process for generating customized stock quotes, as described, is indeed a process that falls within the four statutory categories of invention allowed under 35 U.S.C. 101. The next question in the Alice/Mayo framework asks whether the claim is directed to a judicial exception, which in this case would be an abstract idea. This would be directed to an abstract idea because the comparing and organizing of information for transmission as described could be performed mentally and, as such, the steps are similar to concepts and ideas that courts have identified as abstract ideas. The final question in the Alice/Mayo framework asks whether the claims add significantly more. Although some of the elements when viewed individually do not amount to significantly more (e.g., storing subscriber preferences), when the additional limitations are viewed as a combination, the claim as a whole amounts to significantly more than simply organizing and comparing information (the abstract idea itself). The invention addresses the Internet-centric challenge of alerting a subscriber with time-sensitive information when the subscriber's computer is offline. This is accomplished by transmitting the alert over a wireless communication channel to activate the stock-viewer application, which causes the alert to display and enables the connection of the remote subscriber computer to the data source over the Internet when the remote subscriber computer comes online. These are meaningful limitations that add more than generally linking the use of the abstract idea (the general concept of organizing and comparing data) to the Internet because they solve an Internet-centric problem with a claimed solution that is necessarily rooted in computer technology, similar to the additional elements in DDR Holdings. These limitations, when taken as a combination, provide unconventional steps that confine the abstract idea to a particular useful application, such that the claim as a whole amounts to significantly more than the abstract idea itself. Therefore, the claim recites patent-eligible subject matter.

Which of the following timely actions should you take to accord maximum patent protection at minimum government fees for your client whose invention is described in a provisional patent application that was filed 6 months ago with no claim? A File a request to convert the provisional application to a nonprovisional application, accompanied by a proper executed declaration, an amendment including at least one claim as prescribed by paragraph 2 of 35 USC 112 and the proper fee set forth in 37 CFR 1.17(i). B File a request to convert the provisional application to a nonprovisional application, accompanied by a proper executed declaration, an amendment including at least one claim as prescribed by paragraph 2 of 35 USC 112, the proper fee set forth in 37 1.17(i), and the basic filing fee for the nonprovisional application. C File a request to convert the provisional application to a nonprovisional application, accompanied by a proper executed declaration, an amendment including at least one claim as prescribed by paragraph 2 of 35 USC 112, the proper fee set forth in 37 CFR 1.17(i), the basic filing fee for the nonprovisional application, and the surcharge required by 37 CFR 1.16(e). D File a nonprovisional application including at least one claim accompanied by a proper executed declaration, and the basic filing fee. The application contains an ADS with a specific reference to the provisional application in compliance with 37 CFR 1.78(a)(5). E File a nonprovisional application including at least one claim accompanied by a proper executed declaration but without the basic filing fee. The application contains a specific reference to the provisional application.

ANSWER: (D) is correct. (A), (B) and (C) are wrong because MPEP § 601.01(c) states, "Claiming priority is less expensive [than conversion] and will result in a longer patent term." Conversion requires payment of the conversion fee. (D) is correct because MPEP § 601.01(c) states, "In addition, if the provisional application was not filed with an executed oath or declaration and the filing fee for a non-provisional application, the surcharge set forth in 37 C.F.R. § 1.16(e) is required. (E) is wrong because the action taken results in an additional late filing fee surcharge being required.

On Monday, May 13, 2022, John's secretary deposited in a "Priority Express Mail" drop box prior to the last scheduled pick-up for that day, an envelope properly addressed to the USPTO for delivery to the USPTO by the "Express Mail Post Office to Addressee" service. The envelope was received by the USPTO on Wednesday, May 15, 2022, containing a reply to an Office action which set a shortened statutory period ("SSP") for reply ending on Tuesday, May 14, 2022. The reply was marked by the Office as being received on May 15, 2022. The number of the "Express Mail" mailing label had not been placed on the response papers, and upon receipt of the "Express Mail" mailing label John learned that the "date in" was not clearly marked. John promptly filed a petition requesting the filing date to be the date of deposit. The petition included a showing that the date of deposit accompanied by evidence of USPS corroboration of the deposit. Accordingly, A The reply will be regarded as timely filed in the USPTO on May 15, 2022. B The reply will be regarded as timely filed in the USPTO on May 14, 2022. C The reply will be regarded as timely filed in the USPTO on May 13, 2022. D The reply will be regarded as timely filed in the USPTO if a petition with proper fee for a one month extension of time is filed in the USPTO on or before June 14, 2022. E The reply will be regarded as timely filed in the USPTO if the number of the "Priority Express Mail" mailing label is placed on each page of a copy of the original response and hand carried to the USPTO on May 15, 2022, rather than being sent by "Priority Express Mail."

ANSWER: (D) is correct. 37 C.F.R. § 1.136(a). 37 C.F.R. § 1.136(a) states, "[A]pplicant may extend the time period for reply up to the earlier of the expiration of any maximum period set by statute or five months after the time period set for reply, if a petition for an extension of time and the fee set in § 1.17(a) are filed..." (A) is wrong because the response was not timely filed since it was received by the USPTO after the SSP expired. (B) and (C) are wrong. The reply was not filed on May 14, 2022, because the conditions of 37 C.F.R. § 1.10(b) were not satisfied. For example, the number of the "Priority Express Mail" mailing label must have been placed on each page of the response prior to the original mailing by "Priority Express Mail." The petition should not be expected to be granted inasmuch as the papers did not include the number of the "Priority Express Mail" mailing label on them. See § 1.10(c)(2), (d)(2), and (e)(2). (E) is wrong because 37 C.F.R. § 1.10(b) requires that "the number of the 'Priority Express Mail' mailing label must have been placed on each page of the response prior to the original mailing by 'Priority Express Mail.'" [emphasis added]

The specification of an application does not disclose the utility of the claimed composition. In fact, the claimed invention is useful for shrinking a specific class of tumors. In a first Office action, the primary examiner has properly determined that the claims lack utility, and has rejected all of the composition claims under the first paragraph of 35 USC 112 as lacking utility. Which of the following responses is in accord with the USPTO rules and the procedures of the MPEP for persuading the examiner that the rejection is improper? A Explain that the rejection is statutorily improper because the first paragraph of section 112 is concerned with enablement and written description issues and therefore does not support a rejection for lack of utility. B Point out that the rejection is based on an erroneous finding by the examiner because the specification, in fact, clearly discloses that the composition in question possesses "useful biological" properties. C Show that the rejection is improper by filing probative evidence that the claimed composition has unambiguously proven to be useful for shrinking a specific class of tumors. D File declarations by persons with ordinary skill in the art stating that they would immediately appreciate that the claimed composition is useful for shrinking a specific class of tumors due to the fact that similar compositions having the same characteristics as applicant's claimed composition were known to be effective for this purpose. E Argue that the rejection is improper because the examiner has failed to present evidence in support of his position that the claimed composition has no utility.

ANSWER: (D) is most correct answer. As explained at MPEP § 2107.02, II, B, under the heading "No Statement of Utility for the Claimed Invention in the Specification Does Not Per Se Negate Utility," the fact that a specification does not contain a statement of utility for the claimed invention does not per se negate utility. This is because a claimed invention may have a well-established utility, and an invention has a well -established utility if (i) a person of ordinary skill in the art would immediately appreciate why the invention is useful based on the characteristics of the invention and (ii) the utility is specific, substantial, and credible. In this case, the declarations specify a specific substantial and credible utility and explain why the declarants (i.e., persons of ordinary skill in the art) would immediately appreciate that the applicant's claimed composition would possess this utility. (A) is incorrect. A lack of utility deficiency under 35 U.S.C. § 101 also creates a lack of utility deficiency under the first paragraph of 35 U.S.C. § 112 as fully explained at MPEP § 2107.01, under the heading "IV. Relationship Between 35 U.S.C. 112, First Paragraph, And 35 U.S.C. 101." (B) is not the most correct answer. 35 U.S.C. § 101 (and the first paragraph of 35 U.S.C. § 112) requires that the utility be specific. Therefore, the disclosure of a general utility such as "useful biological" properties does not satisfy this requirement as fully explained at MPEP § 2107.01, under the heading "I. Specific And Substantial Requirements." Response (C) also would not be persuasive since the rejection is based on the fact that the applicant's specification fails to identify any .specific and substantial utility for the claimed composition or fails to disclose enough information about the invention to make its usefulness immediately apparent to those familiar with the technological field of the invention. This is explained at MPEP § 2107.01. The fact that the claimed composition has unambiguously proven to be useful for curing a form of cancer previously thought to be incurable does not negate these specification deficiencies. That is, notwithstanding this unambiguous proof, the fact remains that the applicant's specification fails to identify any specific and substantial utility for the composition. Moreover, it is clear that the specification would not make this specific usefulness immediately apparent to those familiar with the technological field of the composition since the cancer was previously thought to be incurable. Finally, response (E) also would not be persuasive. Under current USPTO policy and procedure, the examiner is not required to present evidence in support of a rejection based on lack of utility where, as here, the specification does not identify a specific, substantial and credible utility and does not appear to provide sufficient information such that a well-established utility would be apparent to a person with ordinary skill in the art. See MPEP § 2107, under the heading "II. Examination Guidelines For The Utility Requirement."

Which of the following is not in accordance with the provisions of the USPTO rules and the procedures set forth in the MPEP? A Where joint inventors are named, the examiner should not inquire of the patent applicant concerning the inventors and the invention dates for the subject matter of the various claims until it becomes necessary to do so in order to properly examine the application. B Under 35 USC 119(a), the foreign priority benefit may be claimed to any foreign application that names a U.S. inventor as long as the U.S. named inventor was the inventor of the foreign application invention and 35 USC 119(a)-(d) requirements are met. C Where two or more foreign applications are combined in a single U.S. application, to take advantage of the changes to 35 USC 103 or 35 USC 116, the U.S. application may claim benefit under 35 USC 119(a) to each of the foreign applications provided all the requirements of 35 USC 119(a)-(d) are met. D One of the conditions for benefit under 35 USC 119(a) is that the foreign application must be for the same or a nonobvious improvement of the invention described in the United States application. E If a foreign application for which priority is being claimed under 35 USC 119 is filed in a country which does not afford similar privileges in the case of applications filed in the United States or to citizens of the United States and the foreign country is not a WTO member country, any claim for the foreign priority thereto by a U.S. application will not be effective.

ANSWER: (D) is the best answer as the inventions must be the same in the foreign and U.S. applications. 35 U.S.C. § 119(a). As to (A) through (C), see MPEP § 213. As to (E), see 35 U.S.C. § 119, which provides that the previously filed application must have been filed in a country that affords similar privileges in the case of applications filed in the United States or to citizens of the United States or in a WTO member country.

Which of the following is not in accordance with the provisions of the MPEP? A Where joint inventors are named, the examiner should not inquire of the patent applicant concerning the inventors and the invention dates for the subject matter of the various claims until it becomes necessary to do so in order to properly examine the application. B Under 35 USC 119(a), the foreign priority benefit may be claimed to any foreign application that names a U.S. inventor as long as the U.S. named inventor was the inventor of the foreign application invention and 35 USC 119(a)-(d) requirements are met. C Where two or more foreign applications are combined in a single U.S. application, to take advantage of the changes to 35 USC 103 or 35 USC 116, the U.S. application may claim benefit under 35 USC 119(a) to each of the foreign applications provided all the requirements of 35 USC 119(a)-(d) are met. D One of the conditions for benefit under 35 USC 119(a) is that the foreign application must be for the same or a nonobvious improvement of the invention described in the United States application. E If a foreign application for which priority is being claimed under 35 USC 119 is filed in a country which does not afford similar privileges in the case of applications filed in the United States or to citizens of the United States and the foreign country is not a WTO member country, any claim for the foreign priority thereto by a U.S. application will not be effective.

ANSWER: (D) is the best answer since the inventions do not have to be identical in the respective foreign and U.S. applications. Rather, the US claim must only find support in the foreign filed specification. As to (A), see MPEP § 602.09. As to (B) and (C), see MPEP § 213. As to (E), see 35 U.S.C. § 119 which provides that the previously filed application must have been filed in a country that affords similar privileges in the case of applications filed in the United States or to citizens of the United States or in a WTO member country.

Inventor Tip, a scientist in a pencil research laboratory, theorized that, based on the abrasive properties of moon dust, a highly efficient erasure can be made by adding a trace amount of moon dust to a normal pencil erasure formulation. Point, in the Sales department, determined that this would be perfect for a high end product. A U.S. patent application has been filed claiming a pencil erasure formulation with a trace amount of moon dust. An example of how to make the formulation with specified percentages of moon dust is presented therein. Thereafter, Tip learns about the duty to disclose information and he recalls signing a declaration under 37 CFR 1.63 stating that he had reviewed and understood the contents of the specification including the claims. Tip becomes concerned that the use of moon dust was only a theory and that to obtain patent would mislead the public to conclude that moon dust was actually used and found to be effective. The application has been allowed, but the issue fee has not yet been paid. Which of the following is most in accord with patent laws, rules and procedures as related in the MPEP? A Point is under a duty to disclose material information to the USPTO. B Tip is under a duty to disclose his concern regarding the moon rock information to the USPTO. C Both Point and Tip are under a duty to disclose material information to the UPSTO. D There is no duty to disclose information regarding how the moon rock formulation was developed to the USPTO. E Inasmuch as the application is allowed, an appropriate Request for Continued Examination pursuant to 37 CFR 1.114 needs to be filed accompanied by an information disclosure regarding the possibility of rejections under 35 USC 101, and 112(a). .

ANSWER: (D) is the most correct answer. 37 CFR § 1.56; MPEP § 2001.05. 37 CFR § 1.56(a) sets forth a duty to disclose information that is material to patentability. MPEP § 2001.05 states that "information is not material unless is comes within the definition of 37 CFR 1.56(b)(1) or (b)(2). If information is not material, there is no duty to disclose the information to the Office." The information that moon dust was never actually used is not material as defined under 37 CFR § 1.56(b)(1) or (2) which state that information is material if "(b)(1) It establishes, by itself or in combination with other information, a prima facie case of unpatentability of a claim; or (2) It refutes, or is inconsistent with, a position the applicant takes in: (i) Opposing an argument of unpatentability relied on by the Office or, (ii) Asserting an argument of patentability." That the use of the moon dust as part of an erasure formulation was only theorized and not actually used is acceptable as is an example for making it. MPEP § 608.01(p), II, under the heading "Simulated or Predicted Test Results Or Prophetic Examples," states that "[s]imulated or predicted test results and prophetical examples (paper examples) are permitted in patent applications. ... Paper examples describe the manner and process of making an embodiment of the invention which has not actually been conducted." Care, however, must be taken not to state that an experiment was actually run or conducted when it was not and that "[n]o results should be presented as actual results unless they have actually been achieved." MPEP § 2004, item 8. (A) is incorrect. 37 CFR § 1.56(a) requires that individuals associated with the filing and prosecution of a patent application have a duty to disclose information to the Office. 37 CFR § 1.56(c) defines which individuals are associated with the filing and prosecution of a patent application and that "(c) Individuals associated with the filing or prosecution of a patent application within the meaning of this section are: (1) Each inventor named in the application; (2) Each attorney or agent who prepares or prosecutes the application; and (3) Every other person who is substantively involved in the preparation or prosecution of the application and who is associated with the inventor, with the assignee or with anyone to whom there is an obligation to assign the application." Point is part of the Sales department and no facts were presented that substantively involved him in the preparation or prosecution of the application. Additionally, as noted in the explanation relating answer to (D), the information given to Point by Tip was not material information. (B) is incorrect. While Tip would be an individual identified under 37 CFR § 1.56(c), there is no material information to be disclosed as noted in the explanation to (D). (C) is incorrect. As noted in the explanation relating to (D), the information is not material. Additionally, as noted in the explanation to answer (A), Point is not an individual defined by 37 CFR 1.56(c) as owing a duty. (E) is incorrect. As there is no requirement that it be explicitly stated that an invention has or has not been actually conducted, as noted in the explanation of (D), the prosecution need not be continued for the purpose of supplying an information disclosure statement regarding the development of the moon rock erasure formulation.

When, in accordance with the patent laws, rules and procedures as related in the MPEP, is a supplemental oath or declaration treated as an amendment under 37 CFR 1.312? A When filed in a nonprovisional application after the Notice of Allowance has been mailed. B When filed in a reissue application at any point during the prosecution. C When filed in a nonprovisional application after the payment of the Issue Fee. D When filed in a reissue application after the Notice of Allowance has been mailed. E (A) and (D).

ANSWER: (D) is the most correct answer. MPEP § 714.16, third paragraph, states "a supplemental reissue oath or declaration is treated as an amendment under 37 CFR 1.312 because the correction of the patent which it provides is an amendment of the patent, even though no amendment is physically entered into the specification or claim(s)." (A) is incorrect because a supplemental oath or declaration is not treated as an amendment under 37 CFR 1.312 except when submitted in a reissue. See MPEP § 603.01. (B) is incorrect because a supplemental oath or declaration in a reissue will be treated as an amendment under 37 CFR § 1.312 only if filed after allowance. (C) is incorrect because amendments filed after the date the issue fee has been paid are no longer permitted under 37 CFR § 1.312. (E) is wrong because (A) is incorrect.

A U.S. patent was granted on May 8, 2015. The sole independent claim in the patent is directed to a combination of elements ABCD. A registered practitioner filed a reissue application on April 11, 2017 to narrow sole independent claim. In the reissue application, the independent claim is amended to a combination to elements ABCDE. The reissue application is accompanied by a transmittal letter stating that the application was filed to narrow a claim, that all inventors could not be located to sign the reissue oath or declaration at that time, and that a declaration would be submitted in due course. No other amendments to the claims were filed on April 11, 2017. On May 8, 2017, a declaration signed by all inventors is filed declaring that they had claimed less than they had a right to claim, and that the error arose without deceptive intent. The inventors also filed on May 8, 2017 a preliminary amendment deleting element A from the sole independent claim leaving elements BCDE. The amendment and declaration are filed using the provisions of 37 CFR 1.10. The practitioner included an authorization to charge the practitioner's deposit account for any necessary fees. Which of the following actions by the primary the examiner in the first Office action is in accordance with the patent laws, rules and procedures as related in the MPEP? A Reject all the claims based upon a broadening reissue outside the two year statutory period authorized by 35 USC 251 since applicant did not file a broadened reissue claim at the time of filing. B Reject all the claims based upon a broadening reissue outside the two year statutory period authorized by 35 USC 251 since applicant did not file a claim to a broadened reissue claim within the two year period set by 35 USC 251. C Reject all the claims based upon a broadening reissue outside the two year statutory period authorized by 35 USC 251 since applicant's indication in the transmittal letter indicated that the filing of the reissue application was a narrowing reissue and that the broadening amendment was not permissible even if filed within the two-years from the grant of the original patent. D Determine that the application is a proper broadening reissue and perform an examination and issue an Office action in due course. E Determine that the application is a proper broadening reissue and reject the claims under the recapture doctrine since the claims are broader than the issued claims.

ANSWER: (D) is the most correct answer. MPEP §§ 1403 and 1412.03, under the heading "When A Broadened Claim Can Be Presented." A broadening reissue claim must be filed within the two years from the grant of the original patent. (D) is the most correct and the examiner should examine the case as any other application and address appropriate issues concerning reissue examination. See Switzer v. Sockman, 333 F.2d 935, 142 USPQ 226 (CCPA 1964) (a similar rule in interferences). Since applicant filed the amendment by Express Mail, the amendment is treated as being filed with the USPTO on the date of deposit with the US Postal Service. Therefore, (A), (B) and (C) are incorrect answers. A reissue application can be granted a filing date without an oath or declaration, or without the filing fee being present. See 37 CFR § 1.53(f). Applicant will be given a period of time to provide the missing parts and to pay the surcharge under 37 CFR § 1.16(e). See MPEP § 1410.01. Choice (E) is not correct since the mere deletion of an element of a claim does not automatically raise a ground of rejection based on the recapture doctrine. See MPEP § 1412.02

Paprika is a known product. A patent application discloses a composition which is made by subjecting paprika to processing steps X, Y and Z. The composition is disclosed to be useful in treating cancer. The application was filed June 1, 2016. A reference published May 1, 2015 discloses a food product made by subjecting paprika to processing steps X, Y and Z. The reference does not disclose that the resulting composition has any properties that would make it useful for treating cancer. In accordance with USPTO rules and procedures set forth in the MPEP, which of the following claims is not anticipated by the reference? A A composition made by the process of subjecting paprika to processing steps X, Y and Z, wherein the composition is effective for treating cancer. B A composition for treating cancer, made by the process of subjecting paprika to processing steps X, Y and Z. C A method of making a cancer-treating composition, comprising subjecting paprika to processing steps X, Y and Z. D A method of treating cancer, comprising administering an effective amount of a composition made by subjecting paprika to processing steps X, Y and Z. E All of the above.

ANSWER: (D) is the most correct answer. See 35 U.S.C. § 102(b); MPEP § 2131. Citing Verdegaal Bros. v. Union Oil Co. of California, 814 F.2d 628, 631, 2 USPQ2d 1051, 1053 (Fed. Cir. 1987), MPEP § 2131, under the heading, "To Anticipate A Claim, The Reference Must Teach Every Element Of The Claim" states, "A claim is anticipated only if each and every element as set forth in the claim is found, either expressly or inherently described, in a single prior art reference.". The claim is directed to a method of use that is not disclosed by the reference. Answer (A) is incorrect. 35 U.S.C. § 102(b); MPEP §§ 2112, 2112.01. The claimed composition is the same as that disclosed in the prior art, because it is made from the same starting material subjected to the same processing steps. The recitation of "the composition is effective for treating cancer," is only a statement of the inherent properties of the composition. Where the claimed and prior art products are identical in structure or composition, or are produced by identical processes, a prima facie case of anticipation has been established. In re Best, 195 USPQ 430, 433 (CCPA 1977). The burden is shifted to applicant to show that the prior art product does not necessarily possess the characteristics of the claimed product. The reference is prior art under 35 U.S.C. § 102(b), and therefore the claim is anticipated. Answer (B) is incorrect. 35 U.S.C. § 102(b); MPEP §§ 2112, 2112.01, and 2112.02. The claimed composition is the same as that disclosed in the prior art, because it is made from the same starting material subjected to the same processing steps. The recitation of a composition "for treating cancer" reflects only a preamble statement of an intended use of the claimed composition, which does not limit the scope of the claim. Answer (C) is incorrect. See 35 U.S.C. § 102(b); MPEP §§ 2112, 2112.01. The claimed method is the same as that disclosed in the prior art, because it subjects the same starting material to the same manipulative steps. The recitation of making "a cancer-treating composition" reflects only a preamble's statement of an intended use of the claimed composition, which does not further limit the claimed method. Answer (E) is incorrect, because (A), (B), and (C) are incorrect.

In accordance with USPTO rules and procedures set forth in the MPEP, which of the following is not a proper basis on which the PTAB may remand a case to the examiner? A Remand for a fuller description of the stated rejection. B Remand for a clearer explanation of the pertinence of the references. C Remand for a selection by the primary examiner of a preferred or best ground of rejection when multiple rejections of a cumulative nature have been made by the examiner. D Remand to the primary examiner with instructions to consider an affidavit not entered by the examiner which was filed after the final rejection but before the appeal. E Remand to the primary examiner to prepare a supplemental examiner's answer in response to a reply brief.

ANSWER: (D) is the most correct answer. See MPEP § 1211.02. (D) is not a proper basis for remand because the Board has no authority to require the examiner to consider an affidavit which has not been entered after final rejection and which was filed while the application was pending before the examiner. Pursuant to 37 CFR § Part 41, "[a] ffidavits...submitted after the case has been appealed will not be admitted without a showing of good and sufficient reasons why they were not earlier presented." The facts are silent regarding whether such a showing was made. However, as discussed in MPEP § 715.09, review of an examiner's refusal to enter and consider an affidavit as untimely if by petition and not by appeal to the Board. In re Deters, 515 F.2d 1152, 185 USPQ 644 (CCPA 1975); Ex parte Hale, 49 USPQ 209 (Bd. App. 1941)." Thus, remand by the Board cannot be expected. Support for each of answers (A), (B), (C) and (E) is specifically provided for in MPEP § 1211.

Inventor files an application for a non-theoretical metal alloy. The application as originally filed contains the following Claim 1: Claim 1. A metal alloy comprising at least 20% by volume of iron; at least 10% by volume of gallium, and at least 10% by volume of copper. In accordance with the USPTO rules and the procedures set forth in the MPEP, which of the following claims would be properly held indefinite under 35 USC 112(b)? A Claim 2: The alloy of claim 1 containing 66% by volume of gallium and 14% by volume of copper. B Claim 2: The alloy of claim 1 containing at least 21% by volume of iron, 11% by volume of gallium, and 10.01% by volume of copper. C Claim 2: The alloy of claim 1 containing 20% by volume of iron, 10% by volume of gallium, and 10% by volume of copper. D Claim 2: The alloy of claim 1 containing 54% by volume of copper and 27% by volume of gallium. E Claim 2: The alloy of claim 1 containing at least 1% by volume of silver.

ANSWER: (D) is the most correct answer. See MPEP § 2173.05(c), under the heading "Open-Ended Numerical Ranges." Paraphrasing the explanation therein, when an independent claim recites a composition comprising "at least 20% iron" and a dependent claim sets forth specific amounts of non-iron ingredients which add up to100%, apparently to the exclusion of iron, an ambiguity is created with regard to the "at least" limitation unless the percentages of the non-iron ingredients are based on the weight of the non-iron ingredients. On the other hand, a composition claimed to have a theoretical content greater than 100% (i.e., 20-80% of iron, 20-80% of gallium, and 1-25% of copper) is not necessarily indefinite simply because the claims may be read in theory to include compositions that are impossible in fact to formulate. Here, because the invention is a non-theoretical alloy, the sum of the claimed constituents cannot exceed 100% unless the percentage is based on weight. In (D), the sum of elements (B) and (C) is 81% by volume, leaving only 19% for iron. Claim 1, however, requires "at least 20% iron," rendering Claim 2 ambiguous as to the percentage of element A. (A) is incorrect. The sum of gallium and copper components is 80%, leaving a possible 20% of the composition for element iron. Claim 1 requires "at least 20% iron," which includes 20% iron. Therefore, the sum of iron, gallium and copper components in Claim 2 is 100%. (B) is incorrect. "At least 20% iron" includes 21% iron, "at least 10% gallium includes 11% gallium, and "at least 10% copper includes 10.01% copper. (C) is incorrect. "At least 20% iron" includes 20% iron, "at least 10% gallium includes 10% gallium, and "at least 10% copper" includes 10% copper. (E) is incorrect because Claim 1 uses the open transition phrase "comprising," which permits additional elements to be added to the composition. Nothing in the problem indicates that an additional component, silver, cannot be added to the composition.

A patent application filed in the USPTO claims a nylon rope coated with element E for the purpose of preventing breakage of the rope. In the first Office action, the examiner rejects the claim as obvious over P in view of a trade journal publication, T. P teaches a nylon rope coated with resin for the purpose of making the rope waterproof. T teaches a nylon tent fabric coated with element E for the purpose of making the tent waterproof, and suggests the use of element E for making other nylon products waterproof. Following proper USPTO practices and procedures, the combination of P and T: A cannot support a prima facie case of obviousness because T lacks a suggestion to combine with P for the purpose of preventing breakage in nylon rope. B cannot support a prima facie case of obviousness because P lacks a suggestion to combine with T for the purpose of preventing breakage in nylon rope. C cannot support a prima facie case of obviousness because T only contains a suggestion to combine with P for the purpose of waterproofing nylon rope. D can support a prima facie case of obviousness, even though T only contains a suggestion to combine with P for the purpose of waterproofing nylon rope. E can support a prima facie case of obviousness because the applicant is always under an obligation to submit evidence of non-obviousness regardless of whether the examiner fully establishes a prima facie case of obviousness.

ANSWER: (D). "It is not necessary in order to establish a prima facie case of obviousness...that there be a suggestion or expectation from the prior art that the claimed [invention] will have the same or a similar utility as one newly discovered by the applicant." In re Dillon, 919 F.2d 688, 692, 16 USPQ2d 1897, 1900 (Fed. Cir. 1990) (emphasis in original). Thus, "[i]t is not necessary that the prior art suggest the combination to achieve the same advantage or result discovered by applicant." MPEP § 2144 ("Rationale Different from Applicant's is Permissible"). Here, T suggests the combination with P to achieve a different advantage or result, i.e., waterproofing, from that discovered by applicant, i.e., reducing breakage. Answers (A) - (C) are incorrect because the suggestion to combine does not need to be for the same purpose as applicant discloses in the application. Dillon, 919 F.2d at 692, 16 USPQ2d at 1900; MPEP § 2144 ("Rationale Different from Applicant's is Permissible"). Answer (E) is incorrect because an applicant is under no obligation to submit evidence of non-obviousness unless the examiner meets his or her initial burden to fully establish a prima facie case of obviousness. MPEP § 2142.

Alex McCoy is a patent practitioner with the firm of Snyder, Allen & Gruden. On January 16, 2019, he reviewed his firm's calendar system and saw that January 16, 2019 had been calendared as the deadline for the patent application to be filed on behalf of his client, Native American Apparel, Inc. He reviewed documents on his computer server relating to the client's patent application that had been previously prepared by Josh Sanchez, another registered practitioner affiliated with the firm. Following this review, and without consulting with the client, McCoy concluded that the provisional patent application previously filed by his client was directed to a prior version of the client's invention. Seeing no basis or value for the nonprovisional patent application to claim priority to the client's prior provisional application, on January 16, 2019, he requested a firm patent agent to prepare the nonprovisional patent application and PCT application, remove any claims to priority from the text of the application, and prepare the applications for filing. The nonprovisional application and PCT applications were ultimately filed on January 18, 2019. Ultimately, the client became unhappy with the representation received, and Native American Apparel moved their portfolio of cases to Jason Dallas of the firm Whitten, Aikman & Irving. When reviewing the file, Dallas noticed the priority gap and advised the client that, for reasons he could not explain, the previous attorney did not file the nonprovisional patent application and PCT application timely, and benefit to the originally filed provisional application was lost. Native American Apparel explained they had told their first patent attorney, Sanchez, about how important the priority date was to keep and never received any communications from McCoy other than to say the filing had been accomplished as planned. Which, if any, of the following rules of Professional Rules of Conduct did McCoy violate? I. Failure to promptly inform the client of a decision or circumstance with respect to which the client's informed consent was required. II. Failure to reasonably consult with a client about means by which the client's objectives are to be accomplished. III. Failure to provide competent representation to the client. A I B II C III D I and II E II and III

ANSWER: (D). 37 C.F.R. 11.104(a) requires that practitioners promptly inform the client of a decision or circumstance with respect to which the client's informed consent is required, and reasonably consult with a client about means by which the client's objectives are to be accomplished. Making a decision about a priority date without consulting with the client would be a violation of this section. 37 C.F.R. 11.101 requires a practitioner to provide competent representation to a client. Competent representation is defined as requiring the legal, scientific, and technical knowledge, skill, thoroughness and preparation reasonably necessary for the representation. Making a mistake, even one like this, does not rise to the level of demonstrating a lack of competence. To demonstrate a lack of competence one would have to demonstrate incompetence, not merely that a mistake was made. It is rather extraordinary for a practitioner to be found to have failed to provide competent representation within the meaning of the Rules of Professional Conduct.

Michael Zhang is the inventor of a method of controlling a melting process in an electric arc furnace for melting a metallic material. Zhang's invention makes it possible to minimize desired process properties such as the melting time or the total power consumption of the melting process. The method includes collecting measured data of at least one process variable, determining the current state of the process, performing an optimization of the melting process, determining a process input based on the result of the optimization, and controlling the melting process by means of the process input with a dedicated, and rather sophisticated control system. Zhang, a prolific inventor and professional independent inventor who has had great success licensing his own inventions, has always used the services of patent attorney Kathy Sanford. Unfortunately, Zhang fell on hard times after Sanford prepared and filed a nonprovisional patent application for Zhang on the aforementioned invention, Zhang lost a significant stream of revenue after having one of his key patents fall in an inter partes review at the Patent Trial and Appeal Board. Faced with financial trouble, and kids finally out of the house, Zhang's marriage fell apart and he was just served with divorce papers, and Zhang has publicly discussed filing for bankruptcy should he lose at the PTAB. Sanford had already been overlooking $50,000 in overdue legal fees given the longstanding nature of her attorney-client relationship with Zhang, and the fact that he had always paid his bills. Unfortunately, it seems that Zhang is in a very bad place financially and things are likely to get worse before they get better. If Sanford were to decide to withdraw from representing Zhang, which of the following best describes her legal obligations? She must: I. Given reasonable notice to the client, prior to the expiration of any reply period the client is facing, that she intends to withdraw from employment. II. Certify to the Office that at least 30 days remain between the filing of the request for withdrawal and the later of the expiration date of a time period for reply or the expiration date of the statutory period. III. Deliver to the client or a duly authorized representative of the client all papers and property (including funds) to which the client is entitled. IV. Notify the client of any replies that may be due and the timeframe within which the client must respond. A I and III B II and III C II, III and IV D I, III and IV E III and IV

ANSWER: (D). Since the current revised withdrawal procedures went into effect in 2008, the Office no longer requires there to be at least 30 days between approval of the request to withdraw as a practitioner of record in a patent application and the later of the expiration date of a time period for reply or the expiration date of the period which can be obtained by a petition and fee for extension of time under 37 CFR 1.136 (a). Instead, pursuant to 37 CFR 10.40, the Office will require the practitioner(s) to certify that he, she or they have: (1) given reasonable notice to the client, prior to the expiration of the reply period, that the practitioner(s) intends to withdraw from employment; (2) delivered to the client or a duly authorized representative of the client all papers and property (including funds) to which the client is entitled; and (3) notified the client of any replies that may be due and the time frame within which the client must respond.

Linda Horowitz is a solo practitioner. She entered into a written engagement agreement with Thomas Holt and Dr. Ronald Knight. On May 15, 2018, Horowitz filed a provisional patent application on behalf of Holt and Knight as named inventors. Internet World LLC, Holt's company, was listed as the assignee. On October 1, 2018, Horowitz filed a trademark application on behalf of Ecommerce Giant LLC, Knight's company. The trademark application related to the underlying patent project due to the fact that Internet World would act as the research and development company licensing out invention rights, with Ecommerce Giant licensing the invention and being responsible for commercialization efforts. On May 15, 2019, Horowitz filed a nonprovisional patent application and listed only Holt as the sole inventor because it was Holt's contention that Knight did not contribute any inventive contribution to the claims. Horowitz did not consult with Knight to independently verify inventorship, or to inform him that he was not named as a co-inventor on the nonprovisional application. Horowitz filed a response to an Office action in the trademark application on July 21, 2019. At some point in July 2019, Horowitz learned that Holt and Knight were no longer communicating. Horowitz filed a power of attorney in the nonprovisional application on July 30, 2019. During September 2019, Knight contacted Horowitz to determine the status of the nonprovisional application and to ask whether he was included as a co-inventor. On September 24, 2019, Horowitz sent Knight a letter stating that she did not know if he was entitled to be named a co-inventor. Subsequently, on September 28, 2019, Horowitz sent to Knight a power of attorney and draft assignment documents assigning his rights in the nonprovisional application to Internet World. Knight then requested a copy of the claims filed in the nonprovisional application and refused to sign either the power of attorney or the assignment documents. What if any Rules of Professional Conduct did Horowitz violate? I. Representing a client when the representation of one client will be directly adverse to another client II. Failing to act with reasonable diligence and promptness III. Failing to explain a matter to the extent reasonably necessary to permit the client to make informed decisions regarding the representation IV. Failure to take steps to the extent reasonably practicable to protect a client's interest A I B I and II C II and III D I and III E I and IV

ANSWER: (D). The key to understanding this question is to realize that Horowitz entered into an agreement with both Holt and Knight and, therefore, represented both Holt and Knight. Nevertheless, without informing or consulting with Knight, Horowitz took actions adverse to Knight at the behest of Holt. The fact pattern here is modeled after In the Matter of Jeffrey R. Ramberg, Proceeding No. D2017-12. Horowitz undertook joint representation of two applicants in a patent application. Horowitz continued to represent both applicants after she became aware of an inventorship dispute without obtaining informed consent from each to continue the representation. Horowitz named one of the two applicants as the sole inventor on the nonprovisional application that claimed priority to the original application. Horowitz failed to inform the excluded applicant and continued to represent the first applicant in the matter, despite an actual or potential conflict of interest, and without obtaining informed consent from each. Horowitz followed the directions of the applicant who paid the legal fees. Horowitz also represented one of the applicants in a trademark application that was related to the patent application, without obtaining informed consent to the representation. These actions would be violations of 37 C.F.R. §§ 11.107 (a)(l)-(2) and (b)(3)-(4) (representing a client when the representation of one client will be directly adverse to another client; representing a client when the representation of that client may be materially limited by the lawyer's responsibilities to another client; representing a client when the representation involves the assertion of a claim by one client against another client represented by the practitioner in the same proceeding without obtaining consent after full disclosure); and 11.104(a)(l)-(2) and (b) (failing to inform the client of any decision or circumstances with respect to which the client's informed consent is required; failing to consult with the client about the means by which client's objectives can be accomplished; failing to explain a matter to the extent reasonably necessary to permit the client to make an informed decision).

The procedures in the MPEP do not require an applicant claiming foreign priority in a nonprovisional utility application to: A submit the processing fee set forth in 37 CFR 1.17(i) if the claim for priority or submission of the certified copy of the priority document is made after payment of the issue fee and before the patent is granted. B identify the foreign application for which priority is being claimed as well as any foreign application for the same subject matter having a filing date before that of the application for which priority is being claimed. C file the claim in the application. D have the same inventive entity listed in the foreign application as in the U.S. application in which the priority claim has been filed. E identify the intellectual property authority or country in or for which the foreign application was filed.

ANSWER: (D). There is no requirement as to the inventive entity being the same. As to (A), see 37 C.F.R. § 1.55(a)(2). As to (E), see MPEP 201.14 at p. 200-82 (right column). As to (C), see MPEP 201.14(a). As to (B), see 37 C.F.R. § 1.55(a)(1)(i) and MPEP 201.14(a).

On January 3, 2016, a registered practitioner filed a continuation application that includes a benefit claim to a prior-filed application. The practitioner simultaneously filed in the prior-filed application an express abandonment in favor of a continuing application. The prior application contained five drawing figures described in the specification. However, the continuation application contains only four of the five drawing figures. The specification of the continuation application did not include a complete description of the missing drawing figure. A postcard from the USPTO, listing the contents of the continuation application, contains a note that only four drawing figures were received. The practitioner inadvertently omitted one of the drawing figures mentioned in the specification when he filed the continuation application. The missing drawing figure shows a claimed feature of the invention. On February 10, 2016, the practitioner received a Notice of Omitted Item(s) properly according a filing date of January 3, 2016 for the continuation application without the missing drawing figure and notifying the applicant that the drawing is missing. Which of the following procedures for filing the missing drawing would comply with the patent laws, rules and procedures as related in the MPEP for according the continuation application a January 3, 2016 filing date with the five drawing figures that were present in the application? A The practitioner files the missing drawing figure in response to the Notice of Omitted Item(s) within the time period set forth in the notice with no change in filing date. B The practitioner files the missing drawing figure and an amendment to the specification to add a complete description of the missing drawing figure in response to the Notice of Omitted Item(s) within the time period set forth in the notice. C The practitioner files an amendment to cancel the description of the missing drawing figure from the specification of the continuation application. D If the continuation application as originally filed includes an incorporation by reference of the prior-filed application to which the benefit is claimed, the practitioner can file the missing drawing figure any time prior to the first Office action. E (A) and (D)

ANSWER: (E) (D) is a correct answer. MPEP § 201.06(c), under the heading "INCORPORATION BY REFERENCE", subheading "B. Application Entitled to a Filing Date," states that "[i]f the application as originally filed includes a proper incorporation by reference of the prior application(s), an omitted specification page(s) and/or drawing figure(s) identified in a "Notice of Omitted Item(s)" may be added by amendment provided the omitted item(s) contains only subject matter in common with such prior application(s). In such case applicant need not respond to the "Notice of Omitted Item(s)." Applicant should submit the amendment adding the omitted material prior to the first Office action to avoid delays in the prosecution of the application." (A) is also correct inasmuch as an assertion of priority in the originally filed paperwork operates as an automatic incorporation by reference for inadvertently omitted subject from the priority application. (B) is incorrect because the application filing date will be the date of the filing of the missing specification section. (C) is incorrect. The continuation purposefully omitted the description of the missing drawing.

Igor filed a design patent application in the USPTO on January 24, 2000, which issued as a design patent on January 23, 2001. Igor's design patent covered a design that became immediately popular, resulting in numerous inquiries for licenses from various manufacturers. Igor would like to financially exploit his patent by licensing for five years. However, Igor has decided to dedicate five years of his patent term to the public. Which of the following is in accord with the USPTO rules and the procedures set forth in the MPEP, while best allowing Igor to pursue his intentions? A Record in the USPTO an assignment of all right, title, and interest in the patent to the public, conditioned on the receipt by Igor of all royalties from licensing the patent after the first five years of the patent term. B File a disclaimer in the USPTO dedicating to the public the first five years of the patent term. C File a disclaimer in the USPTO dedicating to the public that portion of the term of the patent from January 24, 2015 to January 24, 2020. D File a disclaimer in the USPTO dedicating to the public half of all royalties received from licensing the patent for the terminal part of the term of the patent. E File a disclaimer in the USPTO dedicating to the public that portion of the term of the patent from January 24, 2010 to January 23, 2015.

ANSWER: (E) is correct because 37 CFR § 1.321(a) states, in pertinent part, that "any patentee may disclaim or dedicate to the public...any terminal part of the term, of the patent granted." 35 U.S.C. § 173 states, "Patents for designs shall be granted for the term of fourteen years from the date of grant." (A) is wrong because such action would not permit Igor to financially exploit any portion of the term of his patent, since 37 CFR § 3.56 indicates that the result is a conditional assignment, which is regarded as an absolute assignment for Office purposes. (B) is wrong because 37 CFR § 1.321(a) provides for dedication to the public of "the entire term, or any terminal part of the term" only. "[T]he first five years of the patent term" does not qualify as a terminal part of the term. (C) is wrong because Igor would not achieve his objective of dedicating at least a portion of his patent term to the public, since the term of the design patent would expire on January 23, 2015. 35 U.S.C. § 173. (D) is wrong because 37 CFR § 1.321(a) restricts a disclaimer to "any complete claim or claims" or "the entire term, or any terminal part of the term" of a patent. "Royalties received from licensing" are not addressed by 37 CFR § 1.321(a).

Which of the following is not in accordance with the provisions of 35 USC 115 (Oath of applicant)? A The applicant shall make oath (or declaration) that he believes himself to be an original inventor of the process, machine, manufacture, or composition of matter, or improvement thereof, for which he solicits a patent. B In the oath or declaration, the applicant must state that the application is filed with their consent. C An oath may be made before any person within the United States authorized by law to administer oaths. D An oath executed in a foreign country must be properly authenticated. E A declaration which accompanies a patent application must state on the document a warning that willful false statements and the like are punishable by fine or imprisonment or both under 18 USC 1001, and the declaration must be notarized.

ANSWER: (E) is the best answer. The declaration need not be notarized. See 37 C.F.R. § 1.63(a). A declaration may be used in lieu of an oath. 37 C.F.R. § 1.68, MPEP 602. As to (A) through (C) see 35 U.S.C.§ 115. As to (D), see MPEP § 602.04 at p. 600-34, left column.

According to USPTO rules and procedure, which of the following can be overcome by an affidavit under 37 CFR 1.131 in a pre-AIA filing? A A rejection properly based on statutory double patenting. B A rejection properly made under 35 USC 102(a)(1) based on a foreign patent granted in a nonWTO country more than one year before filing in the U.S. C A rejection properly made under 35 USC 102(a)(1) based on a journal article dated one month prior to the effective filing date of the U.S. patent application. Applicant has clearly admitted on the record during the prosecution of the application that subject matter in the journal article relied on by the examiner is prior art. D A rejection properly made under 35 USC 102(a)(2) based on a U.S. patent that was filed 18 months before the effective filing date of the application. The patent discloses, but does not claim, the invention. E None of the above.

ANSWER: (E) is the correct answer. MPEP § 715. (A) is incorrect because an affidavit under 37 C.F.R. § 1.131 is not appropriate where the reference is a prior U.S. patent to the same entity, claiming the same invention. MPEP § 715. (B), (C) and (D) are each incorrect because an affidavit under 37 C.F.R. § 1.131 is not appropriate where the reference is used under AIA 35 U.S.C. § 102.

Which, if any, of the following statements is in accord with the patent laws, rules and procedures as related in the MPEP? A Where an inventor's residence is stated correctly in the 37 CFR 1.76 application data sheet and incorrectly in the inventor's 37 CFR 1.63 oath or declaration, the discrepancy must be corrected by filing a supplemental 37 CFR 1.67 oath or declaration giving the correct residence. B Where two inventors file separate 37 CFR 1.63 oaths or declarations which do not identify both inventors, the USPTO will not presume they are joint inventors and will require new oaths or declarations. C A dependent claim which repeats a limitation that appears in the claim on which it depends and adds an additional limitation is properly rejected under 35 USC 112(d). D In a statement under 37 CFR 1.97(e)(1) specifying that "each item of information contained in the information disclosure statement was first cited in any communication from a foreign patent office in a counterpart foreign application not more than three months prior to the filing of the statement," the three-month period begins on the date the communication was first received by either a foreign associate or a U.S. registered practitioner. E None of statements (A) to (D) is correct.

ANSWER: (E) is the correct answer. Regarding (A), where an application is filed with an application data sheet correctly setting forth the residence of inventor B, and an executed 37 CFR 1.63 declaration setting forth a different incorrect residence of inventor B, the Office will capture the residence of inventor B found in the application data sheet. (B) is not in accord with the patent laws, rules and procedures as related in the MPEP. MPEP § 602, under the heading "Sole Or Joint Designation," states "[w]hen joint inventors execute separate oaths or declarations, each oath or declaration should make reference to the fact that the affiant is a joint inventor together with each of the other inventors indicating them by name." The examiner should notify the inventors that their oaths or declarations are defective and that new oaths or declarations are required. (C) is incorrect. MPEP § 608.01(n), under the heading "II. Treatment Of Improper Dependent Claims," states "[c]laims which are in improper dependent form for failing to further limit the subject matter of a previous claim should be objected to under 37 CFR 1.75(c) by using form paragraph 7.36." The claim should not be rejected. Further as for (C), MPEP § 608.01(n), under heading "III. Infringement Test," states "[a] dependent claim does not lack compliance with 35 U.S.C. 112, fourth paragraph, simply because there is a question as to (1) the significance of the further limitation added by the dependent claim, or (2) whether the further limitation in fact changes the scope of the dependent claim from that of the claim from which it depends. The test for a proper dependent claim under the fourth paragraph of 35 U.S.C. 112 is whether the dependent claim includes every limitation of the claim from which it depends. The test is not one of whether the claims differ in scope." (D) is not in accord with the patent laws, rules and procedures as related in the MPEP. MPEP § 609, under heading "(B)(5) Statement Under 37 CFR 1.97(e)," states "[t]he date on the communication by the foreign patent office begins the 3-month period in the same manner as the mailing of an Office action starts a 3-month shortened statutory period for reply . ... The date which begins the 3-month period is not the date the communication was received by a foreign associate or the date it was received by a U.S. registered practitioner."

In accordance with the USPTO rules and the procedures set forth in the MPEP, which of the following is true? A In rejecting claims the examiner may rely upon facts within his own personal knowledge, unless the examiner qualifies as an expert within the art, in which case he is precluded from doing so, since only evidence of one of ordinary skill in the art is permitted. B If an applicant desires to claim subject matter in a reissue which was the same subject matter waived in the statutory invention registration of another, the applicant is precluded by the waiver from doing so, even though the applicant was not named in the statutory invention registration. C If an applicant, knowing that the subject matter claimed in his patent application was on sale in Michigan and sales activity is a statutory bar under 35 USC 102 to the claims in his application, nevertheless withholds the information from the patent examiner examining the application, and obtains a patent including the claims in question, the applicant may remove any issue of inequitable conduct by filing a request for reexamination based on the sales activity. D An applicant for a patent may overcome a statutory bar under 35 USC 102 based on a patent claiming the same invention by acquiring the rights to the patent pursuant to an assignment and then asserting the assignee's right to determine priority of invention pursuant to 37 CFR 1.602. E None of the above.

ANSWER: (E) is the most correct answer. (A) is incorrect since facts within the knowledge of the examiner may be used whether or not the examiner qualifies as an expert. 37 CFR § 1.104(c)(3). (B) is incorrect since the waiver is only effective against those named in the statutory registration. (C) is incorrect since on sale activities is not proper subject matter for reexamination, and inequitable conduct cannot be resolved or absolved by reexamination. (D) is not correct since a statutory bar cannot be overcome by acquiring the patent.

In accordance with the patent laws, rules and procedures as related in the MPEP, which of the following can a third party submit in a pending published application within six months from the publication date where the submission identifies the application to which it is directed by application number and includes the appropriate fee? A A list referencing a videotape and copy of the tape showing that the process claimed in the application was in use more than one year before the filing date of the application. B A U.S. patent issued more than one year before the filing date of the application and a written explanation of the patent made by the third party on the patent. C A publication with a publication date more than one year before the filing date of the application and including underlining made by the third party on the publication. D A protest raising fraud and inequitable conduct issues. E A list of the sole Japanese language publication submitted for consideration, including the publication date of the publication, a copy of the Japanese language publication and a written English language translation of the pertinent parts of the publication.

ANSWER: (E) is the most correct answer. 37 CFR § 1.290: MPEP 1134.01. The Rule allows that a third party may submit in an application a foreign-language publication and an English language translation of pertinent portions of the publication. The submission must "identify the application to which it is directed by application number," . . "include the fee (if more than 3 references are submitted) include "a list of the patents or publications submitted for consideration by the Office, including the date of publication of each patent or publication;". . . a " copy of each listed patent or publication in written form or at least the pertinent portions;" and an "English language translation of all the necessary and pertinent parts of any non-English language patent or publication in written form relied upon." Post 9/16/12, third party submissions are much broader than under old Rule 99. (A) is incorrect. The rule does not authorize a third-party submission of materials or things other than patents or publications. (B) is incorrect. A third-party submission may not include explanations beyond a concise explanation.. (C) is incorrect. A third-party submission may not include markings or highlights on the publications. (D) is incorrect because a protest cannot be filed in a published application. See 37 CFR § 1.291 (a)(1); MPEP 1901.06

In accordance with the USPTO rules and the procedures set forth in the MPEP, which of the following is true? A Once the issue fee has become due, provided an original application has not been pending more than three years, the applicant may request and the Office may grant a request for deferral of payment of the issue fee. B The time period set for the payment of the issue fee is statutory and cannot be extended. However, if payment is not timely made and the delay in making the payment is unintentional, upon payment of a fee for delayed payment, it may be accepted as though no abandonment had occurred, but there will be a reduction on the patent term adjustment for the period of abandonment. C Upon written request, a person citing patents and printed publications to the Office that the person believes has a bearing on the patentability of a particular patent, may request that his or her name remain confidential. D To obtain benefit of priority based on an earlier filed U.S. patent application, an applicant in a later filed continuation application is not required to meet the conditions and requirements of 35 USC 120. E Each of statements (B) and (C) is true.

ANSWER: (E) is the most correct answer. As to (B), see 35 U.S.C. §§ 151; 154(b)(2)(ii) and (iii); 37 CFR § 1.704(c)(3); MPEP § 1306. As to (C) see MPEP §§ 2203 and 2212. As to (D), the claim for priority is not required, as a person may not wish to do so in order to increase the term of his or her patent. As to (A) deferral under 37 CFR § 1.103 is not available following the notice of allowance. Since (B) and (C) are correct, (E) is the best answer

A registered practitioner files a nonprovisional utility application in 2014. In 2016, the practitioner files a continuation-in-part application and claims benefit of the filing date of the 2014 application for the 2016 application. Thereafter, the practitioner amends the 2016 application to include claims that were not present in the either the originally filed 2014 application or the originally filed 2016 application. The primary examiner properly concludes that the added claims are not supported by the original disclosure in either application. Which of the following is in accord with the USPTO rules and the procedures set forth in the MPEP? A The added claims are rejected for lack of written description under 35 USC 112, first paragraph. B The added claims are rejected as new matter under 35 USC 132. C The added claims are denied benefit of the filing date of the 2014 application. D (A) and (B). E (A) and (C).

ANSWER: (E) is the most correct answer. Both (A) and (C) are correct. MPEP § 2163.01, under the heading "Support For The Claimed Subject Matter In The Disclosure," states that "[I]f the examiner concludes that the claimed subject matter is not supported [described] in an application as filed, this would result in a rejection of the claim on the ground of a lack of written description under 35 U.S.C. 112, first paragraph, or denial of the benefit of filing date of a previously filed application." (B) is incorrect. MPEP § 2163.01 states that unsupported claims "should not be rejected or objected to on the ground of new matter. As framed by the court in In re Rasmussen, 650 F.2d 1212, 211 USPQ 323 (CCPA 1981), the concept of new matter is properly employed as a basis for objection to amendments to the abstract, specification or drawings attempting to add new disclosure to that originally presented." (D) is incorrect because (B) is incorrect.

A registered practitioner files a nonprovisional utility application in 2015. In 2017, the practitioner files a continuation-in-part application and claims benefit of the filing date of the 2015 application for the 2017 application. Thereafter, the practitioner amends the 2017 application to include claims that were not present in either the originally filed 2015 application or the originally filed 2017 application. The primary examiner properly concludes that the added claims are not supported by the original disclosure in either application. Which of the following is in accord with the patent laws, rules and procedures as related in the MPEP? A The added claims are rejected for lack of written description under 35 USC 112(a). B The added claims are rejected as new matter under 35 USC 132. C The added claims are denied benefit of the filing date of the 2015 application. D (A) and (B). E (A) and (C).

ANSWER: (E) is the most correct answer. Both (A) and (C) are correct. MPEP § 2163.01, under the heading "Support For The Claimed Subject Matter In The Disclosure," states that "[I]f the examiner concludes that the claimed subject matter is not supported [described] in an application as filed, this would result in a rejection of the claim on the ground of a lack of written description under 35 U.S.C. 112, first paragraph, or denial of the benefit of filing date of a previously filed application." (B) is incorrect. MPEP § 2163.01 states that unsupported claims "should not be rejected or objected to on the ground of new matter. As framed by the court in In re Rasmussen, 650 F.2d 1212, 211 USPQ 323 (CCPA 1981), the concept of new matter is properly employed as a basis for objection to amendments to the abstract, specification or drawings attempting to add new disclosure to that originally presented." (D) is incorrect because (B) is incorrect.

On January 2, 2016, a registered practitioner filed a patent application with the USPTO for inventor Bock. The application includes a specification and a single claim to the invention, which reads as follows: 1. A new string consisting only of material Z that has the ability to stretch to beyond its initial unstretched length. On June 2, 2016, the practitioner received an Office action from the primary examiner rejecting the claim. The claim is solely rejected under 35 USC 102 in view of Patent A, which discloses a string consisting only of material Z. The Office action states, "Patent A discloses a string consisting only of material Z. Patent A does not expressly teach the stretchability property of the string. Nevertheless, the recited stretchability is inherent in the string of patent A. Accordingly, patent A anticipates the claimed string." Mr. Bock believes he is entitled to a patent to his new string and authorizes the practitioner to reply to the Office action by arguing that his string stretches to ten times its initial unstretched length, something that patent A does not teach. Since this is not expressly taught in Patent A, the practitioner argues, Patent A cannot anticipate the claimed string. In accordance with USPTO rules and procedures set forth in the MPEP, is the practitioner's reply persuasive as to error in the rejection? A Yes. B Yes, but the claim should now be rejected again, this time under 35 USC 103 as obvious over Patent A. C Yes, because the stretchability property is expressly taught by Patent A. D Yes, examiner nowhere addresses the claimed limitation of stretching the string beyond its initial unstretched length. E No.

ANSWER: (E) is the most correct answer. MPEP § 2112, under the heading "Something Which Is Old Does Not Become Patentable Upon The Discovery Of A New Property," states that "claiming of a new use, new function or unknown property which is inherently present in the prior art does not necessarily make the claim patentable. In re Best, 562 F.2d 1252, 1254, 195 USPQ 430, 433 (CCPA 1977)." The issue is whether the argument has persuasively rebutted the examiner's prima facie case of anticipation. The argument does not rebut the prima facie case. The claim is directed to a string consisting only of material Z. Patent A teaches each every element of that string; i.e., the string shape, material Z, and the string only consists of material Z. There is nothing different between the string of the claim and that of patent A. Under those circumstances, examiner correctly stipulated that the stretchability of the claimed string; i.e., the ability to stretch the string beyond its initial unstretched length, would be an inherent property of the disclosed string. Whether or not patent A teaches the stretchability of its string is not defeating. Even if Mr. Bock had discovered a new property for the string, it would still not render the claim patentable. In re Best, supra . Here all the product elements are the same and examiner made out a proper prima facie case of anticipation. The burden now shifts to the practitioner to show that the patent string is not the same. The reply, which seeks to establish a difference in properties without showing a concomitant difference in product material and shape is not persuasive as to error in the rejection. All the other answers are wrong. (A) is not the most correct answer. See MPEP § 2112. (B) is not the most correct answer. The anticipation rejection was correctly established and was not rebutted by the argument. (C) is not the most correct answer. See MPEP § 2112, discussing when a reference can anticipate based on an inherent, as opposed to an expressly or implicit, disclosure. (D) is not the most correct answer. In discussing the stretchability property, the recited ability to stretch the string beyond its initial unstretched length was equally addressed.

In accordance with the USPTO rules and the procedures set forth in the MPEP, which of the following documents, if any, must also contain a separate verification statement? A Small entity statements. B A petition to make an application special. C A claim for foreign priority. D An English translation of a non-English language document. E None of the above

ANSWER: (E) is the most correct answer. MPEP § 410 makes clear that the certification requirement set forth in 37 CFR § 10.18(b) "has permitted the PTO to eliminate the separate verification requirement previously contained in 37 CFR ...1.27 [small entity statements], ...1.52 [English translations of non-English documents], ...1.55 [claim for foreign priority], [and] ...1.102 [petition to make an application special]."

Determine which of the following documents, if any, must also contain a separate verification statement in accordance with the patent laws, rules and procedures as related in the MPEP. A A request to correct inventorship in a pending application. B A petition to make an application special. C A claim for foreign priority. D A substitute specification. E None of the above.

ANSWER: (E) is the most correct answer. MPEP § 410 states that the certification requirement set forth in 37 CFR § 11.18(b) "has permitted the USPTO to eliminate the separate verification requirement previously contained in 37 CFR ...1.48 [correction of inventorship in a patent application], ...1.55 [claim for foreign priority], ...1.102 [petition to make an application special], [and] ... 1.125 [substitute specification]."

A registered practitioner files an application on the client's discovery that adding silica to a known plastic composition containing the flame retardant, X, results in increased flame retardance. The application claims a composition comprising the known plastic composition containing X and also silica. The primary examiner rejects the claim on the basis that applicant admits that X was a known flame retardant and that there is no evidence of improved flame retardance. In accordance with the patent laws, rules and procedures as related in the MPEP, a proper reply could include which of the following argument(s) to rebut and overcome the rejection? A The examiner cannot rely on admitted prior art. B The examiner has not shown that the prior art appreciated applicant's discovery of silica to be a flame retardant. C The examiner has not made out a prima facie case of obviousness due to lack of motivation in the prior art or in the knowledge generally available to one of ordinary skill in the art for adding silica to the known plastic composition. D The applicant does not have to show an improved or unexpected result for the claimed invention. E (C) and (D).

ANSWER: (E) is the most correct answer. See KSR guidelines and MPEP §§ 2141, 2142, 2143 and 2143.01. Regarding (C) MPEP 2142 under the heading "ESTABLISHING A PRIMA FACIE CASE OF OBVIOUSNESS," states "To establish a prima facie case of obviousness, three basic criteria must be met. First, there must be some suggestion or motivation, either in the references themselves or in the knowledge generally available to one of ordinary skill in the art, to modify the reference or to combine reference teachings. Second, there must be a reasonable expectation of success. Finally, the prior art reference (or references when combined) must teach or suggest all the claim limitations. The teaching or suggestion to make the claimed combination and the reasonable expectation of success must both be found in the prior art, and not based on applicant's disclosure. In re Vaeck, 947 F.2d 488, 20 USPQ2d 1438 (Fed. Cir. 1991). See MPEP § 2143 for decisions pertinent to each of these criteria." MPEP § 2143 states the same criteria, and further "The teaching or suggestion to make the claimed combination and the reasonable expectation of success must both be found in the prior art, not in applicant's disclosure. In re Vaeck, 947 F.2d 488, 20 USPQ2d 1438 (Fed. Cir. 1991)." Regarding motivation, MPEP 2143.01 states "Obviousness can only be established by combining or modifying the teachings of the prior art to produce the claimed invention where there is some teaching, suggestion, or motivation to do so found either explicitly or implicitly in the references themselves or in the knowledge generally available to one of ordinary skill in the art. 'The test for an implicit showing is what the combined teachings, knowledge of one of ordinary skill in the art, and the nature of the problem to be solved as a whole would have suggested to those of ordinary skill in the art.' In re Kotzab, 217 F.3d 1365, 1370, 55 USPQ2d 1313, 1317 (Fed. Cir. 2000). See also In re Fine, 837 F.2d 1071, 5 USPQ2d 1596 (Fed. Cir. 1988); In re Jones, 958 F.2d 347, 21 USPQ2d 1941 (Fed. Cir. 1992)." The examiner has not shown any suggestion or motivation, either in the prior art or in the knowledge generally available to one of ordinary skill in the art, to modify the known plastic composition. Regarding (D) MPEP § 2142 states "The examiner bears the initial burden of factually supporting any prima facie conclusion of obviousness. If the examiner does not produce a prima facie case, the applicant is under no obligation to submit evidence of nonobviousness. If, however, the examiner does produce a prima facie case, the burden of coming forward with evidence or arguments shifts to the applicant who may submit additional evidence of nonobviousness, such as comparative test data showing that the claimed invention possesses improved properties not expected by the prior art." (E) is the most correct answer since it addresses both issues raised by the examiner, obviousness and evidence of improved results. (C) alone is incorrect inasmuch as it does not address the examiner's comments regarding improved results. (D) alone incorrect because it does not address the examiner's burden of presenting a prima facie case of obviousness. (A) is not correct. MPEP § 2129. Admissions by applicant can be used as prior art. (B) is not correct. MPEP § 2144. It is not necessary for a finding of obviousness that prior art be combined for the same advantage or result as applicant.

A registered practitioner properly recorded an assignment document for application A identifying XYZ Company as the assignee. The document assigns to XYZ Company the "subject matter claimed in Application A." A proper restriction requirement was made by a primary examiner in application A between two distinct inventions, and the practitioner elected to prosecute one of the inventions. Application A was prosecuted, and later became abandoned. Before the abandonment date of application A, the practitioner filed a complete application B as a proper divisional application of application A. Application B claimed the nonelected invention of Application A, and was published as a U.S. application publication. XYZ Company remains the assignee of application A. What must the practitioner do in accordance with the USPTO rules and the procedures set forth in the MPEP to ensure that XYZ Company is listed as the assignee on the face of any patent issuing from application B? A File a proper assignment document in application B identifying XYZ Company as the assignee. B File a proper assignment document in application B identifying XYZ Company as the assignee, and confirm that USPTO's bibliographic data for application B identifies XYZ Company as the assignee by checking the filing receipt for application B, the U.S. application publication of application B, or the USPTO's Patent Application Information Retrieval (PAIR) system data for application B, depending on when the practitioner filed the assignment document in application B. C Confirm that XYZ Company is identified as the assignee on the U.S. application publication of application B. D File a proper assignment document in application B identifying XYZ Company as the assignee, and confirm that XYZ Company is identified as the assignee on the U.S. application publication of application B. E Upon allowance of application B, the practitioner must identify XYZ Company as the assignee in the appropriate space on the Issue Fee Transmittal form for specifying the assignee for application B.

ANSWER: (E) is the most correct answer. See MPEP §§ 306 and 307. MPEP § 306 states, "In the case of a division or continuation application, a prior assignment recorded against the original application is applied to the division or continuation application because the assignment recorded against the original application gives the assignee rights to the subject matter common to both applications." MPEP § 307 states, "Irrespective of whether the assignee participates in the prosecution of the application, the patent issues to the assignee if so indicated on the Issue Fee Transmittal form PTOL-85B. Unless an assignee's name and address are identified in item 3 of the Issue Fee Transmittal form PTOL85B, the patent will issue to the applicant. Assignment data printed on the patent will be based solely on the information so supplied." A new assignment document need not be recorded for a divisional or continuation application where the assignment recorded in the parent application remains the same. Accordingly, answers (A), (B) and (D) are incorrect. In addition, (A), (B) and (D) are incorrect because unless an assignee's name and address are identified in item 3 of PTOL-85B, the patent will issued to the application and the assignee information, even if recorded, will not appear on the patent. (C) is incorrect for the same reason. (B) is also incorrect. There is no connection between the filing receipt, PAIR or the patent application publication and the recorded assignment. Assignment data is reflected on the filing receipt, PAIR, or a patent application publication when applicant includes assignment information for purposes of publication of the application on the transmittal letter. Assignment data printed on the patent will be based solely on the information supplied on the Issue Fee Transmittal Form PTOL-85B. See MPEP §§ 1309 and 1481. Accordingly, answer (E) is correct and answer (C) is incorrect.

On February 24, 2018, a registered practitioner filed a patent application with the USPTO for inventor Jones. The application includes a specification and a single claim directed to a fertilizer encapsulated so that it can be placed in the ground and slowly leach out over time, thereby releasing said fertilizer to the plant or tree. The claim covers a compound comprising a core that is a mixture of fertilizer and hardening agent X. The core is surrounded by an inner layer made out of inert compound Y, which is water soluble. There is an outer layer made out of compound Z disposed around the inner layer to further encapsulate the core. Jones stumbled across this invention fortuitously having been the first to discover that the addition of Y forestalls the interaction of X and Z, thereby increasing the shelf life of the invention. The patent examiner rejects the claim on obviousness grounds citing Aikman in view of Smith and further in view of Irving. Aikman teaches the fertilizer compound, Smith teaches the addition of hardening agent X, and Irving teaches the encapsulation of the fertilizer with Z. The examiner explains that it would be obvious to one of skill in the art to add a known inactive ingredient, Y. According to the patent laws, rules and procedures, which of the following arguments would overcome the rejection? A A person of skill in the art would have chosen a compound other than Y had they appreciated the problem created by the interaction between X and Z. B Jones stumbled upon the invention fortuitously and, therefore, the invention cannot be rightfully characterized as obvious. C The combination of Aikman, Smith and Irving is inappropriate because the references alone do not provide each and every element found in the claimed invention. D A person of skill in the art would not be familiar with the teachings of Aikman, Smith and Irving and, therefore, they constitute nonanalogous art. E None of the above.

ANSWER: A. Example 4.1. Teaching point: Even where a general method that could have been applied to make the claimed product was known and within the level of skill of the ordinary artisan, the claim may nevertheless be nonobvious if the problem which had suggested use of the method had been previously unknown. SPECIFIC POINT: In re Omeprazole Patent Litigation, 536 F.3d 1361 (Fed. Cir. 2008). The Federal Circuit affirmed the district court's decision that the claimed invention was not obvious, in part because a person of skill in the art likely would have chosen a different modification even if he or she had recognized the problem.

When considering whether a method claim embodies an unpatentable abstract idea, which of the following would tend to weigh in favor of patent elibigility?" 1.A particular machine or apparatus is incorporated into the claimed method." 2.A machine or apparatus is an object on which the method operates." 3.The recited machine or apparatus contributes nominally to the execution of the claimed method. A Statement 1 B Statement 2 C Statement 3 D Statement 1 and 2 E Statement 1 and 3.

ANSWER: A. If a particular machine or apparatus is incorporated into the claimed method, this weighs in favor of patent eligibility in that there is "significantly more" than the method alone. This is particularly true when the method steps are specifically tied to the machine or apparatus. Factors weighing against eligibility are: (1) No recitation of a machine or transformation (either express or inherent). (2) Insufficient recitation of a machine or transformation. (3) Involvement of machine or transformation with the steps is merely nominally, insignificantly, or tangentially related to the performance of the steps. (4) Machine is generically recited such that it covers any machine capable of performing the claimed step(s). (5) Machine is merely an object on which the method operates. (6) Transformation involves only a change in position or location of an article. (7) The claim would monopolize a natural force or patent a scientific fact; e.g., by claiming every mode of producing an effect of that law of nature. (8) A law of nature is applied in a merely subjective determination. (9) A law of nature is merely nominally, insignificantly, or tangentially related to the performance of the steps. (10) The claim is a mere statement of a general concept. (11) Use of the concept, as expressed in the method, would effectively grant a monopoly over the concept. (12) Both known and unknown uses of the concept are covered, and can be performed through any existing or future-devised machinery, or even without any apparatus. (13) The claim only states a problem to be solved. (14) The general concept is disembodied. (15) The mechanism(s) by which the steps are implemented is subjective or imperceptible

Which of the following petitioners may file a inter partes review challenge to an issued patent in accordance with USPTO rules and procedures? A Petitioner previously filed a post-grant review of claims 1-15 citing James and Wyatt, but now seeks to challenge claims 16-30 of the same patent citing Cox and Warner, which are unrelated to James and Wyatt. B Petitioner previously filed a post-grant review of claims 1-15 based on prior art patents to Smith and Jones. Petitioner now wishes to challenge claims 1-15 based on newly discovered thesis. C Petitioner previously filed a civil action challenged the validity of claims 1-15 of the patent but now wishes to file an inter partes review to challenge claims 16-30 of the same patent. D The petitioner seeks to file an inter partes review 13 months after being served with a complaint alleging infringement of the patent. E The petitioner seeks to file an inter partes review 13 months after being served with a complaint alleging infringement of the patent, but before the district court holds its Markman hearing.

ANSWER: A. In the Federal Register Notice announcing the Final Rules to implement changes to inter partes review proceedings, a comment was made expressing concern that successive challenges could be brought against the same patent in piecemeal fashion challenging different claims. This concern is a result of the fact that 35 U.S.C. 315 (e), as amended, and 35 U.S.C. 325(e) provide for estoppel on a claimby-claim basis, for claims in a patent that result in a final written decision. The Patent Office explained: "The Office recognizes these concerns and will exercise its authority under 35 U.S.C. 325(d), where appropriate, to deny petitions that submit the same or substantially the same prior art or arguments previously presented to the Office." Thus, A is correct because the claims being challenged are different and the prior art unrelated. B is incorrect because the estoppel provisions of 35 U.S.C. 318(a) and 328(a) prohibit a challenge being brought in either inter partes or postgrant review on any ground that the petitioner raised or reasonably could have raised. The USPTO interprets this to include new grounds. C is incorrect because a petitioner may not bring an inter partes challenge after the commencement of litigation challenging any claims of the same patent. See 35 U.S.C. 315(a)(1). D and E are incorrect because an inter partes challenge cannot be brought more than 1 year after the date on which the petitioner was served with a complaint alleging infringement.

Applicant submits a patent application claiming rabeprazole and its salts. Rabeprazole is part of a class of drugs known as proton pump inhibitors, which suppress gastric acid production by inhibiting action of a particular enzyme. The examiner asserts that a combination of two prior art references renders the raberprazole claims obvious. These are: (1) European Patent No. 174,726 (owned by Takeda), claiming lansoprazole; and ( 2) an article by Brändström. Lansoprazole, an anti-ulcer compound, differs structurally from rabeprazole at one position, but otherwise the two compounds are identical. Brändström describes a class of antiulcerative compounds in which rabeprazole, lansoprazole, and omeprazole are all core structure compounds. The examiner explains that based on structural similarity and advantageous properties, one of ordinary skill in the art would have selected lamsoprazole as a lead compound and then proceeded to modify this known compound in a particular and predictable way to achieve the claimed rabeprazole, useful for suppression of gastric acid. Applicant, in a factually correct statement, explains that there is no discernible reason for a skilled artisan to begin with lansoprazole in search of an anti-gastric acid compound only to drop the very feature that gives lansoprazole advantageous properties. Applicant concludes that, therefore, the claimed rabeprazole is patentable. This argument should ultimately be: A Successful because a claimed compound is not obvious where there was no reason to modify the closest prior art lead compound to obtain the claimed compound, and modifying the lead compound would destroy its advantageous property. B Successful because the chemical arts are highly unpredictable and, pursuant to KSR, claimed inventions in highly unpredictable arts are not obvious. C Successful because the chemical arts are highly unpredictable. D Unsuccessful because the prior art as it existed prior to the time of invention would give some reasons for one of skill in the art to make particular modifications to achieve the claimed compound. E Unsuccessful because the known and structurally similar lansoprazole for treatment of ulcers would be a logical starting point (i.e., lead compound) for those searching for a new anti-gastric acid compound, such as the claimed rabeprazole.

ANSWER: A. See Eisai Co. Ltd. v. Dr. Reddy's Labs. Teaching point: "A claimed compound would not have been obvious where there was no reason to modify the closest prior art lead compound to obtain the claimed compound and the prior art taught that modifying the lead compound would destroy its advantageous property."

A registered practitioner filed a utility application on February 11, 2016. On April 4, 2016, the practitioner filed an information disclosure statement (IDS) in the application. The practitioner received a notice of allowance dated January 3, 2017 soon after it was mailed. When discussing the application with the practitioner on January 21, 2017, and before paying the issue fee, the client notices for the first time that a reference, which is one of many patents obtained by the client's competitor, was inadvertently omitted from the IDS. The client has been aware of this reference since before the application was filed. The client is anxious to have this reference appear on the face of the patent as having been considered by the USPTO. Which of the following actions, if taken by the practitioner, would not be in accord with the USPTO rules and the procedures set forth in the MPEP? A Before paying the issue fee, timely file an IDS citing the reference, along with the certification specified in 37 CFR 1.97(e), and any necessary fees. B Within three months of the mail date of the notice of allowance, without paying the issue fee, timely file a Request for Continued Examination (RCE) under 37 CFR 1.114, accompanied by the fee for filing an RCE, and an IDS citing the reference. C Within three months of the mail date of the notice of allowance, without paying the issue fee, timely file a continuing application under 37 CFR 1.53(b), an IDS citing the reference, and any necessary fees. D After paying the issue fee, timely file a petition to withdraw the application from issue to permit the express abandonment of the application in favor of a continuing application, a continuation application under 37 CFR 1.53(b), an IDS citing the reference, and any necessary fees. E After paying the issue fee, timely file a petition to withdraw the application from issue to permit consideration of a Request for Continued Examination (RCE) under 37 CFR 1.114, the fee for filing an RCE, and an IDS citing the reference.

ANSWER: Answer (A), describing a procedure that is not in accordance with the USPTO rules and the procedures set forth in the MPEP, is the most correct answer. See MPEP § 609, under the heading "Minimum Requirements for an Information Disclosure Statement," and subheading "B(3). Information Disclosure Statement Filed After B(2), but Prior to Payment of Issue Fee 37 CFR 1.97 (d)", and subheading "B(5) Statement Under 37 CFR 1.97(e)." The statement specified in 37 CFR § 1.97(e) requires that the practitioner certify, after reasonable inquiry, that no item of information contained in the IDS was known to any individual designated in 37 CFR § 1.56(c) more than three months prior to the filing of the information disclosure statement. The practitioner cannot certify this because the reference was known to the client before February 11, 2016, the time of filing of the utility application, which was more than three months prior to the filing of the information disclosure statement. (B), stating a procedure that conforms with the USPTO rules and the procedures set forth in the MPEP, is an incorrect answer. Under 37 CFR § 1.313(a), a petition to withdraw the application from issue is not required if a proper RCE is filed before payment of the issue fee. (C), stating a procedure that conforms with the USPTO rules and the procedures set forth in the MPEP, is an incorrect answer. A practitioner can file a continuing application on or before the date that the issue fee is due and permit the parent application to become abandoned for failure to pay the issue fee. (D), stating a procedure that conforms with the USPTO rules and the procedures set forth in the MPEP, is an incorrect answer. Under 37 CFR § 1.313(c)(3), a petition to withdraw the application from issue can be filed after payment of the issue fee to permit the express abandonment of the application in favor of a continuing application. (E), stating a procedure that conforms with the USPTO rules and the procedures set forth in the MPEP, is an incorrect answer. Under 37 CFR § 1.313(c)(2), a petition to withdraw the application from issue can be filed after payment of the issue fee to permit consideration of a Request for Continued Examination (RCE) under 37 CFR § 1.114. See also MPEP § 1308.

Which of the following statements is not true with respect to determining patent eligibility of a claim? A The subject matter courts have found to be outside of the statutory categories of invention are limited to abstract ideas, laws of nature and natural phenomena. B In order to transform a patent-ineligible law of nature into a patent-eligible invention, the claim must incorporate applications of those laws. C The Supreme Court's concern that drives its jurisprudence relating to judicial exceptions to patent eligibility is pre-emption. D Abstract ideas, laws of nature, and natural phenomenon are the basic tools of scientific and technological work. Thus, the courts have expressed concern that monopolizing these tools by granting patent rights may impede innovation rather than promote it. E The courts have held that an application of an abstract idea, law of nature or natural phenomenon to a new and useful end is eligible for patent protection.

ANSWER: B In Mayo v. Prometheus, the Supreme Court explained that one must do more than simply state the law of nature while adding the words "apply it" (citing Gottschalk v. Benson). (A) is not the correct choice because it is a true statement. At least for the time being, abstract ideas, laws of nature and natural phenomena are the only judicially recognized exceptions to patent eligibility (See MPEP 2103). Courts will from time to time use additional characterizations (i.e., mental processes, disembodied concepts, disembodied mathematical algorithms) but they will related to one of the three judicially recognized exceptions. (C) is not the correct choice because it is a true statement. The Supreme Court continues to explain in case after case that it is primarily concerned with pre-emption. (D) is not the correct choice because it is a true statement. The Supreme Court views abstract ideas, laws of nature and natural phenomena as basic scientific building blocks and fears that granting exclusive rights (or, as they say, "monopoly rights") will inhibit rather than promote innovation. (E) is not the correct choice because it is a true statement. The fact that an invention relies upon, leverages or incorporates an abstract idea, law of nature or natural phenomena does not as a matter of fact (or law) mean that the claimed invention is patent-ineligible. This makes sense; otherwise, nothing would be patent-eligible, given that every invention necessarily starts with an idea.

Claim 25 of a patent you are prosecuting is directed to a method of treating a meat product to reduce a microbial population in the meat product. The method comprises steps for treating meat, such as poultry, with a known PAA composition (i.e., antibacterial solution) under specified conditions. The claim requires specified temperature, spray pressure of 50 psi, and contact time limitations. The Colonel patent is prior art and discloses the temperature and contact time limitations for PAA, but does not specifically disclose treatment of poultry with PAA. The Sanders patent is prior art and discloses spraying an antibacterial solution other than PAA onto poultry at a pressure of 20 to 150 psi. The examiner rejects claim 25 because it would have been obvious to combine the high-pressure parameter disclosed in Sanders with the PAA methods disclosed by Colonel. The rejection made by the examiner is: A Inappropriate because the use of a specific antimicrobial, specifically PAA, is not taught by either Colonel or Sanders; thereby, the combination of Colonel and Sanders does not provide each and every element of claim 25. B Appropriate because the advantages of spraying antimicrobial solutions onto meat at high pressure were known; therefore claim 25 is obvious because the combination of Colonel and Sanders is merely the combination of familiar elements to yield predictable results. C Inappropriate because there is no mention in either Colonel or Sanders suggesting the desirability of using PAA at any pressure, which is a limitation of claim 25 that renders the claimed invention unique. D Appropriate because patent examiners are allowed to combine references that result in a close approximation to the claimed invention and satisfy their prima facie burden by assuming that one of skill in the art would make the combination. E Inappropriate if the applicant argues that an examiner is not allowed to combine references and merely assumes that one of skill in the art would understand the interchangeability of antibacterial solutions.

ANSWER: B. Ecolab, Inc. v. FMC Corp., 569 F.3d 1335 (Fed. Cir. 2009). A combination of known elements would have been prima facie obvious if an ordinarily skilled artisan would have recognized an apparent reason to combine those elements and would have known how to do so. Here, PAA is known to exist. Colonel teaches two-thirds of the claimed invention, with Sanders teaching the remaining one-third of the invention. The only uniqueness would be substituting PAA (a known antibacterial solution) for other known antibacterial solutions disclosed in Sanders. Substituting one known antibacterial solution for another known antibacterial solution does not render a claim non-obvious. An apparent reason to combine in conjunction with the technical ability to combine led to the conclusion that the claimed invention was obvious.

If a general concept is involved in executing the steps of the method, this can be a clue that the claim is drawn to an abstract idea. Which of the following are not examples of a general concept? A Economic practices or theories. B Mental activities such as forming a judgment, observing or evaluating. C A concept illustrated by example in some tangible way. D Instructions on how business should be conducted. E Interpersonal interactions or relationships.

ANSWER: C. A concept illustrated in some tangible or concrete way is not an example of what the Patent Office excludes from patentabililty as a general concept

Regarding obviousness, which of the following are not true? A The mere existence of a large number of options does not in and of itself lead to a conclusion of nonobviousness. B Even when only a small number of possible choices exist, the obvious-to-try line of reasoning is not always appropriate. C A mere reason to combine in conjunction with the technical ability to combine does not generally lead to a conclusion that the claimed invention would have been obvious. D When constituent elements are being employed in accordance with their recognized functions, and would have predictably retained their respective functions when combined, the claimed invention is obvious. E All of the above are true.

ANSWER: C. C is FALSE. Example 4.4. Ecolab, Inc. v. FMC Corp., 569 F.3d 1335 (Fed Cir. 2009). Teaching point: A combination of known elements would have been prima facie obvious if an ordinarily skilled artisan would have recognized an apparent reason to combine those elements and would have known how to do so. A is TRUE. Example 4.17. Bayer Schering Pharma A.G. v. Barr Labs., Inc., 575 F.3d 1341 (Fed. Cir. 2009). Where the prior art teachings lead one of ordinary skill in the art to a narrower set of options, then that reduced set is the appropriate one to consider when determining obviousness using an obvious-to-try rationale. B is TRUE. Example 4.18. Sanofi-Synthelabo v. Apotex, Inc., 550 F.3d 1075 (Fed. Cir. 2008). Obvious to try is inappropriate when, upon consideration of all of the evidence, the outcome would not have been reasonably predictable and the inventor would not have had a reasonable expectation of success. D is TRUE. Example 4.5. Wyers v. Master Lock Co. (Fed. Cir. July 22, 2010).

Miguel Hagarthy is the inventor of a hydraulic utility system for supplying a pressurized fluid to a number of user devices. He filed a nonprovisional patent application on July 20, 2008, and was ultimately awarded a patent on June 3, 2011. Upon receiving a Notice of Allowance, he contacted Norman Heathrow, a well-known patent litigator who specializes in representing so-called nonpracticing entities in patent infringement matters. Hagarthy has reason to know that Narcissist Technologies is selling a hydraulic system that infringes upon at least one of the claims in his patent. At the suggestion of Heathrow, an action is filed in the United States Federal District Court for the Eastern District of Texas. A complaint is also filed with the International Trade Commission (ITC). The ITC institutes an investigation. In the ITC proceeding, Hagarthy submits a particular interpretation of the claims in question, along with various claim charts and supporting documentation. As the case proceeds in the Eastern District of Texas, Hagarthy again submits documents and argument relative to what he believes is the proper interpretation of the claims at issue. The documents and argumentation to the district court are different than those submitted to the ITC. Paranodial Automotive, Inc. (PA) fears that they may be next on the list for Hagarthy and Heathrow. They approach you and inquire about the possibility of filing something with the Patent Office to put forth the inconsistent interpretations of the claim scope, as well as certain other prior art they would like to be made of record in the event that someone decides to file a request for reexamination. Before PA is able to determine what is the best course of action to pursue, Narcissist Technologies files a written submission containing Hagarthy's claim scope statements made in the Eastern District of Texas. The following day, a request for ex parte reexamination of the Hagarthy patent is filed. The request for reexamination submitted to the USPTO, among other things, relies heavily on the statements made by Hagarthy in filings in the Eastern District of Texas. Which of the following statements, if any, are correct with respect to how the Patent Office may use the information relative to the patent owner's filings in the Eastern District of Texas? A Statements made by the patent owner in a Federal court cannot be used during reexamination proceedings. B Statements made by the patent owner in a Federal court may be used to determine whether there is a substantial new question of patentabiity, but may not thereafter be used to determine validity of any claim. C Statements made by the patent owner in a Federal court may not be used to determine whether to order reexamination of the Hagarthy patent, but if reexamination is ordered, such statements may be considered if they have previously been made a part of the file. D Statements made by the patent owner in Federal court can only be considered by the Patent Office if there are contradictory interpretations of the same claim offered by the patent owner. E None of the above.

ANSWER: C. Pursuant to 37 CFR 1.515(a), a submitted statement and any accompanying information submitted pursuant to Section 1.501(a) (2) will not be considered by the patent examiner when making a determination on any request for reexamination. While perhaps somewhat counterintuitive, this interpretation is clearly in keeping with the text of AIA. 35 U.S.C. 301(d) states in relevant part, "A written statement...shall not be considered by the Office for any purpose other than to determine the proper meaning of a patent claim in a proceeding that is ordered or instituted pursuant to section 304, 314, or 324." Thus, the AIA prohibits the use of the statement for any purpose other than determining the claim scope in a proceeding that has already been ordered or instituted. It may not be considered when determining whether to order or institute the proceeding. Notwithstanding, any statement of the patent owner and any accompanying information that is a part of the record in a patent being reexamined may be considered by the patent examiner to determine the proper meaning of a patent claim after a reexamination proceeding has been commenced. See 37 CFR 1.522(a). Therefore, A is incorrect because the statement can be used once reexamination is initiated. B is incorrect because it cannot be used to determine whether to institute reexamination. D is incorrect because there is no requirement that patent owner statements relative to claim scope be permitted only when there is a conflict of interpretation. E is incorrect because C is correct

With respect to obviousness, which of the following are false? 1.A central principle in the obviousness inquiry is whether the improvement is more than the predictable use of prior art elements according to their established functions. 2.A prima facie case of obviousness for a chemical compound begins with the reasoned identification of a lead compound. 3.In the chemical arts, obviousness determinations are typically straightforward and not complicated because potential solutions are likely to be genuinely predictable. A Statement 1 B Statement 2 C Statement 3 D Statement 1 and 3 E Statement 2 and 3

ANSWER: C. Statement 3 is FALSE. See Eisai Co. Ltd. v. Dr. Reddy's Labs, which says: "To the extent an art is unpredictable, as the chemical arts often are, KSR's focus on these "identified, predictable solutions' may present a difficult hurdle because potential solutions are less likely to be genuinely predictable." Statement 1 is TRUE. See Muniauction, Inc. v. Thomson Corp., 532 F.3d 1318 (Fed. Cir. 2008). Statement 2 is TRUE. A prima facie case of obviousness for a chemical compound begins with the reasoned identification of a lead compound.

On behalf of inventor Butch, you file a patent application directed to a retractable segmented covering system, which can be used to cover almost any structure or container, including truck trailers. Independent claim 1 explains the invention includes a plurality of flexible cover sections with a plurality of parallel supporting bows spaced therebetween and a drive assembly. Each cover section is detachably connected between the supporting bows and is removable independent of the other cover sections. You receive a first office action rejecting claim 1 as being obvious over Cassidy in view of Kidd. Cassidy discloses a truck cover including every limitation of claim 1 except for cover sections that can be removed from the cover system independent of the other cover sections. Kidd discloses a truck cover made up of a number of cover sections. In accordance with patent laws, rules and procedures, this obviousness rejection is: A Correct because if a claimed invention incorporates each and every limitation of the prior art, the invention as claimed fails to satisfy the novelty requirement of 35 U.S.C. 102. B Correct because if a claimed invention incorporates each and every limitation of two or more pieces of prior art, the invention as claimed fails to satisfy the nonobviousness requirement of 35 U.S.C. 103. C Correct because an invention is obvious if it is a combination of known prior art elements that would reasonably have been expected to maintain their respective properties or functions after they have been combined. D Incorrect because the Butch invention mechanizes what is taught by the combination of Cassidy and Kidd, thereby introducing a unique and nonobvious inventive concept. E Incorrect because the relevance of Kidd is only understood after first viewing the Cassidy disclosure, thereby making the combination of Cassidy and Kidd for obviousness purposes inappropriate.

ANSWER: C. Sundance, Inc. v. DeMonte Fabricating Ltd., 550 F.3d 1356 (Fed. Cir. 2008). Teaching point: A claimed invention is likely to be obvious if it is a combination of known prior art elements that would reasonably have been expected to maintain their respective properties or functions after they have been combined.

The MPEP and USPTO rules and procedure set out factual inquiries that are employed when making an obviousness-type double patenting analysis. Which of the following is not a factual inquiry that would be properly employed when making an obviousness-type double patenting determination with regard to a pending application vis-a-vis a claim in an issued patent? A Determine the level of ordinary skill in the pertinent art. B Determine the scope and content of a patent claim and the prior art relative to a claim in the application at issue. C Evaluate any objective indicia of nonobviousness of the claim of the application at issue. D Determine the differences between the scope and content of: the patent claim and the prior art determined in choice (B) above and the claim in the application at issue. E None of the above (that is, each of the above factual inquiries is properly employed when making an obviousnesstype double patenting determination with regard to an issued patent).

ANSWER: Choice (E) is the correct answer. MPEP § 804, subpart (II)(B) (1), reads, "Since the analysis employed in an obviousness-type double patenting determination parallels the guidelines for a 35 U.S.C. § 103(a) rejection, the factual inquiries set forth in Graham v. John Deere Co., 383 U.S. 1, 138 USPQ 459 (1966), that are applied for establishing a background for determining obviousness under 35 U.S.C. § 103 are employed when making an obvious-type double patenting analysis." Each of choices (A), (B), (C), and (D) is incorrect because it is a factual inquiry set forth in Graham v. John Deere Co.

If performance of the claimed method involves an application of a law of nature, which of the following are not among the factors for determining whether the method claim embodies merely an unpatentable abstract idea? A Whether the claim generically recites an effect of the law of nature or claims every mode of accomplishing that effect. B Whether the claimed method recites an application of a law of nature solely involving subjective determinations. C The extent to which the application imposes meaningful limits on the execution of the claimed method steps. D Whether one of ordinary skill in the art would understand the claimed method incorporating the law of nature to be obvious. E All of the above

ANSWER: D. A, B and C are all relevant considerations under 35 U.S.C. § 101, which is where patentability of abstract ideas arises. D relates to obviousness considerations, which is inappropriate to mix together with a § 101 determination.

John Henry is your typical paranoid inventor. His invention pertains to a method and device for preventing individuals from reading your mind. The apparatus requires copious amounts of duct-tape and tin-foil, as does the method. As far as you can tell, the apparatus is an ordinary, run-of-the-mill football helmet with tin-foil duct-taped thereto. The method requires first putting the tin-foil clad football helmet on, placing tin-foil over all windows, securing the tin-foil with duct-tape and pensively sitting in the middle of the room while humming Beethoven's 9th Symphony while watching DVDs of old X-files episodes. Notwithstanding the relative lack of merit to Henry's invention, he managed to convince the patent examiner to issue a patent. Accordingly, U.S. Patent No. 8,666,666 was awarded to Henry. One evening while Henry was at the neighborhood bar, one of the locals started talking about some "little known fact" pertaining to Thomas Edison. It turns out that Thomas Edison's first phonograph invention consisted of a grooved cylinder mounted on a long shaft with a screw pitch of ten threads per inch and turned by a hand crank. Edison used a piece of tin foil wrapped around the cylinder as a recording surface. Upon learning of this, Henry became extraordinarily agitated. He now believes that his failure to disclose this prior art during prosecution will irreparably damage the rights he obtained to his patent, which would mean the government could reverse engineer the device and figure out how to read his mind. Henry decides to submit a request for supplemental examination. Without help of a registered practitioner, Henry files a request for supplemental examination, submitting the following: I. 14 independent articles that describe the Edison tin foil phonograph, together with a separate, detailed explanation of the relevance and manner of applying them to each claim. II. A video of a segment from a History Channel documentary about the life and inventions of Thomas Edison. III. A copy of the '666 patent, together with a list of the items submitted, identification of each claim he believes must be reviewed and identification of himself as the only owner of the patent. Which of the following are inappropriate and will lead to the filing being non-compliant and no filing date being awarded? A I B II C III D I and II E I, II and III

ANSWER: D. According to 37 CFR 1.605(a), no more than 12 items may be submitted. Furthermore, pursuant to 1.605(c), a video or audio recording must be submitted in the form of a written transcript. Thus, I and II would lead to a non-compliant filing. III is required pursuant to 1.610(b).

Risedronate, the subject of the rejected claims, is a member of a group of compounds referred to as bisphosphonates. Bisphosphonates, in general, are active in inhibiting bone resorption. The first two promising bisphosphonates studied for the treatment of metabolic bone diseases, etidronate (EHDP) and clodronate, had clinical problems which prevented their commercialization. Applicant went on to conduct a significant amount of experimentation involving hundreds of different bisphosphonate compounds, but could not predict the efficacy or toxicity of the new compounds. Eventually, applicant identified risedronate as a promising drug candidate and later filed the patent application claiming the compound. The examiner rejects all of the compound claims for risedronate based on a patent to Bones, which neither claims nor discloses risedronate. Instead, Bones discloses an intermittent dosing method for treating osteoporosis and addresses the central problem seen in bisphosphonates, namely that they inhibited bone mineralization. Bones teaches the use of a cyclic administrative regimen which alleviates the unwanted side effect of anti-mineralization in patients. The Bones patent lists thirty-six molecules as treatment candidates and eight preferred compounds for intermittent dosing, including EHDP. The examiner's rejection explains the structural similarities between risedronate and EHDP; thus EHDP is a lead compound that would be modified by someone of skill in the art to achieve the claimed invention. Therefore, the claims to risedronate are obvious. This rejection is likely: A Appropriate because when an obvious modification leads to the anticipated success, the invention is likely the product of ordinary skill and is obvious under 35 U.S.C. § 103. B Appropriate because an obviousness determination is appropriate where the prior art teaches a compound having structural similarities. C Appropriate because there is no evidence of any secondary considerations that would mitigate against an otherwise appropriate finding of obviousness. D Inappropriate because even if there was a reason to select EHDP as a lead compound, there was no reasonable expectation of success given its clinical failure. E Inappropriate because the Bones patent does not claim, disclose or teach risedronate.

ANSWER: D. See Procter & Gamble Co. v. Teva Pharmaceuticals USA, Inc. Teaching point: "Where there was reason to select and modify the lead compound to obtain the claimed compound, but no reasonable expectation of success, the claimed compound would not have been obvious."

Natasha Romanova, an employee of Avenger, Inc., has created an invention that enables the management of risk relating to specified, yet unknown, future events. The specification further explains that the "invention relates to methods and apparatus, including electrical computers and data processing systems applied to financial matters and risk management." The claims filed in the Romanova patent application are directed to a concept of intermediated settlement, more specifically, the claims mitigate the risk that only one party to an agreed-upon financial exchange will satisfy its obligation. In particular, the claims are designed to facilitate the exchange of financial obligations between two parties by using a computer system as a thirdparty intermediary. In sum, the patent claims the invention using three distinct claiming techniques: (1) claiming a method for exchanging obligations (the method claims), (2) claiming a computer system configured to carry out the method for exchanging obligations (the system claims), and (3) claiming a non-transitory computer-readable medium containing program code for performing the method of exchanging obligations (the media claims). All of the claims are implemented using a computer. Which of the following rationales would be appropriate for an examiner to cite in rejecting these claims? A The method claims are not directed to a patent-eligible category of innovation and, therefore, the method claims are patent-ineligible. B The method claims are not directed to a patent-eligible category of innovation and, therefore, the method claims are patent-ineligible. Given that the method claims are patent-ineligible, and further given the system claims and media claims are simply attempts to claim the same subject matter, those claims rise or fall with the method claims. Therefore, the system and media claims are likewise patent-ineligible. C The method claims are all drawn to the abstract idea of intermediated settlement and, therefore, the claims are all patent-ineligible. D The method, system and media claims are all drawn to the abstract idea of intermediated settlement and, therefore, the claims are all patent-ineligible. E None of the above.

ANSWER: E (A) is incorrect because 35 U.S.C. 101 specifically says that processes are patent-eligible. See 35 U.S.C. 100(b) ("The term process means process, art, or method, and includes a new use of a known process, machine, manufacture, composition of matter, or material"). While it is true that the Supreme Court in Alice said that the system and computer-readable medium claims rise and fall with the method claims, (B) is still incorrect for the same reason set forth in (A). Further, (B) is incorrect because computer systems and computer-readable medium claims have been found to be patent-eligible. Having said this, computer readable media have been found to be patent-ineligible when it is possible to interpret them to encompass transitory forms of signal transmission, such as a propagating electrical or electromagnetic signal. Therefore, you will frequently see practitioners specifically reference non -transitory computer-readable media in the claims. (C) and (D) are incorrect because the fact that claims are drawn to an abstract idea does not end the inquiry. If claims are drawn to an abstract idea, it is possible that the claims are still patent-eligible if what is being claimed is significantly more than the abstract idea. With no answer choices correct, (E) is the best choice.

Humphrey Totter is an inventor, businessman and sometimes philanthropist. His latest entrepreneurial endeavor relates to a motor-controlled macro rail for close-up focus-stacking photography. It has come to his attention that there is an issued U.S. Patent to Koda Poliride, which could present problems for him if he were to move forward. In researching the Poliride patent, Totter uncovers various litigations and enforcement actions brought by Poliride. It seems he is quite litigious and vigorously enforces his rights whenever possible. As Totter continues to read the various litigation filings, he notices that Poliride has made important, although nuanced, statements that seem to conflict. For example, in the U.S. Federal District Court for the Eastern District of Texas, attorneys for Poliride have made arguments regarding the proper interpretation of the term "controller communicatively coupled," which seem to contradict the arguments made by Poliride in the U.S. Federal Court for the Northern District of California. Totter comes to your office for assistance. He is uncertain whether he will pursue this business endeavor, and he is not willing to pay your fee to challenge the patent, but he does think that the USPTO should be made aware of Poliride's contradictory interpretations of what the claims mean. He also indicates that if he does move forward, he may well want to challenge the patent at some future time. In accordance with his wishes, you file a written submission with the Patent Office. Subsequently, Trotter learns that counsel in California and Texas have raised the question of seemingly contradictory statements made by Poliride. Both district courts rejected the assertion that the statements by Poliride were contradictory. No appeal was taken in either case on this particular issue. What, if anything, must be done to notify the USPTO? A As a registered patent practitioner, you have a duty to supplement the submission made and notify the Patent Office that the district court judges have disagreed with your written assertions. B As the real party in interest, Trotter has an obligation to supplement the submission made to the USPTO to reflect later-learned information of a probative nature. C Both Trotter and you have a duty to supplement the submission made to the Patent Office. D There is a duty to continue to supplement the submission, but only if the party who filed the written submission subsequently files a request for ex parte reexamination. E There is no continuing duty to supplement the submission.

ANSWER: E. During the comment phase after proposing rules, some questioned the USPTO with respect to whether there was a continuing duty to supplement any written submission. The USPTO responded: "The statute does not impose a continuing duty to supplement any submissions made pursuant to 35 U.S.C. 301(a)(2). Should a party determine that a subsequent submission is needed, one can be filed in accordance with § 1.501."

Inventor A filed a patent application and assigned the entire interest in the application to his employer, MegaCorp. The application issued as a utility patent on July 9, 2012. In June 2014, MegaCorp's management first learns that a second inventor, Inventor B, should have been named as a co-inventor with respect to at least one claim of the issued patent. There was no deceptive intent in failing to name Inventor B in the original application. Inventor A, who is unfamiliar with patent law and concepts of inventorship, incorrectly believes that he should be the sole named inventor on the patent, and refuses to cooperate with any effort by MegaCorp to change the named inventive entity. The issued patent contains no other error. In accordance with the Manual of Patent Examining Procedure, which of the following procedures is/are available for MegaCorp to seek correction of the named inventive entity without any agreement, cooperation or action from Inventor A? A File, on or before July 9, 2014, a reissue application, made by MegaCorp only, that seeks to add Inventor B. B File, after July 9, 2014, a reissue application, made by MegaCorp only, that seeks to add Inventor B. C Request a Certificate of Correction to add Inventor B as a named inventor. D Submit in the issued patent file: a Request for Correction of Inventorship Under the Provisions of 37 CFR 1.48 that sets forth the desired inventorship change; a statement by Inventor B that the error in inventorship occurred without deceptive intention on her part; an oath or declaration executed by Inventor B; all required fees; and the written consent of MegaCorp. E A and B are each available procedures.

ANSWER: The best choice is (E). See MPEP § 1412.04. Reissue is a proper vehicle for correcting inventorship in a patent. Because correction of inventorship does not enlarge the scope of the patent claims, the reissue application may be filed more than two years after the patent issued. Answers (A) and (B) are therefore both correct, and (E) is the best response. Although a certificate of correction may be used to correct inventorship where all parties are in agreement, the facts of the question show that Inventor A is not in agreement. Choice (C) is thus not an available option for MegaCorp. Choice (D) is incorrect because the provisions of 37 C.F.R. § 1.48 are not available to correct inventorship in an issued patent.

An international application is filed in the United States Receiving Office on September 18, 2016. In accordance with the PCT and USPTO rules and the procedures set forth in the MPEP, which of the following will result in the application not being accorded an international filing date of September 18, 2016? A The description and claims are in German. B The Request is signed by a registered attorney rather than the applicant. C The sole applicant is a Canadian resident and national. D The application does not contain a claim. E The application is not accompanied by any fees.

ANSWER: The correct answer is (D). PCT Article 11(1)(iii)(e); 35 U.S.C. § 363; 37 CFR § 1.431(a); MPEP § 1810. Under PCT Article 11(1)(iii)(e), to be accorded an international filing date, an application must have "a part which on the face of it appears to be a claim or claims." (A) and (C) are incorrect. Under PCT Rule 11, if an application is not filed in the prescribed language or is filed by an applicant for which the Office to which the application is submitted is not competent, such application will be forwarded to the International Bureau which will act as receiving Office and accord a filing date as of the date of receipt in the USPTO. (B) is not correct. The Request may be signed by an attorney or agent who is registered to practice before the USPTO. In such a situation the application will be accorded an international filing date of September 18, 2016, and under PCT Article 14 an invitation to correct the defect will be mailed. See MPEP § 1805, paragraph 7; MPEP 1810, under the heading "The 'International Filing Date," second paragraph. (E) is also incorrect. Under PCT Rules 14.1(c), 15.4(a), 16.1(f), and 16bis.1 the fees may be paid at a date later than the original receipt date.

Assume that each claim 5 is in a different patent application. Recommend which, if any, of the following wording is in accord with the patent laws, rules and procedures as related in the MPEP for a multiple dependent claim. A Claim 5. A gadget according to claims 1-3, in which ... B Claim 5. A gadget as in claims 1, 2, 3, and/or 4, in which ... C Claim 5. A gadget as in claim 1 or 2, made by the process of claim 3 or 4, in which ... D Claim 5. A gadget as in either claim 6 or claim 8, in which ... E None of the above are proper multiple dependent claims.

ANSWER: The correct answer is (E). MPEP § 608.01(n), under the heading "B. Unacceptable Multiple Dependent Claim Wording." Multiple dependent claims in proper form depend on preceding claims and refer to the claims from which they depend in the alternative only. Answer (A) is incorrect. See MPEP § 608.01(n), under the heading "B. Unacceptable Multiple Dependent Claim Wording," and subheading "1. Claim Does Not Refer Back In the Alternative Only," second example. A proper multiple dependent claim must refer back in the alternative only. Answer (B) is incorrect. See MPEP § 608.01(n), under the heading "B. Unacceptable Multiple Dependent Claim Wording," and subheading "1. Claim Does Not Refer Back In the Alternative Only," fifth example. A proper multiple dependent claim refers back in the alternative only. Answer (C) is incorrect. Answer (C) reproduces the example in MPEP § 608.01(n), under the heading "B. Unacceptable Multiple Dependent Claim Wording," and subheading "3. References to Two Sets of Claims to Different Features." A proper multiple dependent claim refers in the alternative to only one set of claims. Answer (D) is incorrect. See MPEP § 608.01(n), under the heading "B. Unacceptable Multiple Dependent Claim Wording," and subheading "2. Claim Does Not Refer to a Preceding Claim," second example. A proper multiple dependent claim depends only from preceding claims.

Bob has filed an International Design application, which designated the United States, among the various contracting parties. The United States has refused registration following examination. Bob believes the examiner is wrong, and wants to further prosecution in the United States. Your advice to Bob is A File an Appeal to the Board. B File a continuation under Rule 1.53(b). C File a continuation or divisional under Rule 1.53(d). D File a CIP to add further distinguishing features to the design. E (A), (B) and (D)

Answer: (E) You may do all of the same strategic maneuvers in the US phase of the international design application that you can do in any other regular US design application, except, oddly inasmuch as it is a design, file any form of continuation or division under 1.53(d). In all other respects, the US case has a serial number, filing date, etc.

Nancy filed a Notice of Appeal at the USPTO, and followed that with an Appeal Brief. She received an Examiner's Answer mailed 6 months ago. In her view, the Appeal Brief addressed each and every argument and rejection raised by the Examiner. She got a fax yesterday from her client indicating that she should file a Reply Brief to raise a new argument against the Examiner's rejections. The new argument was not mentioned by the client previously and was not a part of the argument in the Brief already filed. In addition, the argument is not responsive to any new argument raised in the Examiner's Answer. Can Nancy have the new argument considered in this Appeal by the Board? A Yes, but she has to file an RCE first, then file a 2nd Appeal. B No; jurisdiction has passed to the Board who will not consider the new argument. C Yes, but she'll have to file a Reply Brief quickly and ask for an extension of time under 1.136(a). D No, the Examiner doesn't have to consider anything after the original brief. E Yes, but only if it can be shown that the client never read the original brief.

Answer: B. The Board only considers the record already before the Examiner. New arguments, after the Examiner's Answer and not responsive to arguments and issues therein, will not be considered.

In the Examiner's Answer that Rick was reading, the Examiner had argued the original rejections contained in the file but had also, in the alternative, argued that even if the rejections could not be sustained under 35 USC 102 as anticipatory, that 35 USC 103 ought to carry the day inasmuch as the references rendered the claims obvious. In addition, since the rigor of exact one-to-one correspondence of claim elements to disclosure is relieved under 103, the rejection should be maintained because common sense would render the invention obvious. Such an argument in an Examiner's answer is A Acceptable. B Not acceptable. C A new ground of rejection and should be so designated. D Merely an extension of the rationale already present in the case and should be sustained. E An automatic reopening of prosecution at the Examiner's election.

Answer: C. Such language would result in a new ground of rejection. The issues arising under 102 and 103 are "vastly" different. E is wrong because it is not the examiner who elects to reopen prosecution in such a situation; rather, it is the applicant.

Roberto has just finished reading the Examiner's answer mailed in response to the Appeal Brief he filed 6 months ago. Apparently, the Brief was very convincing since the Examiner has completely changed his interpretation of the cited art under 35 USC 102, but has again rejected all pending claims under 35 USC 103. Roberto is convinced this a new ground of rejection, but this has not been so designated in the Answer. Roberto would like to argue against the new rejection. His best course of action is to A Call the Examiner and advise him/her of the error. B Call the Board and advise the PTO that this issue should be addressed. C File a Reply Brief and argue against the new rejection. D File an affidavit completely addressing the new rejection. E File a Petition under 1.181 to have the new grounds of rejection so designated.

Answer: C. When an applicant only wants to submit arguments against a new ground of rejection, the most appropriate filing is a Reply Brief including the new arguments.

The Xyborg company, devoted to Artificial Intelligence, has created a new Examiner's Answer writing software for the PTO. It scans the existing file for evidence to help the Examiner bolster their arguments and make them appeal-proof. In one such Answer, the Examiner has listed as Evidence an affidavit that, although filed, was not admitted into the record by the Examiner. Such Evidence: A Is a part of the file and may be relied on by either the Examiner or the Applicant. B Should be remanded for consideration by the Examiner. C Is helpful, but can only be considered by the Board, not the Examiner. D Cannot be relied on unless admitted via petition to the Director under Rule 1.181. E Is admitted to the record by virtue of the Examiner's reliance thereon.

Answer: D. Only Evidence that has been admitted and considered may form the basis of the Appeal. Such non-admitted evidence cannot be relied on by either the Examiner or Applicant.

In the Examiner's Answer that Bev was reading, the Examiner had argued the original rejections contained in the file but had also, in the alternative, argued that even if the rejections could not be sustained under 35 USC 103 as obvious for inability to combine, that 35 USC 102 ought to carry the day inasmuch as the references rendered the claims unpatentable for a lack of novelty. A close reading of one of the references revealed all of the elements in the claims. The rigor of exact one-to-one correspondence of claim elements to disclosure could be met with a different teaching of the reference. Such an argument in an Examiner's answer is A Acceptable. B Unacceptable. C Merely an adaptation of the existing rationales. D A new ground of rejection and should be so designated. E Going to be remanded by the Board upon review.

Answer: D. Whenever a "new" teaching or portion of a reference is relied upon for a new rejection, it is likely a new ground of rejection.

If a substitute examiner's answer is written in response to a remand by the Board and further includes a new ground of rejection, the appellant must, within two months of the date of the substitute examiner's answer, A Reopen prosecution by filing an amendment under 37 CFR 1.111. B Maintain the Appeal, by filing a reply brief. C A or B. D A and B. E All of the above

Answer: E. Kind of a trick question. It is meant to be A or B. But, if any form of amendment accompanies a Reply Brief, it will treated as a request to reopen prosecution.

In the Board Decision that Crenshaw recently received, the Board's Decision had argued the original rejections contained in the file but had also, in the alternative, argued that even if the rejections could not be sustained under 35 USC 102 as anticipatory, that 35 USC 103 ought to carry the day inasmuch as the references rendered the claims obvious. In addition, since the rigor of exact one-to-one correspondence of claim elements to disclosure is relieved under 103, the rejection should be maintained because common sense would render the invention obvious. Crenshaw is very, very familiar with PTO procedure and filed a Request for Rehearing under Board Rule 41.52(a)(4). The Request was granted, and the Board designated the rejection in their decision as a new ground of rejection. Crenshaw can A Reopen prosecution by filing an amendment at the Board. B Request reconsideration of the Board rejection by the Examiner. C Reopen prosecution with the Examiner. D Request reconsideration from the Board as to their rejection. E C or D.

Answer: E. Once a new ground of rejection has been so designated by the Board, the applicant can reopen prosecution with the Examiner or, alternatively, seek reconsideration by the Board of their rejection.

You have filed a request for reissue of a patent owned by your client Megabucks. The only reason for the reissue is to correct an error in the example where a weight was described in tons rather than ounces. The examiner has issued an action rejecting all of the claims based upon a U.S. patent which was filed more than one year before the date of filing of your patent, but which issued after your patent. The reference discloses, but does not claim every detail set forth in your claims. The most appropriate response would: A Request the examiner to withdraw the rejection because its scope exceeds the scope of the reissue issues. B Abandon the reissue application. C Petition the Commissioner to require the Examiner to withdraw the rejection as improper. D File a response arguing that the rejection cannot be made in the reissue since it could not have been made in the original patent. E Appeal

B) is correct. As premised the rejection cannot be overcome

Which of the following are not false with respect to the Patent Prosecution Highway (PPH) at the United States Patent and Trademark Office (USPTO)? A The applicant requesting entry into the PPH must supply the work product of the Office of Earlier Examination (OEE). B A machine translation of the work product of the OEE is prohibited. C If a U.S. application has received a first office action on the merits. it would not be possible to enter the PPH in a continuation filed that claims priority to the earlier U.S. application. D After entry into the PPH at the USPTO, the patent examiner must reach a final decision within 12 months. E None of the above is true

Bizarre as it may be, the call of the question asks which of the follow are "not false," which is the same as asking which of the following "are true." All of (A) through (D) are false statements, making (E) the best answer. (A) is false because the PPH rules state that, if a copy of the office action from the OEE is available via the Dossier Access System, the applicant may request the USPTO obtain the copy via the Dossier Access System. If office action from the OEE is not available via the Dossier Access System, the applicant must provide a copy. If the applicant is unable to verify access via the Dossier Access System, the applicant should provide a copy. (B) is false because machine translations are allowed; however, if the translation is of such poor quality that it is difficult to understand, the USPTO may require the applicant to submit a manual translation. (C) is false. While it would no longer be possible to enter the PPH on the original U.S. application, provided all other requirements are satisfied, it would be possible to file a continuation and enter the PPH in the continuation. (D) is false because the PPH program does not require final disposition of the application within 12 months.

In accordance with the MPEP and USPTO rules and procedure, a joint inventor on behalf of himself or herself and a nonsigning joint inventor in certain circumstances may make a patent application. Which of the following is an acceptable reason for filing an application with a declaration or substitute statement signed by a joint inventor, who is not the legal guardian of the other joint inventor, on behalf of himself and the nonsigning joint inventor? A The nonsigning joint inventor refuses to join in the application. B The nonsigning joint inventor is on vacation and is temporarily unavailable to sign the declaration. C The nonsigning joint inventor is hospitalized and is temporarily unavailable to sign the declaration. D The nonsigning joint inventor is out of town and is temporarily unavailable to sign the declaration. E All of the above.

Choice (A) is the correct answer. MPEP § 409.03; MPEP § 09.03(d); and 37 C.F.R. § 1.47(a). (B) and (D) are each incorrect because MPEP § 409.03, subpart (d) states that "The fact that a nonsigning inventor is on vacation or out of town and is therefore temporarily unavailable to sign the declaration is not an acceptable reason for filing under 37 C.F.R. § 1.47." (C) is incorrect because MPEP § 409.03(d) further states that "the fact that an inventor is hospitalized and/or is not conscious is not an acceptable reason for filing under 37 C.F.R. § 1.47." (E) is incorrect because each of (B), (C), and (D) are incorrect.

Applicant receives an international search report transmitted on August 7, 2018, relative to an international application filed on April 7, 2017, with the U.S. Receiving Office, and which did not claim priority from any previously filed patent application. The applicants plan to pursue PCT Chap 2, International Preliminary Examination. In this Chapter 2 scenario, what is the maximum amount of time the applicant would have within which to amend the application and what may be amended? A August 7, 2018, and only the claims may be amended pursuant to Article 19 B October 7, 2018, and only the claims may be amended pursuant to Article 19 C November 7, 2018, and the specification, claims and drawings may be amended pursuant to Article 34 D February 7, 2019, and the specification, claims and drawings may be amended pursuant to Article 34 E February 7, 2019, and only the claims may be amended pursuant to Article 19

D is correct. Article 34 of the PCT relates to procedures before the International Preliminary Examining Authority, and grants the applicant the right to amend the claims, the description, and the drawings before the international preliminary examination report is established. The amendment made pursuant to Article 34 must not go beyond the disclosure in the international application as filed. Examination by the International Preliminary Examining Authority will start either by the expiration of the 22 nd month from the priority date, or three months from the date of transmittal of the international search report, whichever is later. In this case, three months from the transmittal of the international search report is November 7, 2018, and 22 months from the priority date is February 7, 2019. Therefore, the later is February 7, 2019. Thus, amendments made pursuant to Article 34 of the PCT must be made by February 7, 2019. Amendments may, however, also be made under Article 19 of the PCT. Article 19 deals with amending the claims before the International Bureau. The opportunity to make amendments under Article 19 is available after the applicant has received the international search report and the written opinion of the International Search Authority, and remains available until the end of 16 months from the priority date or two months after the date the report and opinion are mailed, whichever expires later. In this question, the search report was transmitted on August 7, 2018, so two months later would be October 7, 2018. 16 months from the priority date would be August 7, 2018. Thus, October 7, 2018, is later. Thus, it would be possible to amend under Article 19 only until October 7, 2018, but possible to amend under Article 34 until February 7, 2019. Thus, (A), (B), (C) and (E) are incorrect.

On March 21, 2018, you file a patent application on behalf of your client, a small company. The application contains 2 claims. Claim 1 is to a cleaning solution wherein the primary ingredient is uric acid. Claim 2 is to a cleaning solution wherein uric acid and lemons are the primary ingredients. You received an Office action dated August 19, 2019, wherein all the claims were rejected. You provide your client with a copy of the Office action. Today, August 27, 2019, the president of the client company calls you and informs you, for the first time, that what makes the cleaning solution so effective and popular with consumers is that they use lemons from citrus trees located only in Seminole County. He further explains the company wants to keep the fact that the lemons come from Seminole County a trade secret. He further informs you that they have tested lemons from all over and for reasons he can't understand (he is not himself a chemist), there is something, in his words, "peculiarly unique about the cleansing and freshening power of those Seminole lemons." Other lemons in combination with uric acid work, but not nearly as well. He finally tells you that the company has known of the peculiar and advantageous properties of the Seminole lemons since at least January 2018. You do believe that the claims as filed can be amended to overcome the examiner's rejections by adding a limitation that relates to the presence of sulfuric acid in trace quantities, which is disclosed in the originally filed specification. What advice should you give? A Advise that the application should be abandoned because the best mode has not been disclosed, which hopelessly compromises the application. B Advise that there is no longer a need to disclose the best mode, which means that the amendments you envision can be submitted and a patent obtained. C Failure to disclose the best mode is no longer grounds to invalidate claims already obtained, but best mode is still required to be disclosed to satisfy 35 U.S.C. 112 (a). Therefore, it would violate your ethical obligations to proceed with this application. D Cancel claim 2 and make the amendments to Claim 1 you believe will overcome the examiner's rejections and ask for reconsideration. E Amend claims 1 and 2 to add sulfuric acid in trace quantities to overcome the examiner's rejections and ask for reconsideration.

D is correct. It is true that, since the passage of the AIA, the lack of best mode is no longer an acceptable reason to find a patent claim to be invalid. It is also true that 35 U.S.C. 112(a) still requires best mode to be disclosed. But best mode is not a concept that exists in a vacuum. 35 U.S.C. 112(a) requires the disclosure of the best mode of the claimed invention. In Claim 1, there is no mention of lemons whatsoever. As such, the fact that, in one embodiment of the invention, the cleaning solution uses lemons, and specifically Seminole County, Florida, lemons, is irrelevant. With respect to Claim 2, however, you have included lemons and you have not disclosed that the preferred lemons come from Seminole County, Florida. We know you have not disclosed this fact in the specification because the question tells us that on the August 27 call your client tells you this for the first time. This also means that you have not engaged in any kind of ethical violation by seeking claims you know you are not entitled to receive, but it also means that you cannot ethically proceed to attempt to obtain Claim 2. (A) is incorrect because there is no need to abandon the entire application. Claim 1 is still viable. (B) is incorrect because the best mode still must be disclosed. (C) is incorrect, although it is close. The first two statements are correct, but it would not be unethical to proceed with Claim 1. (E) is incorrect because you cannot proceed with Claim 2 now that you know that the Claim should be rejected under 35 U.S.C. 112(a) for failure to disclose the best mode. Although not implicated in the question, notice that, when you do amend, you always must ask for reconsideration.

Jonathan Appleton is the inventor of a system and method of non-destructively sampling material from seeds. The methods are particularly adapted for automation, which permits greater sampling than was previously practical. By using the Appleton invention, it is possible to test every seed in the population and cull those seeds that do not express a particular desired trait. This greatly speeds up the process of acquiring the desired trait and can result in an improved final population, greater yield and a much better harvest. Appleton originally filed a first U.S. provisional application on August 26, 2008 and a second U.S. provisional application on June 15, 2009. Subsequently, Appleton filed a U.S. non-provisional patent application on August 26, 2010, which claimed priority from both the first and second provisional applications filed. This first non-provisional patent application ultimately issued as U.S. Patent No. 8,591,101 on March 21, 2013. Prior to the issuance of the '101 patent, Appleton filed a continuation-in-part application, which was accomplished on March 19, 2013. Although the CIP contained description of additional embodiments in the specification, the claims were limited to that which was disclosed as of the filing date of the non-provisional patent application on August 26, 2010. At the time the CIP was filed, a request for prioritized examination was simultaneously filed, together with the appropriate fee. A First Action in the continuation was mailed on May 22, 2013. Appleton quickly responded and achieved a Notice of Allowance on June 18, 2013, with the CIP maturing into U.S. Patent No. 8,767,892 - issuing on August 29, 2013. Appleton's main competitor, Ingles Orchard, Inc., approaches you on March 17, 2014, and presents prior art that seems to call into question the claims issued to Appleton in both the '101 patent and the '892 patent. Ingles would like to challenge these patents, but would prefer to avoid all-out litigation in the district courts. As of today, March 17, 2014, which statements, if any, do you make to Ingles Orchard? I. Post grant review can be requested against the '101 patent. II. Post grant review can be requested against the '892 patent. III. Inter partes review can be requested against the '101 patent. IV. Inter partes review can be requested against the '892 patent. A I and II B II and III C II and IV D III E III and IV

E is correct. Inter partes review proceedings apply to any patent (issued before, on, or after September 16, 2012). A petition for inter partes review may be filed after the later of either: (1) The date that is nine months after the grant of a patent or issuance of a reissue of a patent; or (2) if a postgrant review is instituted, the date of the termination of that post-grant review. Because the '101 patent issued nearly one year ago, inter partes review can be filed. The '892 patent, issued less than 7 months ago, can yet be subject to inter partes review because it never had an AIA-treated claim, even though it had a filing date post AIA! Therefore, III and IV are both correct, which makes (E) the correct answer. Post grant review is available to challenge only patents issued from applications that have an effective filing date on or after March 16, 2013. Said another way, post-grant review proceedings are only available for patents issuing from applications subject to first-inventor-tofile provisions of the AIA. The application leading to the '101 patent was filed on August 26, 2010 and, therefore, post grant review is unavailable, which means I is incorrect. Notice also that the CIP does not contain a claim that relates to the new disclosure, but rather the claims are specifically limited to the disclosure originally filed on August 26, 2010. That means that the CIP will be treated under pre-AIA and, therefore, the '892 patent is likewise not susceptible to challenge via post grant review, which means II is incorrect. Finally, although the question says that priority was claimed to the two provisional patent applications, both were filed more than 12 months before the filing date of the first non-provisional patent application. Thus, the attempted claim of priority will be unsuccessful, but ultimately irrelevant to the getting this question correct.

Monte Lee is the inventor of a pulmonary mattress and associated method of use to treat severe upper respiratory infections. The invention is used as a patient support device and includes a cover defining an interior region, a layer of three-dimensional material, and an air circulation device disposed adjacent the layer of three-dimensional material. An International Patent Application is filed in the United States Patent Office on March 15, 2013. Believing that methods of use are not patentable outside the US, Lee does not disclose the method of use. A nonprovisional patent application is filed March 3, 2014, which claims the benefit of the earlierfiled International Application. The nonprovisional patent application discloses the same invention as included in the International Application, but also includes two new embodiments. The first further comprises a vibration device located within the inner region. The second incorporates a coin-activated mechanism for the vibration device. The nonprovisional patent application contains 25 claims. Only claims 1 - 14 relate to the pulmonary mattress, Claims 15 - 18 incorporate the vibration device, Claims 19 - 22 incorporate the coin-activation mechanism to the vibration device and Claims 23 - 25 claim the method. Due to lack of funds, Lee does not pursue the International Application. In the First Office Action in the nonprovisional application, the patent examiner rejects claims 1 - 14 under 35 U.S.C. 103 based on a US patent to Taft issued in 1925, in view of a Belgian patent application filed by Claire on April 1, 2013, which published on October 1, 2014, and subsequently was used to support priority in a published PCT application that designated the US. Claims 15 - 18 are rejected under 35 U.S.C. 103 based on Taft, in view of Claire and further in view of McHenry. McHenry is an Irish patent issued on June 15, 2014, which was filed in Ireland on February 15, 2013, and which was subsequently used to support a claim of priority for an issued US patent. Claims 19 - 22 are rejected under 35 U.S.C. 103 based on Taft, in view of Claire and McHenry, and further in view of Cosmappo, which is an Italian patent issued on November 22, 2015, on an application filed in Italy on March 2, 2014, and which was subsequently used to support a claim of priority for an issued US patent. Claims 23 - 25 are rejected under 35 U.S.C. 103 based on Taft, in view of Hampton. Hampton is a UK published patent application, which published on June 15, 2014, which was filed on January 14, 2013, and which was subsequently used to support a claim of priority for an issued US patent. Which arguments can Lee successfully make to overcome these obviousness rejections? A Lee will not be able to remove any of these references. Therefore, he will need to either amend the claims or argue the obviousness rejections on their merits by pointing out that there is no teaching, suggestion or motivation to combine the references, and the claims define an invention that provides unexpected results. B Lee will be able to overcome the rejection of claims 1 - 22 by pointing out that the Claire reference is not prior art. C Lee will be able to overcome the rejection of claims 15 - 18 by pointing out that the McHenry reference is not prior art. D Lee will be able to overcome the rejection of claims 19 - 22 by pointing out that the Cosmappo reference is not prior art. E Lee will be able to overcome the rejection of claims 23 - 25 by pointing out that the Hampton reference is not prior art.

First notice that the Lee nonprovisional patent application will be treated under AIA. Although the filing date of the International Application is prior to March 16, 2013, the nonprovisional patent application contains claims not entitled to the priority date of the International Application. Thus, the entirety of the nonprovisional patent application will be examined under the AIA. Most importantly, that means that US patent applications and US patents will be prior art as of their foreign filing dates, provided that the foreign filing dates are used to support priority in those published US application, granted US patents or published PCT applications that designate the US. Therefore, the Claire reference will be prior art as of its filing date on April 1, 2013, which means that the International Application filing date of March 15, 2013, will remove the Claire reference from the obviousness rejections. We know this to be true because the Claire reference in combination with Taft (from 1925) was used to reject the pulmonary mattress claims that are supported by the March 15, 2013, International Application filing. Thus, the claims that include the pulmonary mattress plus vibration and pulmonary mattress plus coin-operated vibration, will likewise be patentable once Claire is removed; even those extended features do not obtain priority until the filing of the nonprovisional patent application on March 3, 2014. (A) is incorrect because Claire will be able to be removed as a reference. (C), (D) and (E) are all incorrect because the McHenry, Cosmappo and Hampton references each have a filing date prior to the nonprovisional filing date of March 3, 2014.

In which of the following situations may an affidavit or declaration under 37 CFR 1.131 be properly used to remove a reference from a rejection? A Where the reference publication date is more than one year before the applicant's or the patent owner's effective filing date. B Where a US patent was filed less than one year prior to the applicant's pre-AIA effective filing date and shows but does not claim the same patentable invention. C Where the subject matter relied upon is evidence under 35 USC 102(f). D Where the reference is a prior US patent to the same inventive entity claiming the same invention. E None of the above.

First, this question has to relate to pre-AIA 102 because you only use a 131 Affidavit to swear behind under the "first-to-invent" regime. (B) is correct because the reference is within the one-year period an applicant is permitted to attempt to swear behind the disclosed, but not claimed, subject matter. (A) is incorrect because you cannot swear back more than one year after a publication date because that reference would be used under 102(b). You cannot swear behind a 102(b) rejection. (C) is incorrect because 102(f) asks whether the applicant actually conceived and swearing behind will not be useful to overcome a 102(f) rejection. (D) is incorrect because this would result in a double patenting rejection either of the same-invention type, or obviousness type. In same invention type double patenting, the claim must be amended to overcome the rejection. In the obviousness-type double patent scenario, a terminal disclaimer overcomes the rejection.

Raven Hamilton is the inventor of a new gizmo that slices, dices, dashes and spins. An early prototype of the gizmo has shown to be extremely effective, and market testing suggests that there is real commercial potential. Prior to disclosing the invention, Hamilton filed a patent application in Canada, which was accomplished on February 5, 2013. Subsequently, upon receiving international interest from potential licensing partners, Hamilton filed an international patent application in Canada on January 22, 2014, which claimed priority to the earlier-filed Canadian patent application. Hamilton entered the U.S. national stage on August 5, 2015. After some early market successes, it was discovered that the spinning functionality was not appropriate to meet the long-term needs of users because the spinner was overly susceptible to wear and tear. It was further discovered that the U.S. was likely going to be the largest marketplace for the gizmo. This lead Hamilton to file a continuation-in-part application in the U.S. on October 12, 2015, which described a previously undisclosed embodiment of the gizmo having enhanced features addressing the durability issues. Certain claims filed in the October 2015 continuation-in-part specifically related to this previously undisclosed embodiment of the gizmo. On November 2, 2015, Hamilton received notice from the Canadian patent examiner indicating that certain claims to the gizmo in her originally filed Canadian application were allowable. She now would like to enter the PPH in the U.S. Because she believes that the most commercially valuable claims will come from the continuation-in-part directed to the gizmo with enhanced features, she would like to enter the PPH with the continuation-in-part application filed on October 12, 2015. Which of the following would you tell her? I. The continuation-in-part application is not eligible to enter the PPH because it does not share the priority date of the originally filed Canadian patent application. II. The PPH can be entered with respect to the continuation-in-part application, but claims to the enhanced gizmo features will not be able to be obtained. III. Because the continuation-in-part application will have AIA 102 applied, it is not eligible for entry into the PPH. IV. Because the continuation-in-part application will be examined under AIA 102, it will be possible to obtain claims to the gizmo with enhanced features via entry into the PPH. A I B II C III D IV E I and III

Here, the continuation-in-part application shares the same priority filing date of the original Canadian patent application, but only with respect to what was on file as of that date. The continuation-in-part application adds matter not previously disclosed, so that new disclosure will provoke a new priority date with respect to the new disclosure. The U.S. application for which participation in the PPH is requested must have the same earliest date, whether this is the priority date or filing date, as that of a corresponding earlier-filed application from a participating PPH office. Thus, the continuation-in-part application qualifies because it does have the same earliest priority date. Remember, however, that the whole point of the PPH is to accelerate claims that "sufficiently correspond" to allowable claims in the corresponding patent application already reviewed in the Office of Earlier Examination (OEE). The claims identified as allowable in the OEE, which in this case is the Canadian Patent Office, would necessarily be limited to the basic gizmo and not the gizmo with enhanced durability features, because the original Canadian filing only disclosed the basic gizmo. Therefore, it is possible for the continuation-in-part application to enter the PPH, but the claims would need to be limited to the basic gizmo and still be the same or similar in scope to the allowed patent claims in Canada. Thus, II is a correct statement and I is an incorrect statement. Whether AIA 102 or pre-AIA 102 applies is irrelevant to the inquiry; therefore, III and IV are incorrect.

Sam Ottawa filed a patent application in Canada on April 26, 2012, which related to his new and improved ice-hole drilling device, and which subsequently published on June 25, 2013. Ottawa subsequently filed a patent application in the United States relating to the same icehole drilling device on October 25, 2013, claiming priority to the earlier-filed Canadian patent application. The specification of the U.S. application was identical in scope to the application filed in Canada. On December 10, 2014, Ottawa received indication from the Canadian patent examiner that several of his claims relating to a solar-powered version of his ice-hole drilling device were patentable. Unfortunately, the claims originally filed in the U.S. patent application do not sufficiently correspond to the claims the Canadian patent examiner will allow. Undeterred, Ottawa now wants to request entry into the Patent Prosecution Highway in the U.S. Will his request to enter the PPH be granted? A Yes, provided that Ottawa can demonstrate that the claims he is seeking in the U.S. sufficiently correspond to the allowable claims in the Canadian patent application. B Yes, but only if Ottawa requests that priority be restored and the failure to file in the United States within 12 months of the Canadian filing was unintentional. C Yes, but only if Ottawa provides a copy of the allowable claims from his previously filed Canadian application. D No, because the U.S. application does not have the same priority date as the Canadian application. E No, because the claims he will receive in Canada do not sufficiently correspond to those originally filed in his U.S. patent application.

In order to qualify for the PPH, it is required that the application from the Office of Earlier Examination (OEE) have the same priority date as the application filed in the U.S. for which entry into the PPH is sought. Here, the application was filed in Canada on April 26, 2012, which means an application properly claiming priority could only be filed in the U.S. until April 26, 2013. Because the PLTIA did not go into effect until December 18, 2013, Ottawa does not have the ability to restore priority for up to 2 months beyond the 12-month deadline. Therefore, the U.S. application on the ice-hole drilling device does not enjoy the same earliest date of priority, which will keep it out of the PPH. (A) is not the best answer, although it does correctly identify one of the PPH requirements. Ottawa cannot enter the PPH because of the priority date of the U.S. application. If he could, he would have to demonstrate sufficient correspondence. (B) is incorrect because it erroneously states that priority can be restored even though the PLTIA does not go into effect, offering that possibility, until December 18, 2013. (C) is not the best answer, although it does correctly identify one of the PPH requirements. Ottawa cannot enter the PPH because of the priority date of the U.S. application. If he could, he would have to provide a copy of the claims allowed in Canada. (E) is incorrect, because if Ottawa could otherwise enter the PPH, he would be able to amend the claims to sufficiently correspond. Claims are considered to sufficiently correspond where, accounting for differences due to translations and claim format requirements, the claims are of the same or similar scope or amended to be of the same or similar scope (i.e., "sufficiently correspond").

Jenna Tyson filed a patent application in Canada on April 17, 2013, relating to an inflatable lounge chair with integrated radio. She subsequently filed a patent application in the United States on May 15, 2014, which claimed priority to the earlier-filed Canadian patent application. Papers were filed to restore priority, together with the applicable fee, based on the fact that Tyson mistakenly docketed May 17, 2014, as the date by which she needed to file a U.S. application. The Canadian patent application received a favorable office action on November 26, 2014, indicating that several claims were allowable. She requested participating in the PPH in the United States. The request for entry into the PPH was deemed non-compliant, and the USPTO notified Tyson that her request could not be granted because her request did not satisfy all of the requirements for entry in the PPH. Which of the following most accurately reflects the status of Tyson's attempt to enter the PPH in the United States? A Having failed to satisfy the requirements of the PPH, being un-prejudiced by any previous failed attempts, Tyson may subsequently request participation in the PPH program again. B Tyson will be given one opportunity to perfect the request in a renewed request for participation. C Tyson may ask the USPTO to suspend action on the U.S. application so that she can file a renewed request for participation in the PPH. D Because Tyson filed a timely request to enter the PPH in the United States, even if the U.S. application is subsequently examined prior to her filing a renewed request, participation in the PPH can still be achieved without the need to file a continuation. E Having failed to satisfy the requirements of the PPH, Tyson will be notified by the Patent Office that the application now must await action in its regular turn.

In those instances where the request for participation in the PPH pilot program does not meet all the requirements, the applicant will be notified and the defects in the request will be identified. The applicant will be given one opportunity to perfect the request in a renewed request for participation. Action on the application by the USPTO will NOT be suspended (37 CFR 1.103) awaiting a renewed request for participation. If the application is picked up for examination after the applicant has been notified of the defects in the request, any renewed request will be dismissed. If the renewed request is perfected and examination has not begun, the request and special status will be granted, the applicant will be notified, and the U.S. application will be advanced out of turn for examination. If the request is not perfected, the applicant will be notified and the application will await action in its regular turn. (A) is incorrect because the applicant has only a single chance to fix the inadequate PPH request. (C) is incorrect because the USPTO has stated explicitly that suspension of an application is specifically not an option for purpose of delaying examination to renew a PPH request. (D) is incorrect because, if examination begins prior to a renewed request, the request will be dismissed. (E) is incorrect because the applicant does have one opportunity to submit a new, compliant request.

On October 8, 2016, Derek built what he believed to be a new and improved frame for a oneperson, open-cockpit, lightweight, fixed-wing aircraft known as an "ultralight aircraft." In trying it out for the first time, the frame collapsed, luckily before lift-off was achieved. After further work in his garage, Derek added an additional brace member. On October 15, 2016, Derek tested the new frame with the brace member. The ultralight held together, and he was satisfied that his invention was operative and complete. Happy with the results, Derek allowed other members of his make-it-yourself aircraft club to have unrestricted use of his ultralight since at least October 19, 2016. On February 20, 2017, Derek added an individual, personalized parachute compartment. The compartment was much like a glove box in a standard automobile, but with a quick-release latch holding the compartment closed until the parachute may become necessary. The idea for the compartment with a quick-release latch came to Derek after reading an article on December 5, 2016, which described a quick-release latch and compartment used in a Cessna, which is a small, piston-powered aircraft. The compartment and quick-release latching mechanism employed by Derek was very similar to the one he read about in the article. On November 10, 2017, Derek filed a patent application at the USPTO. Two claims appeared in the application: independent claim 1, which was drawn to an ultralight aircraft with a frame and brace, and dependent claim 2, which further added the limitation of the quick-release latching mechanism to secure a compartment. Given only the facts set forth above, which rejections, if any, would lead to Derek not being entitled to a patent for the invention set forth in claim 2? A 35 U.S.C. 102(a)(2) B 35 U.S.C. 102(a)(1) C 35 U.S.C. 101 D 35 U.S.C. 103 E None of the above

Note that the question specifically asks about claim 2, not claim 1. If the question had asked about claim 1, the answer would be (B) because the invention was in use (i.e., unrestricted use given to club members) for more than one year prior to the time a patent application was filed. Nevertheless, that unrestricted use will be prior art for claim 2 as well. However, claim 2 adds more than was present in the device that was publicly used (i.e., a compartment with quick-release latching mechanism). Thus, the public use alone is not enough to render claim 2 unpatentable. Still, the public use together with the article and an ordinary glove box from an automobile would render claim 2 obvious. Thus, (D) is correct. (A) is incorrect because there are no facts in this question that suggest that anyone filed an application on basic device prior to Derek. (C) is incorrect because Derek did contribute conception. The hallmark of a 101 rejection for not being the inventor is that the applicant has taken the work of someone else. (E) is incorrect because (D) is correct.

Which of the following applicants may not file and prosecute the applicant's own case without retaining the services of a patent practitioner? A An inventor. B A corporate assignee. C All joint inventors. D One joint inventor where the other joint inventors are unavailable. E All of the above.

See 37 CFR 1.31. An applicant for patent may file and prosecute the applicant's own case, or the applicant may give power of attorney so as to be represented by one or more patent practitioners or joint inventors, except that a juristic entity must be represented by a patent practitioner even if the juristic entity is the applicant. Further, pursuant to 1.33(b)(3), all papers submitted on behalf of a juristic entity must be signed by a patent practitioner. Juristic entities include corporations or other non-human entities created by law and given certain legal rights. Thus, (B) is the correct answer. A corporate assignee must be represented by a patent practitioner

An international application designating the United States and other countries was filed in London at the U.K. Receiving Office on September 3, 1994, claiming priority from a French application filed June 23, 1994. A demand for international preliminary examination was properly made on October 14, 1995. The basic national fee to enter the U.S. was submitted to the USPTO on December 12, 1996, one year after the International Application was published in English. On March 12, 1997, in response to a Notice of Missing Parts a translation of the international application and a declaration of the inventors were submitted. The declaration was objected to by the Examiner because it did not indicate the residence address of one of the three inventors in an Ex parte Quayle action dated April 14, 1997, setting a two month period for response. A new declaration was submitted together with a request for reconsideration on May 5, 1997. The application issued as a U.S. patent on July 23, 1998. What is the effective date of the patent under 35 USC 102(e): A July 23, 1998. B December 12, 1995. C March 12, 1997. D September 3, 1994. E May 5, 1997.

Since this PCT was filed before 11/29/00, the 102(e) date for a laterissuing US patent is the day the filing requirements (under 371, a.k.a. the FOoT rule) were complete. Here, the day the last of these (fee, oath, translation) arrived was March 12, 1997. The later objection to the declaration does not move that date.

Your client informs you of the existence of a document that may bear on the patentability of one or more claims in an application you are currently prosecuting on behalf of that client. You review the document and determine that it is more pertinent than anything disclosed and that at least one claim as presented is likely unpatentable as the result of the teachings contained in the reference. The client also informs you that this document has been obtained through discovery in an ongoing patent infringement case in the United States Federal District Court for the District of Delaware. The Court in that proceeding has issued a protective order requiring this document to remain confidential. Based on this information, which of the following most accurately reflects appropriate USPTO procedure? A The document cannot be disclosed to the USPTO because of the protective order. B The document must be disclosed to the USPTO and an Information Disclosure Statement needs to be filed in due course. C Because the document is material to patentability and non-cumulative, it must be disclosed to the USPTO. The document must be marked as either "Restricted" or "Confidential" and hand-delivered to the Customer Window at the USPTO in Alexandria, Virginia. D Because the document is material to patentability and non-cumulative, it must be disclosed to the USPTO. The document must be marked "Subject To Protective Order" and it may be either hand-delivered to the Customer Window at the USPTO in Alexandria, Virginia, or mailed to the Commissioner for Patents. E Because the document is material to patentability and non-cumulative, it must be disclosed to the USPTO. The filing must include a transmittal letter, the document and transmittal letter must be marked "Subject To Protective Order" and identify the proceeding and tribunal issuing the order. It may be either hand-delivered to the Customer Window at the USPTO in Alexandria, Virginia, or mailed to the Commissioner for Patents

Situations arise in which it becomes necessary, or desirable, for parties to proceedings relating to pending patent applications or reexamination proceedings to submit to the Office trade secret, proprietary, and/or protective order materials. As in all other situations, it is incumbent upon those substantively involved in prosecution to bring non-cumulative, material information to the attention of the Office. This is true even when the material information is classified as a trade secret, or as proprietary material, or is subject to a protective order. Thus, (A) is incorrect. Trade secret material, proprietary material, and any material subject to a protective order must be clearly labeled as such and be filed in a sealed, clearly labeled, envelope or container. Therefore, (B) is incorrect. Each document or item must be clearly labeled as a "Trade Secret" document or item, a "Proprietary" document or item, or as an item or document "Subject To Protective Order," as is appropriate. The Office forbids the use of terms "Confidential," "Secret," and "Restricted" or "Restricted Data" in order to avoid confusion with national security information documents which are marked with these terms. Therefore, (C) is incorrect. The Office says that the envelope or container with the information should be hand-delivered to the Customer Service Window at the USPTO, but it may also be mailed to the Commissioner for Patents at the Office PO Box. The envelope or container must be accompanied by a transmittal letter that itself also contains the same identifying information as the envelope or container. Therefore (D) is incorrect and (E) is correct. For more on this, see MPEP 724.

An invention relates to a bicycle having a "shaped handlebar" that provides improved aerodynamic properties for bicycles. The invention is described in a pending U.S. patent application. The "shaped handlebar" is disclosed as being only "Y" shaped. The application as filed, however, contained only a single claim (i.e., claim 1) to the bicycle having a "shaped handlebar." Claim 1 was properly rejected under 35 U.S.C. 102 as anticipated by a U.S. patent to Jones, which discloses a "V" shaped handlebar on a bicycle. Claim 1 was then amended to add a bicycle wheel structure not disclosed or suggested by the Jones patent, but which is supported in the original disclosure of the application, although only in the drawings. Dependent claims 2 and 3 are added and provide further limitations to the invention. Claim 2 is dependent from claim 1 and further defines the handlebar as being "Y" shaped. Claim 3 is also dependent from claim 1 and further defines the handlebar as being "U" shaped. Which of the following statements is true? A Claim 3 is unpatentable under 35 U.S.C. 112 as being indefinite. B Claim 2 is unpatentable under 35 U.S.C. 112(d) because it does not further limit the subject matter of independent claim 1. C Claim 3 is unpatentable under 35 U.S.C. 112(a) since the description requirement is not satisfied. D Claim 2 is unpatentable under 35 U.S.C. 132 as being drawn to new matter. E Claims 2 and 3 are unpatentable under 35 U.S.C. 102 as being anticipated by the Jones patent.

The addition of a wheel structure not disclosed in the Jones patent will successfully cause claim 1 to overcome the 102 rejection based on Jones. It does not matter that the wheel structure was only supported in the drawings. Since the wheel structure was present at the time of the original filing, it will not be considered to be new matter. Now, more directly, the call of the question: Are claims 2 and 3 patentable? While claim 2 may be unpatentable because it depends from claim 1, it will not be unpatentable for the reasons set forth in (B), (D) and (E). Therefore, (B), (D) and (E) are incorrect. More specifically, the limitation added in claim 2 is a limitation that requires the handlebar to be "Y" shaped. This is supported in the original disclosure, so it is not new matter. Furthermore, it narrows claim 1 because in claim 1 the handlebar need only to be a "shaped handlebar." Therefore, the introduction of a specific shape narrows from any shape to the "Y" shape. Claim 3 will not be patentable. There is nothing on these facts to tell us one way or another whether the claim will be indefinite, thus (A) is incorrect. What we know about claim 3 is that it is dependent from claim 1, but the shape limitation inserted is a "U" shape. While this does narrow the claim, there is nothing in the question to suggest that the "U" shape embodiment has been disclosed in the original filing. In fact, the question specifically says that the only specifics given in the disclosure state that the handlebar is "Y" shaped. Thus, the written description does not support the claim, which means that the claim is unpatentable under 112(a).

Your new client Mary Williams has come to you because she is dissatisfied with her previous agent. She has provided you with a copy of her application as filed on June 4, 2011. She is uncertain what, if anything, might have been done or received by her previous attorney. She said that her previous attorney hasn't returned her phone calls for more than a year and his assistant told her that he was on an extended trip to Tibet for religious reasons. What is the first thing you should do to protect Mary's interest at the least expense? A Have Mary sign a Power to Inspect and obtain a copy of the Official Patent Office file as soon as possible. B Write to the Committee on Enrollment and Discipline and point out that the previous attorney has violated the USPTO's ethical requirements. C File a letter in the USPTO today requesting that further action be suspended until you have obtained a copy of the file. D File a substitute application and abandon the application filed June 4, 2011. E Do nothing until the previous attorney has withdrawn from representation.

The answer is (A). This is a "most appropriate" question. Since no information can be had from the previous attorney, the highest priority is to find out what, if anything, is happening in Mary's application.

Your new client Mary Williams has come to you because she is dissatisfied with her previous agent. She has provided you with a copy of her application as filed on June 4, 2011. She is uncertain what, if anything, might have been done or received by her previous attorney. She said that her previous attorney hasn't returned her phone calls for more than a year and his assistant told her that he was on an extended trip to Tibet for religious reasons. In addition to the filing of June 11, 2010 in Canada, Mary filed an application with the same disclosure in Canada on June 1, 2010, but withdrew the application three days later before it was published. No claim for priority was ever made based on the June 1, 2010, Canadian filing. Which of the following is true? A Mary cannot take the benefit of priority from the June 11, 2010 foreign filing in Canada. B Mary cannot take the benefit of priority from either the June 1, 2010 or June 11, 2010 filing. C Mary can take the benefit of priority from the foreign filing in Canada on June 1, 2010. D Mary can take the benefit of priority from the foreign filing in Canada on June 11, 2010. E Mary cannot take the benefit of the filing in Canada because the filing in Canada was not the first filing in a foreign country.

The answer is (D). 35 USC 119(C). MPEP 213.

Which of the following statements about foreign priority is false? A An inadvertent failure without deceptive intent to claim or perfect a claim for foreign priority can be cured by seeking a reissue. B The priority period for design applications is six months. C A translation submitted to establish foreign priority rights of an application not in English need not be sworn or in the form of a declaration. D A translation must be submitted to perfect priority unless the certified copy is in English. E Rather than submitting another certified copy of a priority document reference can be made to a document previously filed in another U.S. application.

The answer is (D). See 37 C.F.R. 1.55(g)(3).

Which of the following results in abandonment of the invention? A An express abandonment of a pending application. B Failure to take appropriate action in a pending application. C Failure to pay the issue fee. D Passage of more than twelve months in the case of a provisional application. E None of the above.

The answer is (E). (A) through (D) will all abandon an application, but not the Invention.

A provisional U.S. patent application covering a unique chair was filed in the U.S. Patent Office on March 14, 2010. The application claims priority from a U.K. application filed April 30, 2009. The U.K. application was published on November 23, 2010. A Utility application claiming priority from the provisional application was filed in the U.S. Patent Office on September 25, 2010. An official action in the U.S. application was sent to the applicant on November 3, 2011 setting a shortened statutory period of three months to respond. No response was filed. A proper continuation of the utility application was filed on December 14, 2011, and issued as a U.S. patent on July 5, 2013. When will the patent expire, assuming all maintenance fees are timely paid? A December 14, 2031. B July 5, 2030. C March 14, 2030. D April 30, 2029. E September 25, 2030.

The answer is (E). Of course the provisional application cannot take any priority from the U.K. application, but that does not influence the question.

Which of the follow is not incorrect? A An oath made in the United States must be made before either a judicial official or a registered patent practitioner. B The oath or declaration does not always need to identify each inventor by his or her legal name; nor does the oath or declaration always need to identify a mailing address where the inventor customarily receives mail, and residence. C An oath or affirmation made in a foreign country must be administered by diplomatic or consular officer of the United States authorized to administer oaths. D Deficiencies in an oath or declaration cannot be fixed without filing either an RCE under Rule 1.114 or a Continuation under 37 CFR 1.53. E None of the above.

The call of the question is awkward, no doubt. But sometimes you encounter awkward questions like this, as well as "which is not true" and "which is not false." Here "which is not incorrect" means we are looking for the statement that is true. (B) is a true statement because, as Rule 1.63(b) explains, the oath or declaration must identify the inventor by legal name and provide a mailing address, unless that information is supplied in an Application Data Sheet (ADS) submitted in accordance with 1.76. Thus, it is not necessary for an oath or declaration to contain this information in all instances. With respect to (A), 1.66 says: "An oath or affirmation may be made before any person within the United States authorized by law to administer oaths." Therefore, the oath does not need to be made before only a judicial officer or registered patent practitioner. With respect to (C), the answer is, likewise, too restrictive. While diplomatic or consular officers may administer the oath, so may others in the foreign country who have authority to administer such oaths. 1.66 says: "An oath made in a foreign country may be made before any diplomatic or consular officer of the United States authorized to administer oaths, or before any officer having an official seal and authorized to administer oaths in the foreign country in which the applicant may be, whose authority shall be proved by a certificate of a diplomatic or consular officer of the United States, or by an apostille of an official designated by a foreign country which, by treaty or convention, accords like effect to apostilles of designated officials in the United States. " With respect to (D), Rule 1.67 explains that deficiencies in the oath or declaration may be corrected with a the filing of a supplemental oath or declaration. 1.67(a) says: "The applicant may submit an inventor's oath or declaration meeting the requirements of § 1.63, § 1.64, or § 1.162 to correct any deficiencies or inaccuracies present in an earlier-filed inventor's oath or declaration." Since (B) is correct, (E) is incorrect

On September 15, 2016, you file an application on behalf of your client. Because you are a new patent practitioner without many clients, you do not have a deposit account and instead, as a matter of routine practice, you pay as you make filings. Ultimately, the examiner mails a final rejection in the application, dated October 1, 2018. The action sets a three-month shortened statutory period for a response. An amendment was filed on February 18, 2019, in response to the final rejection. The amendment included a petition and fee for a two-month extension of time. On March 10, 2019, an Advisory Action was mailed indicating that the amendment had been considered, but was not deemed to have placed the application in condition for allowance. On March 11, 2019, a Notice of Appeal was filed. On April 21, 2019, it was noted that a petition and fee for a third month extension of time had not been filed with the Notice of Appeal. On April 22, 2019, a petition and fee for an extension of time for the third month was filed. On which of the following dates, if any, would the filing of an Appeal Brief be considered timely? A May 10, 2019 B May 11, 2019 C June 20, 2019 D June 21, 2019 E None of the above

The first thing to notice here is that the question explains that you do not have a deposit account. This means that there can be no general authorization in the application on file because the USPTO does not accept a general authorization to charge fees to a credit card. Therefore, if you needed an extension at any time, you needed to file a request for the extension and pay for the necessary extension (i.e., with no general authorization to charge a deposit account, there is no implicit request for extensions of time). When you filed the Notice of Appeal on March 11, 2019, you needed to also file a request for a three-month extension together with the appropriate fee, which did not happen. The six-month statutory period to take action ran through April 1, 2019. The application went abandoned on April 2, 2019. Therefore, your attempt on April 22, 2019, to pay the fee and request a three-month extension was too late; the application was already abandoned. Clearly, you did not intend the application to go abandoned; thus you would be able to revive this application as having been unintentionally abandoned. But that is not what the questions asks.

Harry Selden is the owner of a patent that relates to a communications device that allows for the transmissions of intergalactic signals in near real-time. The broadest claim, Claim 1, includes a transmission device that includes a hyperbolic dish. Claim 2, the only other claim in the patent, covers a transmission device that includes a hyperbolic dish that is specifically tilted at an angle of 45 degrees in relation to the base, which continually and automatically rotates a full 360 degrees. The Selden patent issued January 18, 2010. On February 1, 2018, Salvador Harding approaches Selden and expresses interest in either licensing or acquiring the patent. While conducting due diligence, Harding's attorneys learn that Professor Daniel Olivaw of Trantor University publicly used the invention covered by Claim 1 13 months prior to Selden's filing a patent application in the U.S. After learning this, Harding expresses concern because the embodiment of the invention that he is interested in producing is one that includes a hyperbolic dish that continually and automatically rotates a full 360 degrees, but which is not specifically tilted at an angle of 45 degrees. In fact, Harding believes that the optimal tilt would be 60 degrees. Although there are not claims covering the version of the invention desired by Harding, the specification of the patent would provide support for such a claim. What should Selden do? A File a reissue application, cancel claim 1 and add claim 3 directed toward the embodiment preferred by Harding. B File an ex parte reexamination request, cancel claim 1 and add claim 3 directed toward the embodiment preferred by Harding. C File a request for a certificate of correction to cancel claim 1 and petition to add claim 3 covering the embodiment preferred by Harding in its place. D First file a disclaimer of claim 1 and subsequently file a reissue application and add claim 3 directed toward the embodiment preferred by Harding. E Nothing, because anything he would do would constitute an impermissible broadening since Claim 1 will be invalid and the new filing would be made more than 2 years of the patent issued date

The most common bases for filing a reissue application are: (1) the claims are too narrow or too broad; (2) the disclosure contains inaccuracies; (3) applicant failed to or incorrectly claimed foreign priority; and (4) applicant failed to make reference to or incorrectly made reference to prior co-pending applications. Reissue is the proper procedural tool here. But would this be an impermissible broadening reissue? Broadening reissues can only be brought within 2 years. A broadened reissue claim is a claim that enlarges the scope of the claims of the patent, i.e., a claim greater in scope than each and every claim of the original patent. If a disclaimer is filed in the patent prior to the filing of a reissue application, the disclaimed claims are not considered to be a part of the "original patent" under 35 U.S.C. 251. That is what makes (D) incorrect. Because, if you first disclaimed the claims then you would be seeking to broaden, which would not be permissible. See Vectra Fitness Inc. v. TNWK Corp., 49 USPQ2d 1144, 1147, 162 F.3d 1379, 1383 (Fed. Cir. 1998), which held that a reissue application violated the statutory prohibition against broadening the scope of the patent more than two years after its grant because the reissue claims are broader than the claims that remain after the disclaimer, even though the reissue claims were narrower than the claims that were disclaimed by the patentee before reissue. The Vectra Fitness issue is exactly what we have here. As long as claim 1 remains in the patent, the addition of claim 3 will not broaden because claim 3 is narrower than claim 1. Therefore (E) is incorrect. Ex parte reexamination is not the proper procedural tool because the defect is a patentability problem caused by public use and in ex parte reexamination, only patents and printed publications may be considered. Thus (B) is incorrect. A Certificate of Correction is not the proper procedural tool either. Certificate of Corrections fix clerical errors. Therefore, (C) is incorrect.

An application includes independent claims 1 and 2. Which of the following, in a reply to a non -final Office action, provides the proper basis for a rejection under 35 USC 112(a)? A Applicant amends claim 2 of the originally filed application by adding a limitation which was previously written only in claim 1 of the originally filed application, and one of ordinary skill in the art is enabled by the original disclosure to make and use the invention as claimed in amended claim 2. B Applicant amends claim 1 of the originally filed application by adding a limitation that was written in the original disclosure of the application, but the original disclosure does not enable one of ordinary skill in the art to make or use the invention as claimed in amended claim 1. C Applicant amends and broadens claim 2 by removing a limitation which was written in the original disclosure of the application, and one of ordinary skill in the art is enabled by the original disclosure to make and use the invention as claimed in amended claim 2. D Applicant adds new matter to the disclosure, but does not amend the claims of the originally filed application, and one of ordinary skill in the art is enabled by the original disclosure to make and use the invention as described in each of the claims. E None of the above.

The most correct answer is (B). MPEP § 2163.01. (A) is incorrect because the claims as filed in the original application are part of the disclosure, MPEP §§ 2163.03 and 2163.06(III), and claim 2 is enabled by the original disclosure. (C) is incorrect. The original disclosure enables claim 2. (D) is incorrect because although the specification should be objected to, the original disclosure enables each of the claims. MPEP § 2163.06(I). (E) is incorrect because (B) is correct.

Alexandria Lacey is the inventor of a widgerator. Lacey filed a non-provisional patent application in the United States on March 4, 2013. The originially filed non-provisional application contained claims to a method of using the widgerator and to various embodiments of the widgerator device. The patent examiner issued a restriction between the method and the device. On May 6, 2013, Lacey elected to proceed with the method claims, canceling the device claims. Subsequently, upon receiving international interest from potential licensing partners, Lacey filed an international patent application in the United States on March 3, 2014, which claimed priority to the earlier-filed U.S. patent application and which was identical in scope to the U.S. non-provisional patent application. Because development of the widgerator prototype was moving slower than anticipated, Lacey abandoned the U.S. non-provisional patent application on March 5, 2014, prior to receiving a First Office Action. Having identified the countries with greatest market potential, Lacey entered the national stage in Spain, France, Belgium and Australia, as well as in the United States. After withstanding a variety of rejections in Spain, France and Belgium, the first piece of good news Lacey received was from the Australian patent office, which granted claims drawn to a solar embodiment of the widgerator, which was disclosed in the originally filed U.S. non-provisional patent application. Will Lacey be able to enter the PPH in the United States based on patent claims being granted in the Australian national stage? A Yes, provided that the claims sought in the United States sufficiently correspond to claims granted in Australia. B No. Because there were previous rejections in Spain, France and Belgium, the Office of First Examination has rejected the claims and, therefore, entry into the PPH is prohibited. C No. Because the Office of First Filing was the United States and the patent application originally filed was abandoned, it is no longer possible to enter the PPH in the United States. D No, because in the original non-provisional patent application, Lacey responded to a restriction requirement. E No, because in the original non-provisional patent application, Lacey elected to proceed with only the method claims, and the Australian patent office has granted claims to a particular embodiment of the device.

There is no reason why Lacey could not enter the PPH in the U.S. after having received allowable claims in Australia, which is a partnering PPH office. (B) and (C) are incorrect because it is possible to enter the PPH in the United States when an Office of Earlier Examination (OEE) has allowed claims. There is no requirement that the claims be allowed in the first patent office to examine; thus, (B) is incorrect. Similarly, the PPH has been amended over the years to no longer require the allowable claims to be from the Office of First Filing (OFF); thus, (C) is incorrect. (D) is incorrect because a restriction requirement is not considered substantive examination. Furthermore, the original application has been abandoned in favor of an international application. There are no facts in the question to suggest that examination has started in the U.S. national stage. (E) is similarly incorrect because the application where election was made has been abandoned and there is no mention in the question that the method and device claims were deemed to violate "unity of invention," which is applicable to international applications instead of U.S. restriction law.

In which of the following situations would it be appropriate for the United States Patent and Trademark Office to accept an international patent application filed pursuant to the Patent Cooperation Treaty as either an International Receiving Office or directly on behalf of the International Bureau in Geneva? A All applicants are identified as residents and nationals of Germany and the international application is filed in English. B At least one applicant is a resident or national of the United States and the international application is filed in English. C All applicants are identified as residents and nationals of the United Kingdom and the international application is filed in English D The nationality and residence of at least one applicant is not indicated and the international application is filed in English. E All of the above.

This question is tricky. The key word in the question is "accept." Many of you may pick (B) because 37 CFR 1.412(a) says: "(a) The United States Patent and Trademark Office is a Receiving Office only for applicants who are residents or nationals of the United States of America." However, 37 CFR 1.421(a) says that, if there is no identification of at least one U.S. applicant, and there is at least one applicant that is either a resident or national in a PCT Contracting State or there is no residence or nationality indicated, the U.S. will forward the application to the International Bureau for processing. The United States will accept an international application for processing in the U.S. where at least one applicant is a resident or national of the U.S. The U.S. will also accept and then forward the application to the International Bureau for processing in (A), (C) and (D), which makes (E) the correct answer

Under which of the following conditions, if any, may the USPTO suspend or exclude an individual from further practice before the USPTO? A The Director may only suspend or exclude an individual from practice before the USPTO if the practitioner has had a hearing and been given the opportunity to be heard in a proceeding conducted by the USPTO. B The Director may, without notice, at his/her discretion and at any time, suspend an individual from practice before the USPTO. C The Director has the authority to place a practitioner on interim suspension if the practitioner has been convicted of any crime in a court of the United States. D The Director has the authority to place a practitioner on interim suspension if the practitioner has been convicted of a serious crime in a court of the United States. E None of the above.

Upon being convicted of a crime in a court of the United States, any State, or a foreign country, a patent practitioner must notify the Director of the Office of Enrollment and Discipline (OED) in writing within 30 days from the date of such conviction. At this point, the OED will make a preliminary determination about whether the crime constitutes a "serious crime" warranting interim suspension. If the crime is a serious crime, the OED Director must provide the USPTO Director proof of the conviction and a complaint against the practitioner predicated upon the conviction of a serious crime. The USPTO Director then has authority to place a practitioner on interim suspension after hearing the request for interim suspension on the documentary record. Therefore, (D) is correct. (A) is incorrect because the Director may suspend the practitioner after reviewing the documentary evidence provided by the OED Director without the practitioner being given a formal opportunity to be heard. This seeming lack of due process is due to the fact that the practitioner has already been afforded due process by the tribunal in the underlying criminal matter. This is a form of reciprocal discipline that allows the Director to immediately suspend practitioners who have been convicted of a serious crime. (B) is incorrect because it is far too strong. The Director does not have the ability to suspend a practitioner at any time for any reason and without notice. (C) is incorrect because an interim suspension is only appropriate where the practitioner has been convicted of a serious crime. In fact, the pertinent rule specifically says: "If the crime is not a serious crime, the OED Director shall process the matter in the same manner as any other information or evidence of a possible violation of any USPTO Rule of Professional Conduct coming to the attention of the OED Director." (E) is incorrect because (D) is correct.

Gabriel Kaplan, a citizen of Canada and resident of Toronto, is the inventor of a new blackboard with a mirror system that incorporates an integrated laser stunning device. The device can be used by teachers to keep students awake, or otherwise maintain a disciplined learning environment. When facing the board, the teacher may glance at one or more of the mirrors and notice a student dosing, or perhaps engaging in mischief. With a triple-tap to the board with a single finger, a laser will fire, stunning the student. Kaplan first files a Canadian patent application on January 2, 2013. Subsequently, Kaplan files a US patent application on January 1, 2014, claiming priority to his Canadian filing date. At the time he files the US patent application, Kaplan also files papers to opt out of publication in the United States. The Canadian application publishes on June 2, 2014, and the Canadian patent issues on September 22, 2014. On March 14, 2015, the US patent examiner rejects all claims in the Kaplan application. Having determined the US is not a market worth exploiting, Kaplan does not file a response and the US application goes abandoned on September 15, 2015. Which of the following would be the earliest date the Kaplan disclosure could be used as prior art against others? A January 2, 2013. B January 1, 2014. C June 2, 2014. D September 22, 2014. E September 15, 2015.

When a US patent, a published US patent application, or WIPO publication of a PCT application claims priority to a foreign filing, the date the reference will be prior art is as of the foreign filing date. Thus, this question turns on whether there was a publication of a US patent application, because there is no mention of a PCT filing, and the US patent application was abandoned, which means that there will not be a US patent granted. The question says that Kaplan opted out of publication in the US, but this is not possible. An applicant may opt out of publication at the time they file a nonprovisional patent application, but by doing so, they waive their right to file a foreign application. Here, Kaplan had already filed a Canadian application; thus, it is not possible to opt out of publication in the US. The Kaplan US application will publish and the Kaplan reference will be prior art as of its Canadian filing date, which is January 2, 2013.

Today is January 7, 2018. It is 6pm Pacific Time as you sit in your office contemplating what you must do on an important matter for a new client. A patent application was filed in Germany on January 7, 2017. The German filing included a specification, claims and drawings. Your client tells you that he absolutely wants you to file a U.S. patent application claiming the benefit of the German filing date, but despite your continued and repeated warnings over the last several months, the client has not provided you with an English translation of the German application because he has been tied up with other business. Your client is now gone away on vacation for two weeks and is unreachable. Given these facts, which is the best course of action to choose? A File a copy of the German application. B File a copy of the German specification. C You must obtain and file an English translation of the application. D You must obtain and file an English translation of the specification. E You do not need to do anything at this point because priority can be claimed up to 14 months after the filing of the German application.

While the best course of action would be to have obtained an English language translation of the application, that is no longer realistically possible given the hour. Furthermore, you do not absolutely have to file an English translation, thus (C) and (D) are unrealistic and too strongly worded. You can file an application in a foreign language to obtain a filing date. You would then have to file an English translation in response to a Notice of Missing Parts. Many may be tempted to say that (E) is correct because of the Patent Law Treaties Implementation Act (PLTIA), which went into effect on December 18, 2013. As a result of the PLTIA, it is possible to claim priority to a previously filed application for up to 14 months. In order to take advantage of the additional two-month period, the delay must have been unintentional. Here, the facts do not suggest an unintentional delay. In fact, the delay here suggests intentional delay, or at least that the delay may ultimately be characterized as something other than unintentional. The client was apprised of the need for a translation, promised to provide one, but was focusing on other business matters the client presumably deemed to be of greater importance. Furthermore, going away on vacation for two weeks prior to the deadline does not suggest the type of unintentional delay normally recognized by the USPTO. This is not to say that the delay could be appropriately characterized as unintentional, but the facts of the question do not clearly indicate the delay was unintentional so there is a risk. Thus, at the very least, (E) is not the best answer. With respect to (B), while it is now possible to file an application without claims or drawings in order to obtain a filing date, the USPTO has specifically cautioned against doing this and has said that, while this is possible, it is still best practice to file a complete application. While the PLTIA no longer requires claims and drawings at the time of filing, the failure to include claims and drawings raises concerns with respect to new matter, particularly with respect to drawings. While it is theoretically possible to add drawings that are supported by a specification, drawings typically (if not nearly always) disclose more than do the accompanying written text. Thus, (B) is not the best answer. The best answer is to file a copy of the entire German application. Notice that this would need to occur prior to midnight at the USPTO in Virginia if the filing is accomplished through EFS. It may be accomplished up until midnight local time if the filing is accomplished via U.S. Postal Service EXPRESS MAIL.

Which of the following statements about an Application Data Sheet are correct? A The Office will review the inventor's oath or declaration to determine if the bibliographic information contained therein is consistent with the bibliographic information provided in an application data sheet. B An unsigned application data sheet will be treated only as a transmittal letter unless it has been filed by an patent practitioner who is "of record" in the application. C Information in a previously submitted application data sheet, inventor's oath or declaration may be corrected or updated until the Notice of Allowance has been transmitted. D An application data sheet must be titled "Application Data Sheet." E All of the above.

With respect to (D), 37 CFR 1.76(a) specifically requires that an ADS be titled "Application Data Sheet." (A) is incorrect because 37 CFR 1.76(d)(3) explains that the USPTO will not ordinarily conduct a review to determine whether the information on the ADS matches the information submitted as part of an oath or declaration. (B) is incorrect because 37 CFR 1.76(e) explains that an ADS must be signed in compliance with 37 CFR 1.33(b) and that an unsigned ADS will only be treated as a transmittal letter, not as a properly filed ADS. There is no exception for an ADS filed by a patent practitioner of record. (C) is incorrect because 37 CFR 1.76(c) explains that information submitted in an ADS, oath or declaration can be corrected until the issue fee is paid. However, there are some exceptions. Inventorship changes must comply with the requirements of §1.48, foreign priority and domestic benefit information changes must comply with §§1.55 and 1.78, and correspondence address changes are governed by §1.33(a). (E) is incorrect because only (D) is true

Benjamin Bampion is an inventor that you represent with respect to a utility patent application filed on April 15, 2016. You receive a First Action on the Merits from the Patent Office with a mailing date of November 28, 2018. You report the Office Action to Bampion, who becomes immediately dejected. He doesn't have the funds to proceed and it looks like it will be an uphill battle to obtain a patent, so he tells you that he will almost certainly allow the application to go abandoned. On Tuesday, May 28, 2019, Bampion is standing at the door to your Laguna Beach, California Office as you arrive in the morning. He is extremely upbeat, which is unusual for him. He tells you that he has recently won $10,000 in the Powerball Lottery, and he also has an investor who is willing to front the money for manufacturing and distribution if he can get a patent - any patent. He gives you your fee in cash and asks you to prepare an Office Action Response. You indicate that you should have time today to prepare the Response. Bampion leaves. Upon firing up your computer, you realize you have several statutory deadlines and today is the last day to take action on the Bampion application. You proceed to work diligently and at 8:30pm you have finally finished the Bampion Response, the last thing you needed to accomplish. In order to have the Bampion Response filed timely what could you do? I. File the Response by Facsimile. II. File the Response using EFS Web. III. File the Response by Express Mail. A I B II C III D II or III E I, II or III

With respect to I, A Response to an Office Action may be transmitted via facsimile to the United States Patent and Trademark Office. The MPEP uses an example where a facsimile transmission begins day 1 at 8:45pm Pacific Time but concludes at 9:05 am Pacific Time the next day. The date of the facsimile would be day 1 if a Certificate of Transmission were used, because an Office Action Response is not on the list of proscribed documents at 1.8(a)(2). Therefore, it can take the benefit of a Certificate of Transmission. With respect to II, EFS Web can be used to file an Office Action Response and will be granted a date as of the date it is received, which in this case would be timely. With respect to III, any correspondence received by the USPTO that was delivered by the "Express Mail Post Office to Addressee" service of the United States Postal Service (USPS) will be considered filed with the USPTO on the date of deposit with the USPS. Although it is not mentioned in the answers, note that you could have also sent the response via first class mail with appropriate postage and a certificate of mailing.


Conjuntos de estudio relacionados

Fundamentals of Nursing - Taylor - Ch 1

View Set